2019 Spring SKKU Math History (수학사) (Record of our class)
사회수학 : 400년의 파란만장
수학사 (Flipped/PBL Action Learning)
Prof : Sang-Gu LEE
Students: 강민석;김기헌;김윤수;이헌수;조영은;최민수;
(2019, Spring) SKKU MH(수학사) Lectures
사회수학 : 400년의 파란만장 강의
- Lectures of Prof. - Student Presentations
수학사-1강-Intro 사회(社會)와 수학(社會數學) http://youtu.be/2FBIdAla8jI
수학사-2강-동양수학과 서양수학의 만남 https://youtu.be/vBKfwF4SrUg
수학사-3강-대포의 탄생과 미적분학 https://youtu.be/a4Cyq_8icpQ
수학사-4강-전쟁-프랑스-화법기하학 https://youtu.be/30_tpCCDSaw
수학사-5강-확률론 https://youtu.be/oseQFNsc3sA
수학사-6강-통계학의 탄생 https://youtu.be/3wh7RSbsicQ
수학사 7강 보험의탄생 런던대화재 https://youtu.be/UqHpPPeQ3jw
수학사 8강 위상수학의 탄생 https://youtu.be/BkHQtlmk8Mk
수학사 9강 마방진, 추측통계학(stochastic)의 탄생 https://youtu.be/8MGQGwi21yI
수학사 10강 Midterm PBL Talk https://youtu.be/-XynD2LebXo
수학사 11강 대항해와 세계지도, 사영기하학(변환법) https://youtu.be/RQlB8k17FMY
수학사 12강 2차세계대전과 최적화이론 9장 https://youtu.be/i9mUh-mqn4o
수학사 13강 계량학의 시작 10장 https://youtu.be/8wTg1B_evXg
수학사 14강 암호학-퍼즐 11장 https://youtu.be/r5NMfINC1Vw
수학사-15강-법과수학-12장 https://youtu.be/KZKu9dGPn5c
수학사-16강-문학과 수학 (문문법)-13장 https://youtu.be/9H2Y1bLWw24
수학사 17강 Epilogue 수학용어 14장 https://youtu.be/OlMR6Z-gTqk
2019 S MH 수학사 PBL 발표 1 최민수 김윤수 https://youtu.be/sIOWSaSkB3g
2019 S MH 수학사 PBL 발표 2 이헌수 강민석 https://youtu.be/leW5JgRTd3o
2019 S MH 수학사 PBL 발표 3 김기헌, 조영은 https://youtu.be/jkBM5I2BiqA
(Final) PBL Report, Spring, 2019
목차:
1주 -Intro
2주차 수업: 아라비아 등 고대 수학의 역사(미분의 시작을 알린 대포, 패러독스 등)
- 동서양 수학의 만남, 필산법, Abacus
- 대포의 탄생과 미적분
- 아라비아(과거의 학문선진국), 중세시대에 유럽은 암흑시대를 겪어 문화의 침체가 일어난다. 이 시기에 그리스 수학의 중요 부분이 아라비아에 전달됨으로써, 인류가 세계의 수학 문화를 보존할 수 있었다. 아라비아에 동방의 인도, 서방의 그리스 문화가 모두 모이게 된다.
- 인도의 대수학 & 그리스의 기하학
- 대포를 통한 미적분학의 성장
3주차 수업: 전쟁과 수학, 투영도
- 수학자가 전쟁에 참여한 예
- 수학은 전쟁의 필수조건
- 투영도의 탄생
4주차 수업: 확률론(도박에서의 수학 등)
- 이탈리아 해운항의 전통
- 확률에 대한 기초 지식
5주차 수업: 통계학-사회 부흥의 실마리(보험법)
- 자료로 미래를 예측하다.
- 런던 대화재 피해에 대한 반성에서 생긴 보험법
6주차 수업: 위상수학
- 산책로에서 탄생한 위상수학(한붓그리기)- 쾨니히스베르크 거리
- 위상동형
- 불가사의한 위상수학의 세계
·뫼비우스의 띠- 공장 컨베이어 벨트, 에스컬레이터 손잡이에 활용
·클라인 병- 현실 세계에서 존재 불가능한 4차원 구조
·토러스
7주차 수업: 지도와 회화 연구에서 나온 변환법
-구면이나 입체물을 평면에 표시
·콜럼버스가 지구가 둥글다는 것을 확신->대항해 시대 시작->지도 제작 경쟁->투영법(네덜란드에서)
·이탈리아의 레오나르도 다 빈치의 투시법 고안-> 독일로 이어짐-> 기하학 발전 기여
-변환의 이용과 효용
·합동변환, 상사변환(축소, 확대), 사영변환, 아핀변화, 위상변환
-변환을 통일적으로 통합하는 시점
8주차 수업: 시험기간
9주차 수업: 최적화 이론 & 야외수업
-최적화이론의 탄생
·독일의 U보트, 일본의 자살특공대에 대한 대책
-경영과학의 성립과 종류
·미국의 과학팀 활약->함대의 피해가 30%이하로 줄음.
- 컴퓨터를 이용한 수학
·경영과학
10주차 수업: 계량학
-수량화의 필요와 연구
-계량화 사회의 건설
·15세기 대항해 시대 전-각기 다른 도량형, 자연적 척도 이용
·이후- 공통 기준의 인공적 척도 사용(m)
-계량학과 발전
·국제 단위계 통일(SI단위계)
·측정기구 발전, 모든 영역 수량화
11주차 수업: 암호, 법과 수학
-일본 육군의 나가노 학교, 미국의 CIA, 구소련의 KGB 뿐만 아니라 기업에도 비밀유지를 목적으로 하는 조직 필요(ex. 인사팀)
-범죄행동을 저지를 때 미리 암호로 범행 계획을 알리는 연쇄살인자 점차 증가
-현대사회는 정보의 시대-> 정보를 지키는 것=경쟁력-> 암호의 중요성
12주차 수업: 문학과 수학
-문문법: 문장의 특징을 찾아 유사성 발견하는 것
ex) 토마스 메리엄은 문문법을 활용하여 작가불명의 작품 몇 개가 셰익스피어의 작품임을 밝힘
-니콜라스 부르바키: 프랑스의 10여명의 수학자 집단->교재 집필->프랑스 수학 성장
-소설 <이상한 나라의 앨리스>의 작가는 실제 옥스퍼드 수학과 교수(기호 논리학)
13주차 수업: 새로 도입된 외래 수학용어
-일본의 수학용어 변천
·일본은 중국과 우리나라(백제)를 통해 수학을 전달 받음
·19세기 말 유럽의 천문, 조선, 과학, 전쟁에 필요한 고급 실용수학이 수입
-수학의 학제 간 연구
13주 – 컴퓨터 수학
- 중국, 일본의 수학
14주 – 동아시아 수학사
15, 16주 : Final PBL 보고서 발표와 동료평가
2019 S MH 수학사 PBL 발표 1 최민수 김윤수 https://youtu.be/sIOWSaSkB3g
2019 S MH 수학사 PBL 발표 2 이헌수 강민석 https://youtu.be/leW5JgRTd3o
2019 S MH 수학사 PBL 발표 3 김기헌, 조영은 https://youtu.be/jkBM5I2BiqA
(1) State what you learned from the first Chapters.
-수학사를 공부하면서 자연스럽게 과거에 일어났던 전쟁에 대해서도 배웠는데, 이 과정에서 우리가 무심코 일상생활에서 사용하는 단어 중에 이러한 전쟁에서 유래한 외래어들이 꽤 많다는 것을 배웠다. 2주차 수업에서 대포를 통한 미적분학의 성장에 대해 학습할 때는 우리가 일상에서 흔히 사용하는 단어인 ‘무대포’가 일본의 대포에서 유래된 말임을 알게 되었고, 3주차 수업에서는 ‘마지노선(MaginotLine)’의 단어유래에 대해 알게 되었다. 제1차 세계대전에서 대포 공격을 잘 막아낸 독일의 지그프리트 방어선을 참고하여 제2차 세계대전에서 프랑스가 만든 방어목적 성벽의 이름이 ‘마지노 선(Maginot Line)’이다.
이처럼 수학사 시간은 역사, 언어 등 다양한 분야에 대한 지식을 동시에 얻을 수 있다는 점에서 굉장히 유익한 과목이라고 생각한다.
-개인적으로 6주차 위상수학에 대한 내용이 가장 흥미로웠습니다. 일반인이라면 산책, 그 외에는 별 의미를 부여하지 않을 쾨니히스베르크 거리에서 한붓그리기를 떠올리고 그에 대해 증명을 시도한 수학자의 발상이 정말 독특했습니다. 위상수학 수업 시간에 위상동형이라는 용어에 대해 처음 배웠고, QnA 활동을 통해서 상식으로만 알고 있던 ‘뫼비우스의 띠’, ‘클라인 병’이 각각 어떻게 활용되는지 생각하고 고민해보는 시간을 가졌습니다.
-5주차 수업시간에 보험법이 처음 시작된 배경을 배웠습니다. 1666년 9월 2일 새벽, 빵집에서 발생한 불이 번진 런던 대화재 사건 이후, 새로운 도시 계획을 실시하면서 보험법이 시작되었습니다. 당시 대화재 이후, 런던의 찰스 2세가 화재에 관하여 내린 칙령(ex. ‘시내의 건물은 벽돌이나 돌로 만들어야한다.’,‘템스 강 연안에 집을 세우는 것은 소화에 방해가 되므로 금지한다.’)에 대해서도 배웠고, ‘런던 대화재 기념비의 높이가 빵집에서 떨어진 거리인 62m와 같다’라는 상식도 알게 되었습니다.
한국의 근대와 현대의 수학교육에 대해서 배웠습니다. 주로 갑오개혁 이후 부터의 수학 공교육과 일제 강점기때의 교육, 또한 광복 이후와 초중등 교육기관과 고등교육기관이 어떻게 수학을 가르쳤었는지 등을 배웠습니다. 그리고 각 시대별 수학 교재와 그것들의 특징을 배웠습니다. 추가적으로 일본이 임진왜란에서 찬탈해간 산학 교재를 이용해 급격하게 수학을 발전시킬 수 있었다는 것 또한 배웠습니다.
17세기에 영국 런던에서 전염병을 관리하기 위해서 근대 통계학이 탄생했습니다. 그 전에는 산수는 산술이라 불려졌을 뿐이였지만 시간의 변화나 사건의 흐름에 따라 변화하는 것을 파악하게 된 것이 근대 통계학의 큰 특징입니다. 전염병으로부터 시작되었지만 영국은 사회과학으로 그것을 전파시켜서 크게 활용을 해내었습니다. 보험의 기초적인 형태는 동물들이 겨울을 대비해서 식량을 모으는 것에서도 찾아볼 수 있고, 계와 같은 형태 또한 많았습니다. 하지만 통계학과 확률론을 이용해서 대비를 한 것은 런던 대화재가 계기입니다. 불에 탄 집을 재건하고 수리하기 위해서는 얼마가 들지와 화재가 일어날 확률과 빈도를 체계적으로 적용해 초기의 보험형태를 이루었습니다. 그리고 천문학자 헬리는 대수의 법칙을 이용해서 생명표를 작성해 연령대별로 내야할 보험료를 수학적으로 계산해서 생명보험을 만드는 것에 기여했습니다.
수학 퍼즐은 좋아하는 사람이 꽤 많은데 가장 유명한 수학 퍼즐 중 하나인 쾨니히스베르크의 다리 문제는 오랫동안 난제였습니다. 일곱 개의 다리를 단 한번씩만 건너면서 모든 섬을 산책할 수 있느냐는 문제인데 이것을 위대한 수학자 오일러가 증명에 성공했고 여기에서 한붓그리기가 나왔습니다. 이것이 위상수학의 근간이 되었습니다. 이것을 해결하는 데에 오일러는 점과 선의 존재와 개수 연결상태만을 중시하는 그래프를 그렸는데 이것은 유클리드의 기하학과는 완전히 다른 내용입니다. 여기서 발전한 위상수학은 4색문제, 뫼비우스띠를 이용한 컨베이어 벨트 등 다방면에서 연구되고 있습니다.
변환법은 물건등을 목적에 맞게 그림등으로 나타내거나 할때 사용합니다. 대표적으로는 지도 등이 있었고, 여기에서 투영법 투시법 등이 발전했고 이것이 구면기하학, 화법기하학 등으로 발전하였습니다. 요즘 볼 수 있는 일상적인 예로는 길안내도, 기차역의 요금표, 버스의 노선도 등이 있습니다. 최적화 이론은 2차세계대전때 공격에 대비하기 위해 창설되었으며 최소의 노력으로 최대의 결과를 내려고 하는 문제를 다룹니다. 2차대전때 그 문제에 대해서 군인이 아니라 수학자가 엄청난 성과를 내었습니다. 그리고 미국의 과학팀은 가미카제에 대응하기 위해서 공격받았을때의 대응법, 대피하는 법등을 통계론과 확률론에 기반해서 최적의 방어법을 찾아내서 뒤에는 피해가 30퍼센트 수준으로 줄었다고 합니다. 요즘같이 자본주의 사회에선 최고 효율을 내기 위해서 최적화 이론이 많이 사용됩니다. 대표적으로 선형계획법(혼합비료의 배분), 창구 이론(대기 행렬, 버스나 열차의 운행횟수), 게임이론(동업회사의 판매전략), 네트워크 이론(가구 배치) 등이 있습니다.
암호론은 전쟁에서 유래되었지만 현재도 정치, 외교, 기업, 통상 등 모든 분야에서 필요합니다. 암호 풀기는 범죄를 예방하기 위해서도 중요합니다. 암호에는 규칙이 있기 때문에 그것을 해독하기위한 변환표, 난수표 등을 알아내는 것이 중합니다. 계량학은 인간 사회가 고도화되고 체계화 되면서 측정을 정확하게 할 필요가 생겼고 그리하여 발전하게 되었습니다. 그래서 단위를 통일하게 되었는데 대표적으로 1미터를 어떻게 기준세울것이냐가 큰 논란이였습니다. 그래서 정립된 단위는 대표적으로 미터, 킬로그램, 초, 암페어 등이 있습니다. 현대 사회는 거의 모든 것들이 계량화되고 수량화 되면서 계량학의 중요성이 커지고 있습니다.
법은 사실 수학과 관련 없어보이지만, 재판은 수학과 같다고 할만큼 비슷한 논리체계나 구성을 가지고 있습니다. 수학이 공리 정의 증명 결론 등의 구조를 가지고 있는 것처럼, 재판도 헌법 판례 논지 판결 등의 구조를 가지고 있습니다. 하지만 약간 다른 점이 있는데 재판에서는 비슷한 사건이라도 판결은 달라질 수도 있고 그렇기 때문에 법률은 포괄적인 해석이 가능하게 문구를 정하게 됩니다. 또한 미국 헌법은 유클리드 원론을 기반하여 만들어졌다고 할 만큼 법과 수학은 관련이 높습니다.
문문법은 사실 수학과 관련이 없어보이는 영역이지만 수학이 발전해나가면서 연관이 생기고 있습니다. 대표적으로 누가썼는지 표절인지를 밝혀내는 것에 수학의 통계론이나 패턴 등을 이용해서 분석하는 것이 있습니다. 이상한 나라의 엘리스의 저자는 기호논리학의 저자일 만큼 수학과 교수일 만큼 의외로 문학과 수학은 가까운 면이 있습니다.
외래 수학용어는 수학자체가 서로 교류하고 외국에서 들어오고 나가고 하는 경우가 많다 보니 나라나 번역자마다 용어가 다른 경우가 많아서 어려움이 많다는 것을 말합니다. 수학자들이 새롭게 정립했지만 일반인들이 계속 다른 말을 쓰다보면 결국 일반인들이 쓰는 말로 정립되는 경우도 많습니다. 하지만 요즘 같은 새로운 용어가 계속 생기는 시대에는 번역을 한다고 해서 그 의미가 온전히 전달될 것이라고 기대하기는 힘듭니다.
동아시아 산학이 오랜기간 세계 최고 수준을 유지했습니다. 2000년전에 이미 연립 방정식의 해법 및 양수와 음수의 계산법인 정부술을 개발하고, 700년 전에 이미 임의의 고차방정식 해법을 발견했었습니다. 피타고라스의 정리의 구고현의 정리 증명은 가장 아름다운 증명으로 손꼽힙니다. 17세기까지만 하더라도 동아시아와 서양은 수학의 수준차이가 없다고 여겨집니다. 하지만 서양수학을 접하고 더 이상 배울것이 없다고 느낀 동양 수학자들은 발전을 멈추게 됩니다.
고대 문명의 발상과 함께한 전 세계 각 지역의 고대의 수학에 대한 정보, 그리고 수학의 전파 과정과 그 중요성이 대두되기 시작한 계기를 배웠습니다.
(2) State what you can tell after you studied the first Chapters.
이제 조선시대부터 현대까지의 한국 수학교육의 대략적인 흐름에 대해서 남들에게 설명할 수 있습니다. 그리고 갑오개혁과 일제 강점기때 어떤 식으로 교육이 이루어 졌는지에 대해 말할 수 있습니다. 또한 광복 이후나 625전쟁 이후로 대학교육이 어떤 식으로 이루어졌는지, 강사들은 어디서 구하고 어느 정도의 교육 수준을 갖춘 사람들이 교단에 섰는지 말할 수 있습니다. 조선이 일본의 수학 발전에 끼친 영향에 대해서 말할 수 있습니다.
단순히 숫자들을 정리해 둔 것과 근대의 통계학이 과연 어떻게 크게 다른 지를 설명할 수 있습니다. 보험의 기본인 위기 대비와 실제의 보험과는 어떤 차이가 존재하고 어느 정도의 차이가 있는지를 설명할 수 있습니다. 위상수학의 시초가 무엇인지, 그래프를 어떤 식으로 단순화 시킬 수 있는지를 말할 수 있습니다. 변환된 것을 보고서 이것이 어떤식으로 변환된 것인지, 그것의 장점을 설명할 수 있습니다. 암호가 어떤식으로 생성되었는지 역할이나 규칙은 어떤것들이 있는지를 설명할 수 있습니다. 계량의 단위들이 어떤 것이 있는지, 미터법은 어떻게 정의되어왔는지 계량단위를 정립하는 것이 왜 좋은 등을 설명할 수 있습니다.
법과 수학의 유사성과 차이점등을 설명할 수 있습니다. 현대 법구조와 유클리드 원론의 유사점을 설명할 수 있습니다. 문학의 분야에서 수학을 이용해서 표절등을 잡아내는 방식등을 설명할 수 있습니다. 외래어가 들어오면서 수학에 끼치는 영향, 바뀌지 않는게 나은 외래어 등이 존재함 등을 설명할 수 있습니다.
동아시아 수학이 왜 대단하였는지 어떤 면에서 서양수학에 뒤처지지 않았는지 말할 수 있습니다. 어떤 면에서 동양수학이 늦어지게 되었는지, 어떤 식으로 어느 부분에서 격차가 벌어졌는지 말할 수 있습니다.
학기 초반, 그리스의 수학과 아라비아 수학이 나오고 필산법, Abacus 등을 배웠다. 이때까지만 해도 우리가 흔히 책에서 접해왔던 서양 중심의 수학사, 이집트와 그리스시절 아라비아를 지나 근대로 들어서 뉴턴, 오일러, 가우스 등의 인물들이 나오고 시간의 흐름에 따라 수학의 변천사와 인물들에 대해서 배울 것이라 생각했다. 하지만 이후부터 나의 생각과 달라지기 시작했다. 수업시간에 사용 하던 교제는 교수님께서 일본 책을 한글로 옮겨온 책이었다. 당연히 초반에는 세계수학사 이야기로 잘 나가다가 후반부로 갈수록 ‘수학의 발전과 그에 따른 일본의 변화’를 공부한다는 생각이 들었다. 교수님께서도 이를 정정하고 싶어하셨지만 원작자가 반대하여 못하셨다. 하지만 이미 학기초가 조금 지난 시점부터는 교제는 그저 학습도구중 아주 작은 것 하나에 불과했다. 가장 큰 변화는 Q&A게시판 사용이었다. 3월달, 처음 몇 번의 수업을 지금까지 수업을 듣던 것처럼 조용히 듣고 고개를 끄덕이며 넘어갔는데, 시간이 지남에따라 Q&A 게시판을 점점 활발하게 사용하게 되었고 교제에 있던 내용에서 생긴 질문점을 중심으로 또 다른 지식을 쌓아 나가기도하고 교수님께서 올려주신 자료들을 보고 그것을 토대로 또 새로운 지식을 쌓아나가기도 했다.
대포에 대해 이야기하며 미적분에 대해 알아보고 확률론을 이야기하며 블랙잭과 나이팅게일을 알아보고 법과 수학을 이야기하며 판결 프로그래밍을 이야기 하는 등 교제에서 얻은 주제로 다양한 질문과 궁금점을 가지며 스스로 다른 주제들을 찾았다.
교제에서는 동아시아의 수학사에 대해 많은 것을 얻었다. 일본저자라서 그런지 동아시아의 수학 (중국, 일본)이야기를 다른 일반적인 책들보다 비중있게 다뤘고 많은 것을 알 수 있었다.
<유클리드 원론>
세계 최초의 수학 교과서로 고대 그리스의 수학자 유클리드가 쓴 책이다. 여러 공리와 정리에 대한 내용이다. 이후 로마시대로 접어들면서 실전되었다가, 로마를 침공한 아랍인들에게 발굴되어 아랍의 수학으로 발전한다.
<함수>
정적인 계산이 아닌 움직임을 계산하는 수학의 탄생하여, 전쟁에 지대한 영향을 미쳤다.
<투영도>
유클리드 기하학이 아닌 공간을 보여주는 정밀하면서, 효과적인 방법들이 개발되었다.
<확률>
대항해 시대에 발전한 새로운 수학으로 성공만한다면 일확천금을 얻을 수 있는 항해의 성공패턴을 연구하면서 발생하였으며, 이후 프랑스를 거쳐 러시아에서 완성되었다.
<통계>
영국과 독일에서 비슷한 시기에 발전하였는데, 영국에서는 사망표를 기반으로 한 사회통계학, 독일에서는 국세통계학이 시작이었다. 단순한 숫자의 표에서 시간의 흐름에 따른 변화를 찾고 그 형태나 실태를 파악하는 것이다.
<보험>
오래 전부터 장례비용을 나누어 내는 계 같은 것이 재해를 예방하는 방법이었는데, 확률론과 통계학의 발달 으로, 수학적인 계산을 통한 보험은 런던 대화재 이후의 화재보험이 최초이다.
<변환법>
프랑스에서 발달된 학문으로 공간을 보여주는 정밀하면서, 효과적인 방법들이 개발되었다. 이러한 기하학은 구면기하학, 화법기하학, 사영기하학이 있다. 눈에 보이는 대로가 아니라, 점들의 연결 상태에 집중한 학문으로 후에 위상수학으로 발전한다.
<최적화 이론>
2차 세계대전 중 독일의 과학 팀의 연구로 최소한의 노력으로 최대한의 이익을 취하는 학문이 발전하게 된다. 이는 후에 경영 과학으로 발전한다.
<동아시아 수학사>
2세에서 16세기 까지는 동아시아의 수학은 유럽보다 월등히 앞서 있었다. 피타고라스의 정리, 선형연립방정식, 파스칼의 삼각형, 방정식의 근사 해를 구하는 방법 등 유럽보다 더 빠르게 발전했다.
조선시대에 중국에서 쇠퇴기로 접어들게 되고, 조선의 고유의 산학이 발달했다. 임진왜란당시 우리 고유의 산학 책을 모두 약탈당하여, 일본 수학의 시작을 이끌었다.
조선과 중국은 서양의 수학에 대해 크게 관심을 갖지 않았고, 유럽의 수학은 대항해 시대이후 크게 발전하여 동아시아의 수학을 추월하여 역으로 영향을 미치게 된다.
수학이 어떻게 발전해왔고, 왜 중요하게 여겨지게 되었는지, 옛날에는 어떤 수학이 어떻게 받아들여졌고 그 근거와 기록이 어디에 있는지, 서양 수학이 어떤 경로로 발달해왔고, 동아시아 수학은 왜 학계에 잘 받아들여지지 않는지, 우리가 왜 수학을 배워야 하는지, 현대에는 수학이 어떤 분야에서 어떻게 사용되고 있는지를 발표까지는 버겁더라도 사담 정도는 나눌 수 있을 정도로 대략적으로 말할 수 있게 되었습니다.
- 서양 수학과 동양 수학을 어떤 식으로 다르게 받아들일 것인가. (Finalized)
A : 동양 수학에만 있었던 개념이나 정체성을 찾는 방향이 아니라, 같은 개념이 다른 경로로 어떻게 탄생하고 연구되는 방식에 어떤 차이가 있고 어떻게 사용되었느냐에 중점을 맞추는 편이 낫다.
- 정수론이 실용적으로 사용되는 예 (Finalized)
A : 정수론에서 나온 관념이 철학적인 문제로 발전한 경우도 있고, 현대 정보학이나 알고리즘, 쌍곡선 연구 등에 정수론이 사용되며 ݝ¿(phi)나 황금률을 연구하는 데에도 유용하다.
- 질문은 아니고 짤막하게 달았던 답에 대한 답글이지만, 수학자 장이탕(이탕 장)의 생애와 업적. (이탕 장에 대해 잘 몰랐던 부분들을 크게 알게 되었습니다)
- 질문은 아니지만, 보험과 수학의 관계에 대한 지식 전반. 막연히 관계가 있겠지 하고 생각만 해오던 부분이 생생하게 다가온 점이 좋았습니다. 마침 응용수학에서 Black-Scholes Equation을 배우던 참이라 금융수학의 이론과 그 역사와 결부시킬 수 있었던 점도 흥미로웠습니다.
▪ 나의 질문 및 제공
1. Q 63 : 일본은 중국과 교류할 때 수학적, 학문적인 것들을 교류하지 않았는가 (19.03.17)
● A : 당시 일본은 명과 조선처럼 동맹국이 아니었다. 따라서 정식으로 산학책을 구할 수는 없었고 일본이 중국 해안가에서 노략질로 확보할 수 있는 고급정보와 책은 거의 없었다.
일본이 임진왜란 시 조선의 왕궁, 경복궁을 점령하고, 보물과 함께 귀한 문서와 책들을 약탈해 가면서 비로서 그 중에 고급 산서를 구했고, 약탈해간 조선산학의 내용을 이해하는 산학자를 배출하게 된 것은 그 후 수십 년이 지나서이다. (이상구 교수님)
● 링크 내용
- 수학의 군사적인 목적 : 수학은 수세기 동안 군사적인 목적에 의해 자극을 받고 재정 지원을 받고 인정받아 왔다. 어떤 수학적 사고와 수학적 기술 또한 전쟁에서 필수적이었다.
- 중국의 산학책이 일본으로 전해짐 : 에도 시대에 일본에 소개된 13세기 말에 중국에서 출판된 수안큐 키멩(상아쿠 게이모)이라는 수학적 책이 잘 알려지게 되었고, 이 책에 소개된 수학적 기법 중 하나인 톈위안슈(天安 yuan)가 주목을 받게 되었다. 이 텡겐주트수(톈안위안슈)는 현대의 수학 용어로 계산봉을 사용하여 2차 방정식이나 입방정식을 설정하는 기법이다. 만약 그것을 사용할 수 있었다면, 그것은 그 시대의 어려운 수학 문제들에 실질적으로 적용될 수 있었을 것이다. (이상구 교수님)
● 정리 : 당시 일본이 무기만 도입했지 아직 선진문물과 학문을 접하지 도입한 것은 아니었다. 수학과 전쟁, 군사적인 목적 은 밀접한 관련성을 가지고 있고 임진왜란 후 중국에서 들여온 산학책은 일본 수학의 발전을 가져오게된다. 결국 일본은 서양과 동양 수학 모두를 접하게 되어 큰 발전을 하게된다.
2. Q 140 : 수업시간에 반원과 직선의 점의 갯수 설명 들을 때 일대일대응 설명하시며 말씀해주셨는데, 아무리 작은 원이라도 아주아주 긴 어떤 직선보다도 항상 점의 갯수가 더 많다 이것도 맞는 말이 되는건가요? (19.03.27)
● A : 무한론을 가장 일반적인 언어로 풀어 쓴 예로는 힐베르트의 '무한 호텔'이 있습니다.
그냥 말장난 같기만 할 수도 있으니 머릿속으로 받아들여지느냐의 차이는 있지만... 슈뢰딩거의 고양이나 디렉 델타 함수도 발표 당시에는 학계의 수많은 사람들에겐 말장난으로 받아들여졌으니 결국 받아들이기 나름입니다.
이를 응용하면 자연수의 개수는 2의 배수의 개수와 같습니다. 1대 1 대응하니까요.
유리수의 개수와도 같습니다. 분모를 p로 고정시키면 분자 q와 1대 1 대응하니까요.
무리수의 개수는 더 많습니다. 칸토어의 대각화 논증으로 증명되는데, 그 대각화 논증에서 보이는 논리는 결국 무리수는 유리수와 1대1 대응이 불가능하다는 증명입니다. 물론 1대 1로 대응시킬 수 없어도 같다고 보는 경우도 있습니다. 반원의 점의 개수는 직선의 점의 개수보다 2개 더 많아 완벽한 1대1 대응이 불가능하지만 같은 무한으로 칩니다.
무한보다 작은 실수 범위의 차이이므로 같게 보는 겁니다. 이제 직선의 길이가 달라도 점의 개수가 같나는 물음에 답하자면, 1대 1 대응하므로 같습니다. (강민석)
● 원 질문 Final : Aleph alpha + n = Aleph alpha (for every natural number n, for every alpha) 는 성립한다 책에서는 무한의 레벨에서는 별 차이가 없다는 것을 강조하려고 했을 뿐이다. 실제로는 반원쪽이 일대일 대응보다 점이 많긴 하지만 무한의 레벨에서는 차이가 없다고 봐도 무방하다 (최민수)
● 정리 : 무한은 아주 큰 수라 생각했고 다른 일반적 개념들과 일맥상통 할 것이라 생각했다. 하지만 무한은 접근 자체가 달라야 이해할 수 있는 개념이었다. 이러한 무한에 대한 추상적인 점을 칸토어가 객관화하려 노력했고 큰 공헌을 하였다.
3. Q 177 : 정수론이 암호학에 많이 쓰인다 하셨는데. 양자컴퓨터가 나오고 나서는 현재 보안의 페러다임이 완전히 바뀌어야한다고 읽었습니다. 그렇다면 양자 컴퓨터가 나오고 난 후에는 어떤 방법으로 암호를 만들 수 있는건가요? 엄청난 계산속도를 가지고도 못푸는 암호가 존재하나요? (19.03.27)
● A : 현재 수학의 발전 상태로는 거대한 어떤 수를 주고 그 수가 합성수라고 했을 때 그 수가 도대체 몇 곱하기 몇인가, 그걸 특정지어 단박에 알아낼 수 없는 방법은 없다시피 합니다.
3의 배수인가? 아니고... 그럼 7의 배수인가? 그것도 아니고... 식으로 깜깜이식으로 찾아갈 수밖에 없습니다. 물론 곱해서 거대한 수가 될 정도로 충분히 큰 두 수를 주고 곱해봐라 그런다면, 연산 속도에 따라 단박에 됩니다. 즉 거대한 수를 주고 이건 소수가 아닌데(소수인지 아닌지 가려내는 것도 사실 큰일입니다) 그럼 몇 곱하기 몇인가 하고 묻는다면, 이에 대한 답은 굉장히 구하기 어려운 반면, 일단 답을 구했다면 그게 맞는지 틀린지는 쉽게 검산 가능합니다. 그냥 곱해보면 되니까요. 이는 P대 NP 문제로 이어지지만, 자잘한 거니까 생략합니다. 좋은 암호의 조건은 외부인은 굉장히 풀기 어렵지만(풀기 불가능할수록 좋겠죠) , 암호의 규칙을 알기만 하면 굉장히 풀기 쉬워야 합니다. 좋은 컴퓨터가 있다는 가정 하에 이 소수/합성수 문제는 굉장히 좋은 조건을 충족합니다. 그래서 실제로도 쓰입니다.
여기서 대답인데, 인류의 수학 발전 상태가 현 상태에서 답보가 없다면 암만 컴퓨터 성능이 좋아도 그 성능값이란 게 유한한 이상은 이런 체계를 이용한 암호는 풀기 어려울 것입니다. 정확히 말하면, 풀기 불가능한 암호란 건 존재하지 않는다고 쳐버리고, 그럼 풀리긴 풀리는데 풀 때까지 평생의 한 100배쯤 걸리는 암호를 만들어버리면 된다는 식입니다. 이런 접근방식으로 다가가면 컴퓨터의 성능은 더 이상 암호를 풀 수 있는가의 여부에 대한 변수가 되지 못합니다. (강민석)
● A : 동형암호
동형암호는 현존 최강 암호체계 중 하나로 꼽히는 ‘격자기반암호’의 한 종류다. 격자기반암호란 수학계에서 절대 답을 찾을 수 없다고 알려진 ‘격자 문제(lattice problem)’를 응용한 암호를 말한다. 동형암호를 포함한 격자기반암호는 양자컴퓨터가 도입돼도 깨지지 않는 차세대 암호체계라는 것이 암호학계의 중론이다. 현재 전자상거래 등에 널리 쓰이는 RSA 같은 기존 암호체계는 양자컴퓨터가 도입되면 붕괴될 수밖에 없다. 양자컴퓨터는 큰 수의 소인수분해와 데이터베이스 검색을 매우 효과적으로 수행할 수 있다고 알려져 있는데, RSA의 토대가 소인수분해이기 때문이다. 반면, 격자 문제는 아직까지 해법을 찾지 못했다 (조영은)
● 정리
현재 주로쓰이는 암호체계들은 소인수 분해를 기반으로 한다. 암호를 만들어내는 것 보다 풀어 내는 것이 비교도 안될만큼 어려워 암호로 쓰이고 있는 것 이다. 양자 컴퓨터는 기존 컴퓨터와 작동원리가 달라 계산속도가 비약적으로 빨라지는것인데 이론적으로 컴퓨터의 계산 속도가 아무리 빨라도 결국 그 속도가 유한하다면 즉 연산속도가 유한하다면 위와 같은 원리로 그에 대응하는 암호를 만들어낼 수 있다.
기존의 체계를 그대로 사용한다면 양자컴퓨터로 풀기힘든 암호를 만들기위해선 큰 숫자 즉 어마어마한 데이터량을 처리해야 할 것이다.
이런 양자컴퓨터 문제에 대응하기 좋은 암호 중 하나로 동형암호가 있다. 아직 답을 찾을 수 없는 문제인 격자문제를 이용한 암호시스템으로 양자컴퓨터로도 풀 수 없는 암호이다. 관련 기사들이나 정보들을 보면 현재 동형암호가 양자컴퓨터를 대비한 가장 좋은 암호인 것 같다. 미래에 격자문제를 푸는 방법이나 알고리즘이 나왔을 때 또 어떤 새로운 암호체계가 나올까 궁금해진다
4. Q 188 : 고대 유명한 수학자들을 보면, 대부분 귀족집안의 사람들 인 것 같다는 생각이 듭니다. 이들은 애초에 생존에 문제가 없었기 때문에 이런저런 생각들을 많이 하게되고 더 많은 새로운 것을 추구하고 진리를 탐구하는데 관심을 쏟은 것 같다는 생각이 들었습니다. 그렇다면 예전 수학자나 학자들 중에 가난했지만 주위에 관심을 가지고 위대한 업적을 남긴 수학자들도 많이 있나요? (19.03.27)
● A : 동페렐만 과 이탕 장 이라는 예들이 있다.
쌍둥이 소수 이론의 증명 가능성에 획기적인 한 획을 그은 수학자 이탕 장은 시간제 강사로 벌어먹고 살던 사람이었습니다. 시간제 강사 자리도 서브웨이(그 샌드위치 말하는 거 맞습니다) 아르바이트 하면서 여기저기 이력서 넣고 아무 지원 못 받고 개인논문 쓰다가 겨우 잡은 자리였습니다. 쌍둥이 소수 이론의 증명 가능성을 높인 업적으로 교수직을 얻고 테렌스 타오 등의 수학자와 수학 커뮤니티에서 협업하고 있다고 압니다.
그레고리 페렐만은 푸앵카레 추측의 증명으로 유명해졌는데, 이 사람은 수학계의 협잡질(중국계 수학자 야우싱퉁이 아주 지독하게 굴었습니다)에 신물이 나서 필즈상이고 상금이고 다 거절하고 때려치운 채 평생의 은사와도 연락을 끊고 잠적해버린 것으로 유명합니다. 필즈상도 상금도 거절하고서도 정부에서 주는 저소득층 지원금 타먹으면서 홀어머니와 살았습니다. 상술했듯이 잠적해버려서 현재 근황은 모르겠습니다. (강민석)
● A : 페렐만 과 이탕 장 에 대하여 좀 더 알아보는 것도 재미있을 듯 합니다.
언론은 좀 과장하여 극적으로 ... 표현 하기도 하지만 ... 현실은 좀 더 현실 적일 것입니다. ... 그런데 둘 다 원하기만 했다면 ... 적당한 대학에서라면 ... 대학에서 강의 하면서 ... 꾸준히 논문을 쓰면서 ... 성실히 강의했다면 ... 승진도 하고 ... 월급도 올라가면서 ... 너무 어렵게 생계를 꾸리지는 않았을 것입니다.
보통 석사, 강사, 박사 다음 전임강사, 조교수 , 그리고 다음 단계로 하고싶은 연구를 롱텀으로 할 수 있도록 계약기간을 충분히 길게 보장해 주는 안정된 Tenured Job 을 잡기 전에 ... 미리 다른 서비스는 포기하고 ... 집에서 나오지 않고 ... Big problem 에만 집중하는 경우에 대한 ... 상황은 나라마다 좀 다른데 ... ....
그리고 그런 사람들도 많기 때문에 ... 사회주의 국가에서는 최소 임금을 주면서 연구소에 머물면서 연구 하도록 하여 페렐만 은 수학연구소에서 적지만 월급받고 게속 연구를 할 수 있었는데 중국도 비슷 .. Publish or Perish 를 주장해온 미국대학 그리고 ... 미국은 자본주의 사회라 ... 좀 달랐을 것입니다.
(... Yitang Zhang 도 성과를 보이니까 바로 UH 에서 강사에서 교수로 승진시켜주고, 이어서 UC 산타바바라에서 교수로 모셔갔듯이 ... )
그래서 미국 프린스턴 IAS 에서 더 나은 여건에서 ... 아인스타인, 폰 노니만, 괴델 ... 등, 그런 수학자들을 모아서 ... 다른 나라가 못 이룬 큰 성과를 이룬 것이랍니다.
천재 수학자를 알아보고 선발하는 구체적인 방법만 있다면 ... 지금은 ... 어느나라라도 ... 그런 인재들을 모아서 ... 월급을 주고 ... 하고 싶은 이론 연구를 하도록 해 주고 싶을 것입니다.
우리나라도 IBS 가 지금 그런 역할을 하려고 만들어 졌답니다. (이상구 교수님)
● A : 동직업적인 수학자 (고대의 많은 수학자 들) 와 다른 수입이나 직업이 있으면서 수학을 연구한 수학자 (페르마 등) 그리고 직업적인 수학자도 수입이 많은 연구 여건이 좋은 직장 (프린스턴 고등연구소 등) 이 있는 직업수학자와, 수입이 적은 그러나 연구 여건은 나쁘지 않은 소련의 수학연구소들 , 또는 말단 공부원 등 그리고 제대로 된 직장은 못 잡아보고 수학을 연구하다 죽은 분들 (아벨, 갈로아 등) (이상구 교수님)
● A : Niels Henrik Abel 닐스 헨리크 아벨, 노르웨이를 대표하는 수학자.
천재적인 수학자였으나 인정받지 못한 채 20대 한창 나이로 죽은 비극적인 인물로, 비슷하게 젊은 나이에 세상을 뜬 천재 수학자 갈루아와 비교되기도 한다. 아벨의 시대에 가장 오래된 난제 중 하나는 1차부터 4차 방정식 처럼 5차 방정식에도 계수를 이용해 근의 공식을 만들 수 있느냐였다. 아벨 역시 처음엔 대수적으로 풀 수 있다고 굳게 믿고 증명을 시도하였으나 후에 생각을 바꾸고 풀 수 없음을 증명하였다.
하지만 당시로선 너무 난해하고 추상적이던 아벨의 증명을 누구도 인정하지 않아 자비로 논문을 출판해야 했으며 그나마도 묻혔다. 이 증명은 뒷날 프랑스의 수학자 갈루아에게 영향을 미쳐서, 군을 이용한 고차 방정식의 비가해성으로 확장된다.
당대 수학의 신이라고 불리우던 카를 프리드리히 가우스에게 책을 보냈으나 하도 많이 오는 여러 아마추어 수학자들의 책과 편지에 질려버린 가우스가 무시했다. 코시에게도 논문을 보냈었는데 코시가 논문을 잃어버려서 역시 읽히지 못하였다.
19세에 아버지를 잃고 가난에 허덕이던 그를 그나마 인정하고 알리려던 사람이 독일 수학자 아우구스트 레오폴트 크렐레(August Leopold Crelle ,1780~1855)였는데 그의 노력으로 아벨에게 베를린 대학 수학 교수직을 얻게 해주었다. 그러나 아벨은 가난에 시달리고 있었고 이로 인해 결핵을 얻은 상태라 결국 소꿉친구이자 약혼자였던 크리스티네 켐프(Christine Kemp)의 품에서 26세의 젊은 나이로 요절하고 만다. 그리고 아벨이 죽고 난 바로 이틀 뒤에 교수 임명서가 도착했다. 반응이 없다는 대학 측 답변에 이상하게 여긴 크렐레는 편지를 보냈는데 '닐스는 이미 죽었답니다' 라고 아벨의 지인이 쓴 답장을 받고서야 뒤늦게 죽은 걸 알게 되었다. 나중에 크렐레는 그의 무덤에 와서 이 천재를 너무나도 빨리 데려갔다면서 슬퍼했다.
이 사람의 이름을 붙인 이론이 상당히 많다. 대표적으로 군론에서 어떤 연산 *에 대해 군에 속하는 임의의 두 원소 a, b에 대해 a*b=b*a처럼 교환 법칙이 성립하는 군(group)을 아벨 군(abelian group)이라 한다. 그 외에도 위상수학, 실해석학 등에도 이름이 붙어 있는 이론이 있고 적분에도 아벨 적분이라는 게 있다. (김기헌)
● 정리 : 좋은 환경에서 연구하여 훌륭한 성과를 낸 수학자들도 많았지만 아벨과 같이 가난한 환경에서 학구열을 가진 훌륭한 분들도 많았다. 또 어려운 환경을 탈피할 수 있었음에도 그런 선택을 하지 않은 사람도 있었다. 더 궁금해지는 점은 이들의 비율이다. 수학사에 한 획을 그은 인물 중 힘든 환경에서 업적을 이뤄낸 사람들의 분포비율은 어떻게 될것인가
Q 237 : 1. f 는 전단사 함수이다.
-f 는 전단사 함수이다
-f는 연속 함수이다.
-역함수 f^-1 도 연속 함수이다
만일 이러한 세 가지 조건을 만족시키는 함수가 두 위상 공간 사이에 존재하면, 두 위상 공간이 서로 위상 동형(영어: homeomorphic)이라고 한다. 라 하셨는데 1번은 서로 가역적으로 바꿀 수 있다는 뜻인가요? 그리고 2,3 번에서 연속이라는건 어떤 의미인가요 물리적으로 이어져있다는 뜻인가요?? (19.04.10)
● 원 질문 : 위상동형이 구부리거나 잡아당기거나 해서 같은 모양이면 서로 위상동형이 된다고 했는데, 그것의 수학적 엄밀한 정의나 함수관계로 어떻게 나타낼 수 있는지가 궁금합니다.
(최민수)
에 대한 답변 중 위상동형의 조건 설명이 나왔고 그것을 이해하지 못해 질문드림
● A : 전단사 함수는 전사이자 단사인 함수입니다.
전사함수는 그냥 1-1 대응 함수라고 보면 됩니다.f(x1) = f(x2)와 x1 = x2가 서로 동치인 함수. 단사함수는 치역과 공역이 같은 함수. 즉 공역(Ran(f)로 표기하곤 합니다)에 있는 어떤 원소 y를 짚어도 y = f(x)인 x가 틀림없이 정의역 내에 존재하는 함수. 직관적으로 당연히 전단사 함수는 역함수를 갖습니다. 연속에 대해서는 고등학교 수학에서 주로 다루는 그 개념 맞습니다. 다만 위상수학에서 함수를 다루고 위상동형임을 증명할 때는 해석학에서 다루는 연속함수의 정의를 씁니다.
https://ko.wikipedia.org/wiki/%EC%97%B0%EC%86%8D_%ED%95%A8%EC%88%98
여기의 정의 부분 맨 위부터 네 줄 참고하시면 되고, 간단히 말해버리자면 lim(x→a) f(x) = f(a)이면 a에서 연속하는 겁니다. (강민석)
● A :
1번 전단사도 이미 100% 정확히 정의된 수학용어이고
2 번 연속 함수도 이미 100% 정확히 정의된 수학용어이고
3번도 이미 100% 정확히 정의된 수학용어
이니까 100% 엄밀한 정의로 정의대로 그대로 이해하시고 사용하시면 됩니다. (이상구 교수님)
● 정리 : 위상동형이란 어떤것인가 알 수 있었다. 전사 단사 전단사의 용어 뜻을 알고 위 상황에 어떻게 적용시켜야 할지 알 수 있었다.
5. Q 279 : 페르마의 마지막 정리와 푸앵카레의 추측을 보며 의문점이 들어 질문드립니다. 다들 어떻게 생각하시는지
페르마의 마지막 정리는 왜 정리 라고 이름이 붙었고 푸앵카레의 추측은 왜 추측이라 이름붙은걸까요?
엄밀히 말하면 페르마도 증명 없이 (본인은 증명했다 주장했다하지만) 내용만 말한것이고 푸앵카레도 증명없이 내용을 말한것인데 페르마는 정리라고 이름붙이고 푸앵카레는 추측이라고 할까요?
(19.04.10)
● A : 증명이 되었을 경우 정리라고 하고 꽤나 설득력 있고 의미있는 명제인 경우에 추측이라고 한다. 페르마의 마지막 정리의 경우에는 최고의 수학자 중 하나인 페르마가 증명을 했다고 주장하고 다른 많은 사람들도 풀었다고 주장을 했었던 바가 있었으므로 추측이였던 시절이 길었음에도 정리라고 명명했었다. 푸앙카레 추측의 경우에는 추측이였던 기간이 또한 길어서 요즘에도 추측이라고 부르는 사람들이 많지만 이제는 정리이다. (최민수)
● A : 맞습니다. 증명했다고 몇번을 했던 문제라 몇 번을 페르마의 마지막 정리로 불려졌다가 ....다시 미해결 문제로 있다가 등의 우여 곡절을 거친 후 지금은 페르마-와일스 정리라고 부르는 것이 합리적인데 페르마를 기념하여 대부분의 사람들은 그냥 페르마의 마지막 정리로 부릅니다. (이상구 교수님)
● 정리 : 페르마의 정리는 엄밀한 의미에서 증빙자료가 없어서 정리라고 불렀으면 안됬지만 페르마의 주장과 명예로 이런 예외적인 상황이 나타나게 되었다. 말 한마디한마디 사실 하나하나가 중요한 수학에서 이런 예외적인 상황이 나오게 된 것이 놀랍다.
6. Q 296 : 컴퓨터로 증명을 하면 그 증명이 맞는지 검증을 하려면 또 컴퓨터밖에 검증할 수 없다 는 것인데 종종 프로그램들을 사용하다보면 오류나 버그가 있을 수 있는데 수학계에서는 이런 컴퓨터 증명을 문제를 증명한 것으로 받아들였나요?
(19.04.10)
● A : 물론입니다. 그래서 기존의 증명에 새로운 방식의 이런 증명을 포함하는 개념으로 Mathematical Verification 이라는 용어가 나왔습니다. (이상구 교수님)
● A : 논리적 결함이 없고 케이스바이케이스로 나눠서 푼것이므로 받아들여지긴 했습니다
컴퓨터를 사용했기도 했고 더욱 우아하고 간단한 증명을 찾는 사람들도 아직 있다고 합니다 (최민수)
● 정리 : 컴퓨터를 이용한 증명은 그 증명과정이 맞는지 인간이 확인할 수 없다는 논란이 있지만 Mathematical Verification 이라는 용어가 새로 나올 정도로 이미 받아들여지고 있다. 수학이 실생활이나 기술에 영향을 주는 것이 일반적인데 이 경우는 반대로 기술의 발전이 수학의 발전을 이끈 경우이다. 하루가 지나면 또 다른 기술이 나오는 현대에서 과연 앞으로 기술의 발전에 힘입어 어떤 수학이 나올것인가 기대가 된다.
8. Q 230 : 한국 근대수학 100주년 기념전시 후기
(19.04.10)
● 내용 :
한자로 수학을 가르치고 있는것이 인상적이었고, 신기한것은 수학책임에도 숫자가 별로 없다는 것이었습니다. 아라비아 숫자가 없을 것은 예상 했지만 한일 두이 석삼 같은 한자도 별로 보이지 않고 설명이 주로 되어있었습니다. 책을 펼쳐볼 수 없으니 책 뒤에 연습문제처럼 따로 숫자가 가득한 페이지가 있나 궁금해서 소장하고계신 분께 질문을 드렸는데 한자가 가득하다고 하셨습니다. 어떻게 글자만으로 수학을 가르친건지..
또 제가 흥미롭게 봤던건 (사진은 못찍었지만) 일제강점기때의 수학책이었습니다
당연히 일본어가 가득한 책이었습니다. 제가 흥미를 가졌던 이유는 언어가 아닌 내용이 지금의 수학책과 너무 똑같았기 때문입니다. 고등학교 수학책을 펼쳐보았는데 지금 고등학생이 배우는 내용과 같은 내용들과 구성이었습니다. 설명과 예제 그리고 뒤에 연습문제를 모아놓은 것 까지 거의 100년 전 책들이었는데 지금의 책들과 너무나 똑같아서 깜짝 놀랐습니다. 심지어 난의도도 거의 같았습니다. 소장하시는 분께 질문을 드렸는데 난의도는 오히려 지금보다 일제시대가 더 높았다 하셨습니다
● Q : 한자로 수학을 가르치고 있는것이 인상적이었고, 신기한것은 수학책임에도 숫자가 별로 없다는 것이었습니다. 아라비아 숫자가 없을 것은 예상 했지만 한일 두이 석삼 같은 한자도 별로 보이지 않고 설명이 주로 되어있었습니다. 책을 펼쳐볼 수 없으니 책 뒤에 연습문제처럼 따로 숫자가 가득한 페이지가 있나 궁금해서 소장하고계신 분께 질문을 드렸는데 한자가 가득하다고 하셨습니다. 어떻게 글자만으로 수학을 가르친건지..교수님 도와주세요 (이헌수)
A : 이전에는 한자도 그렇지만 종이도 귀해서 종이에 쓰면서 산학과 계산을 한 것이 아니라 토론하고 산대를 이용하여 게산 하여 답을 구햇답니다. 그래서 백농 최규동 선생이 신학문을 배우려고 경성으로 가다가 수원 향교에서 아라비아 숫자로 수학을 가르키는 것을 보고, 이것이 미래라고 생각하고 수학을 하기로 마음 먹고 당시 수학을 제일 잘 가르친다는 유일선 선생의 정리사에 가서 수학을 배우고 나중에 중동학교 교사가 되고, 교장이 되고, 경성대 이사장과 국립 서울대 총장이 되셨답니다. (이상구 교수님)
● Q : 저렇게 전시된 오래된 책들을 현재 어떤 기관에서 보관하고 있는건가요? 그리고 저 책들이 어떠한 가치를 가지고 있는지도 궁금합니다. (조영은)
A : 지난 주말에 전시 한 것들은 나와 김영구 소장자가 가지고 계신 것 들입니다 (이상구 교수님)
A : 먼저 소장은 어떤 기관이 하고있는것이 아니라 개인의 소장품이었습니다. 그날 소장하고계신 분께서도 오셨는데 집에 저런책들이 몇천권 있다고 구경하러 오라고 명함도 주셨습니다 우리나라 문화재들 중에서도 개인소장품들이 많으니 그런것이 아닐까 생각합니다
가치에 대해선 소장하신 분께서 하신 말씀이
우리나라의 유물이나 주요 자료들은 일제시대 사라진 것이 많다 하시고 오래된 책들은 희소성도 크고 조선시대에 수학을 어떻게 가르쳤는지 보여주는 자료라 하시네요. 물론 일제 강점기 포함 그 이후 책은 물량이 많아서 가격도 저렴하고 구하기도 쉽다 하셨습니다. 실제로 구경하러 오신분이 자신도 얼마전에 똑같은 책 어디서 샀다 하면서 대화하셨어요 (이헌수)
9. 교수님 이야기 중 기차에 치여죽은 유학생 (19.05.07)
● 내용 :
교수님께서 이야기 해주신 학사학위를 받고 안타깝게 기차에 치어죽으신 유학생 이야기입니다. 우리나라 농업 발전을 한참 앞당길 수 있었을텐데 안타깝네요..
최초의 이학사 변수(1861-1891)
변수는 1882년 3월부터 개화파인 김옥균, 서광범, 강위 등과 일본에서 화학과 양잠술을 공부했고 제물포 조약 후 박영효 정사를 수행해 김옥균, 민영익, 서광범 등과 도일 차관 교섭 요원으로 일했다.
1883년 보빙사의 수행원으로 채용돼 민영익 전권대신과 세계 일주를 하고 귀국한 변수는 이후 갑신정변에 참가했다 김옥균, 박영효, 서광범, 서재필 등과 일본에서 망명 생활을 했다. 그는 민영익 대신이 홍콩은행에 맡겨놓은 거액의 돈을 훔쳐 일본에서 망명처를 찾고 있던 중국 유학생 민주호, 윤정식을 만난후 신문학을 배우고 싶어 이들과 함께 도피처로 1886년 1월 미국으로 온다.
그는 워싱턴시에 있는 벨리츠 어학학원에 입학, 영어 공부를 하고 1년반 후 1887년 9월 메릴랜드 농과대학(현 메릴랜드 주립대의 전신)에 입학한다. 4년 뒤 1891년 6월 변수는 미국인 학생 4명과 함께 졸업함으로써 한국인 최초의 미국 대학 B.S. 학위를 받은 사람이 됐다.
농학 학사 학위를 취득한 그는 귀국할 수 없는 처지였기 때문에 졸업 후 미국 농무성에서 촉탁(consultant) 자리를 얻어 일본의 농업에 관한 통계 보고서를 작성했다. 그의 졸업논문 및 연구보고서는 당시 미국농무성 차관이었던 윌리츠에게서 극찬을 받기도 했다고 전해진다. 이어 중국의 농업에 관한 연구 계약을 맺고 준비하던 중 1891년 10월22일 College Park 기차역에서 기차에 치어 30세의 나이로 사망했다. 워싱턴 포스트는 1891년 10월23일자 신문에 그의 사망기사를 실었다. (이헌수)
10. Q : 동아시아의 수학이 근대 이전까지 서양보다 앞서있었다고 하는데 연립방정식의 해법, 고차방정식의 해법 등 여러 가지가 서양보다 한참 앞섰다면 이것이 수학사에 언급되지 않은것은 서양에서 인정하지 않으려해서일까요..??
(19.05.08)
● A : 동양의 수학은 고대로부터 중세에 이르기까지는 서양수학에 밀리지 않을만큼 훌륭한 수준이었고 "동양은 대수, 서양은 기하"라는 말이 있을정도로 더 뛰어난 부분도 많았습니다.
하지만 근대에 들어서 부터 서양수학은 눈부신 발전을 이뤄왔고 지금 우리가 사용하고 있는 수학은 대부분 서양수학이라 봐도 무방할 정도로 현대에서 서양수학이 차지하는 비중이 대부분입니다. 그래서 현재 관점에서 세계수학사의 중심이 서양수학사 임은 무시 할 수 없습니다. 그 이유는 동아시아 수학은 서양 수학과 학문적 교류가 크지않아 서양수학의 발전에 큰 영향을 주지 못한 것이 이유라고 합니다. 반면 아랍은 서양과 학문적 교류도 활발하였고 특히 유클리드의 원론을 보존하고 발전시킨 아랍은 서양수학의 부흥에 큰 영향을 주었기 때문에,
서양 사람들은 수학사에서 아랍의 수학의 그 기여를 인정 하고 비교적 많은 분량을 배당합니다. 그러나 동아시아의 수학은 사양수학의 발전에 크게 영향을 주지 않았다고 생각하기 때문에, 비중을 적게 준다고 교수님이 수업 시간에 말씀해 주셨습니다. . (이헌수)
11. 조선시대 정보기관 제국익문사(帝國益聞社) 과 암호 화학비사법(化學秘寫法) (19.05.28)
● 내용 : 조선시대에도 미국의 CIA나 소련의 KGB 와 같은 정보기관이 있었다. 그 이름은 제국익문사(帝國益聞社)이고 1902년 6월 고종이 황제 직속으로 설립한 비밀정보기관이다. 이 기관은 정부고관과 서울 주재 외국 공관원의 동정, 국사범과 외국인의 간첩행위를 탐지하는 것이 주요 임무였다. 제국익문사는 근대적 형태의 정보기관이며, 표면적으로는 '매일 사보를 발간해 국민들이 보도록 하고 국가에 긴요한 서적도 인쇄'하는 현대판 통신사 기능을 담당했다. 요원은 총 61명으로, 요원들이 고종에게 정보를 보고할 때 화학비사법(化學秘寫法)이라는 특수한 방법으로 보고를 하였다. (이헌수)
● 추가 정보 - 화학비사법
화학비사법은 과일즙이나 화학용액을 이용해 글씨를 투명하게 쓴 다음 읽을 때는 열이나 또다른 화학용액을 사용하는 방식입이다. 이렇게 작성한 문서를 황제에게 보고할 때 또한 제국익문사를 통한 것인지 확인할 수 있도록 자두꽃 무늬와 성충보좌라는 글씨가 새겨진 제국익문사 고유의 인장을 사용했다. (조영은)
● Q : 조선시대에는 화학비사법 이외에 눈에는 보이지만 해독을 해야하는 암호는 없었나? (김윤수)
● A : 독립군 들이 한글을 바탕으로 암호화 한것이 기록으로 남아있다. 발견된 암호부에는 암호의 원리가 상세히 담겨 있다. 한글의 자음과 모음은 아라비아 숫자로, 아라비아 숫자와 동서남북은 특수기호로 나타낸 걸 확인할 수 있다. 예를 들어 자음 ‘ㄱ, ㄴ, ㄷ…’과 모음 ‘ㅏ, ㅑ, ㅓ…’는 아라비아 숫자 ‘1, 2, 3…’으로 썼다. 이 암호는 3.1운동에도 사용됐을 것으로 추정된다. (이헌수)
▪ Final한 게시물
1. Q.아마존에서 뽑는 인재상이 궁금하다. (19.04.14)
● A : 위의 기사에서는 아마존이 원하는 인재상으로 절약,효율,타인과의 소통 정도로 나오지만 모든 회사들이 인재상이나 사훈이 있듯, 아마존에는 "리더쉽 원칙"이라는것이 있습니다.
Customer Obsession, 고객에 집착하라.
Ownership, 결과에 주인의식을 가져라.
Invent and Simplify, 발명하고 단순화하라.
Are Right, A Lot, 리더는 대부분 옳다.
Learn and Be Curious, 배우고 호기심을 가져라.
Hire and Develop the Best, 최고 인재만을 채용하고 육성하라.
Insist on the Highest Standards, 최고의 기준만을 고집하라.
Think Big, 크게 생각하라.
Bias for Action, 신속하게 판단하고 행동하라.
Frugality, 근검절약을 실천하라.
Earn Trust, 다른 사람의 신뢰를 얻어라.
Dive Deep, 깊게 파고들어라.
Have Backbone; Disagree and Commit, 기개를 가져라 – 반대하되 받아들여라.
Deliver Results, 구체적인 성과를 내라.
이 14가지가 아마존의 리더쉽 원칙입니다.
보통 회사들이 인재상과 사훈에 비중을 많이 두지 않지만 아마존에서는 직원채용과 프로젝트 착수 등 모든 의사결정이 필요한 곳에 이 원칙들이 기준이 된다고 합니다. 제 생각에는 이 리더쉽 원칙들은 분명 많은 장점들이 있지만 마냥 좋다고만은 할 수 없을 것 같습니다. 지나친 성과주위도 문제가 될 수 있고, 호율이나 근검절약 같은 경우도 복지가 부족하다는 문제로 이어집니다. 실제로 아마존 평균 근속기간은 2년이 되지 않아 큰 회사이기는 하지만 좋은 회사인가라고 하면 쉽게 그렇다고 할 수 없습니다. 하지만 이 리더쉽 원칙은 많은 장점들을 가지고있고 온라인 서점으로 시작한 아마존을 많은 분야에서 선두를 달리고 있는 세계일류기업으로 만들어 준 원동력이라 생각됩니다.
(이헌수)
A :아마존 평균 근속기간은 2년이 되지 않아 좋은 회사가 아닐 것이라고 생각하는 경우가 있는데 구글도 그렇고 ...사실은 그와 다릅니다. 사실은 그에 상응하는 대우를 해 주고 그곳에서 벌어서 모은 돈과 그 곳에서의 경험으로 더 나은 대우를 해 주는 곳으로 ... 적어조 자신이 원하는 근무 조건을 제공하는 더 나은 곳으로 이직하여 가는 경우가 대부분이므로 그곳에서 그것은 전혀 문제가 되지 않는 답니다. (이상구 교수님)
2. Q.한국의 수학사를 나누는 기준 . (19.05.08)
● Q :유럽의 경우 고대 중세 근대 등으로 수학사를 나누는 기준이 있었고 또 그에 따라 수업을 진행해 왔습니다. 그렇다면 한국의 경우 시기적으로 수학사를 나눌수 있거나 또는 다른 기준으로 나누어서 정리할 수 있는지 궁금합니다. (김기헌)
A : 19세기 말까지의 산학자들이 해 온 전통산학과 19세기 말 부터의 근대수학 그리고 1945년까지의 식민지 교육정책에 따른 초등-중등 수학교육 과 연희전문학교 수물과에서 가르친 수학 그리고 해방 후 한참 지나면서 현대수학교육과 수학에서의 대학과정의 고등교육이 자리잡히면서 현대수학연구 가 큰 구분이 될 수 있습니다. (이상구교수님)
A : 유럽의 경우 고대 중세 근대 등으로 수학사를 나누는 기준이 있었고 그에 따라 수업을 진행해 왔다. 그렇다면 한국의 경우는 어떤 기준으로 수학사를 나눌 수 있는가. 한국 수학사에도 몇가지 큰 반환점이 있었고 그것이 한국 수학사를 나누는 기준으라 할 수 있다. 그 기준점으로는 19세기 말까지 산학자들이 해온 전통산학, 19세기 말부터 세계수학의 영향을 받은 근대수학, 일제강점기의 1945년까지의 식민지 교육정책에 따른 초.중등 수학교육 과 연희전문학교 수물과에서 가르친 수학, 해방 후 현대수학교육과 수학에서의 대학과정의 고등교육이 자리잡히면서의 현대수학이 있다. (이헌수)
3. Q. 한국의 수학사 수학의 역사. (19.05.22)
● 동아시아의 수학은 근대까지 서양보다 앞서있었지만 현대수학은 서양수학이라 할 수 있어 서양수학을 중심으로 그 역사가 서술되어 수학사에서는 많은 흔적을 찾아볼 수 없다. 우리나라는 중국의 수학의 영향을 많이 받아왔고 훌륭한 수학자도 있었지만 수학의 역사를 소홀히 해왔던 것도 사실이다.
아라비아 숫자가 들어오기 이전까지 숫자는 한자로 쓰였고 계산은 산대 라는 것으로 했다.
백제는 화려한 문화를 미루어봤을 때 산학이 발전된 것으로 보이고 통일신라는 그것을 이어받은 것이로 보인다. 하지만 통일신라시대에는 산학을 가르쳤다는 짧은 기록만 있을 뿐 산학연구가 활발하지는 않았다. 고려시대에는 수학의 과거인 명산과가 치러졌고 세금계산과 역법 등에 활용했다. 조선시대에는 조세, 측량 등 실용적인 목적을 가진 수학을 중심으로 발전하였다.
한국 수학은 중국수학의 영향을 많이 받은 것이 사실이지만 무조건적인 모방이 아닌 중국산학책의 재출간, 재해석, 응용을 해왔다.
● 주판과 산대는 정해져있는 계산, 단순한 계산만 가능하다면.. 미적분같은 복잡한 계산이나 숫자 단위가 큰 계산은 전자계산기 발명이전까진 손으로만 계산한것인지 아니면 다른 도구나 방법이 있었는지 궁금합니다 (이헌수)
● 태반이 당연히 반이라는 단어 때문에 절반정도를 말하는 줄 알았는데 3분의2라는 것은 정말 흥미롭네요. 중반이 정말 절반이라는 의미고요. 성리학이나 유교등으로 유명했던 조선이나 옛날 한국의 나라들도 수학을 꾸준히 신경쓰고 발전시키려고 노력했던 것을 보면 수학의 중요성을 절감합니다. 요즘도 BK사업등으로 밀어주려고 하는 것을 보면 한국 수학도 부흥할 수 있지 않을까요 (최민수)
● 지즈강의 <수학의 역사>
중국인들이 보통 주장하길 피타고라스 정리도 중국의발견이고 십진법도 중국의 것이었다고 하며 산발적으로 어떤 것이 있다라고만 말하지 그 개연성과 연속성을 들어가며 조리있게 설명하지 않는다. 하지만 이 책은 근대까지만 해도 서양보다 뛰어났던 중국의 수학이 아라비아를 거쳐 서양으로 흘러들어갔고 그것이 세계수학이 되어 다시 중국으로 들어온 것을 개연성, 연속성 있게 잘 설명해주는 책이다. (이상구 교수님)
● 피타고라스의 정리를 그렇게 옛날부터 발견해서 실제로 사용하고 있었다니 대단합니다. 십진법은 지금 모두가 사용하고 있는데 그것의 시초 또한 중국으로 볼 수 있다는게 정말 부럽습니다. (최민수)
● 중국의 대표적인 근대 수학자
후 밍푸 (1891 - 1927) : 중국인 최초의 수학 박사 (코넬대 수학박사)
화뤄겅 (1910-1985) : 중국의 해석적 정수론, 행렬기하학, 군론, 함수론 등의 연구의 창시자이자 개척자입니다.
천 싱선 (1911-2004) : 중국인 중 가장 먼저 세계적으로 first-class의 수학자로 인정받은 분으로 어린 시절부터 수학적인 재능을 보이며 미분기하학 분야에 탁월한 공헌을 하신 분입니다. (필즈상 수상자인 하버드 수학과 교수 Yau 의 박사 지도교수입니다.) (이상구 교수님)
● 동아시아의 화려했던 수학이 세계에서 인정받지 못하고 서양위주로 쓰인 것은 안타깝게 생각한다. 예전에는 물리적으로도 멀고 정보를 공유할 수 있는 방법도 없고 그럴 생각도 하지않았지만 이제는 학문이나 문화 등 모든 것이 세계가 하나로 묶여 서로 공유해가고 영향을주며 발전하고 있기에 이런 일은 일어나지 않을 것 같다. (이헌수)
● 동아시아가 제일 먼저 생산한 수학 중 현재 세계가 사용하고 있는 수학은 무엇인지 ...
사회 발전에 크게 기여한 수학의 분야 중 동아시아가 최초로 발견한 것은 무엇인지? 등
을 재발견하는 것을 동아시아의 수학사학자들이하고 잇는 것이랍니다. (이상구 교수님)
4. Q. 우리가 추구해야할 덕목, 교육의 변화 …"學而時習之면 不亦說乎아'의 열(說)은 나누라는 의미 . (19.05.23)
● 우리나라의 많은 학생들은 자신이 왜 공부해야하는지도 모르고 하기 싫은 공부를 억지로 하고있다. 하지만 학이시습지면 불역열호아라는 말처럼 공부하는것을 좋아하고 즐기는 학생이 좋은 결과를 내고 또 앞으로고 꾸준히 잘 해나갈것이다. 또한 공부하고 그것을 남들과 나누는 것 또한 큰 공부이다. (이헌수)
● 요즘 우리는 열심히 공부는 하지만 무엇을 위해 공부하는지는 잘 생각하지 않는 것 같습니다. 좋은 대학, 좋은 직장을 위해 막연히 점수를 올리기 위한 공부를 하기에 바쁩니다. 물론 사회 구조가 그러니 어쩔 수 없는 것도 사실입니다. 하지만 발전을 위해서는 학문 그 자체에 집중해 열을 가지고 학업을 증진시켜 나가야 한다 생각합니다. 이를 위해 사회가 개인을 도울 필요도 있다고 봅니다. (김기헌)
● 확실히 동의하는 내용입니다. 자신이 좋아하는것은 해도해도 질리지 않고 또 항상 그것에 몰두하기때문에 자연스럽게 좋은 결과가 따라오는 것 같습니다. 또 남들과 나누면서 새로운 것을 터특하며 더욱발전하는것도 아주 좋아보입니다. (이헌수)
5. 중국의 산경십서와 조선의 산학 . (19.05.27)
● 중국의 산경십서는 한에서 당에 이르는 이 고전 수학서를 말한다. 주비산경, 구장산술, 해도산경, 손자산경, 오조산경, 장구건산경, 하후양산경, 오경산술, 수술기유, 집고산경이 10산경이다. 초기에는 조충지의 철술이 들어있었지만 후에 난해하다는 이유로 빠지고 집고산경이 들어갔다.
제도화된 한국 수학은 중국 수학을 본따서 천문관측과 역산이 활발히 이루어진 삼국시대때 시작되었다고 볼 수 있다.
682년에 중국의 산학서들로 학생들을 가르쳤다는 것으로 보아 훨씬 전에 한국에 유입되었을 것으로 추정된다. 산학(고대수학)은 측량, 조세 , 국고 등을 관리하고 필요와 국가의 권위때문에 발전하게 되었다, 하지만 통일신라 초기에는 활발하게 산학을 연구하였지만 말기에는 필수적인 회계정도만 사용하는 수준으로 퇴보하였다
고려도 별반 다를것 없이 통일신라의 산학 제도를 도입하여 가르치고 연구하는 수준에 머물렀다. 중국이 엄청난 수학자들을 동시대에 배출하면서 저술도 활발하였다는 것과는 대조적이라 안타깝다고 할 수 있다.
조선시대에는 이런 수학연구의 정체기를 지나서 세종때부터 행정상의 필요때문에 수학교육을 장려하고 시행하였고 세종부터 솔선수범으로 공부하였다.
두번의 왜란으로 자료는 소실되었지만 그 이후에 산학의 규모가 커지고 활발하게 연구는 지속되었다. (이헌수)
● 중국수학에서 전파받아서 꾸준히 사용하다가 정체되던 때도 있었지만 다시 필요에 의해서 연구해나가는 조상들의 모습을 보면서 조금만 더 일찍 시작하였다면 중국에 밀리지 않았을거란 생각이 들었다 (최민수)
● 우리나라가 일찍 수학의 중요성을 깨닫고 주요과목으로 연구하기 시작했으면 어땠을까.. 세계적으로 수학과 과학이 눈부시게 발전하고있던18, 19세기 우리나라가 조선이었던 이 시기에 세계의 흐름을 따라 함께 발전했더라면 하는 아쉬움이 든다 (이헌수)
● 임진왜란 이전의 조선 산학책을 한권이라도 찾으면 ... 그간 조선을 bypasss 하려고 불안한 논리를 주기적으로 주장하는 ... 일본 수학사는 완전히 다시 써야 합니다. (이상구 교수님)
-
임진왜란 당시 일본의 발전 원인
Ask by 김윤수, Date: 2019.03.06
Finalized by 김윤수, Date: 2019.03.20
Final OK by SGLee
<질문> - 김윤수
일본은 그런 기본적인 지식도 없는 상태로 어떻게 서양과 무역을 할 수 있는 해양기술을 보유할 수 있었을까? 단순히 섬나라라서 라고 생각할 수 도 있겠지만 일본이 본인들도 자각하지못하는 과학기술의 발전 사상을 보유하지는 않았을 까 하는 의문이 듭니다.
<답변> - 이상구
당시 임진왜란 전까지 중국과 조선은 현재의 반도체나 통신장비의 지식 같이 당시 최고 지식, 국가/기업 극비 정보를 담은 조선의 첨단 산학책을 일본에게 안 팔고, 보여주지도 않고, 정상적인 무역도 안하였습니다.
그럼에도 네델란드에서 온 화란인들과 (타의로) 물물교환을 시작하면서 서양과 무역을 할 수 있었습니다.
그러나 조선 산학이 무엇인지 잘 아는 일본 사람이 없었다는 의미지 일본이 기본적인 기술, 배 만들고, 항해하고 하는 기술이 조선 보다 크게 못했다고 얘기 할 수 없습니다.
조선 시대에는 이미 일본도 나름대로 자신의 기술에 보태 백제로 부터 전래된 과학기술 과 인재들을 보유하고 있었을 것이고...
<정리> - 김윤수
일본이 임진왜란 전에 서양과 교역을 한 것이, 일본의 기술로 서양까지 간 것인 것으로 착각하였습니다. 일본은 네덜란드에서 온 서양인들과 교역을 한 것이고, 그들을 통해 무기를 수입한 것이었습니다.
서양에서 일본까지 갈 수 있는 기술이 있다면, 우리나라로도 분명히 왔을 텐데, 이를 받아들이려는 노력을 하지 않은 것이 안타깝습니다.
미분의 발전 필요성
Ask by 김윤수, Date: 2019.03.13
Finalized by 최민수, Date: 2019.03.13
Final OK by SGLee
<질문> - 김윤수
미분과 적분중에 적분이 훨씬 먼저 개발되었는데, 이 요인은 생각해보면, 측량의 중요성 때문이라고 인지하고 있습니다.
대포의 화력증진을 위해 함수가 발전되었다고 하셨는데, 미분은 어떤 필요성에 의해 개발된것인가 하는 의문이 듭니다.
그냥 함수를 연구하다보니 미분이 따라온 것인지, 아니면 응용수학의 미분방정식을 식으로 정리하고 이해하기 위해 미분이 발전된 것인지 궁금합니다.
<답변> - 이상구
답: https://science.jrank.org/pages/2014/Derivative-History-usefulness.html
기본적으로 대포의 성능 개선을 하는 과정에서 연구 결과들이 누적되면서
... 오늘 수업 시간 강의한 교재의 내용이 그것을 설명한 것인데 ..
오늘 교안을 다시 한번 보고, 한번 더 물어보시면 어떨까요?
<정리> - 김윤수
대포의 성능을 향상시키기위해서 접선의 기울기를 연구했다는 말씀이신것이군요! 포탄이 떨어지는 위치만 생각해서 성벽을 무너뜨리는 과정에서 접선의 기울기와 대포성능이 연관되있있다는것을 생각하지 못했습니다
에드워드 솔프의 '블랙잭 필승 전략'
Ask by 김윤수, Date: 2019.03.25
Finalized by 최민수, Date: 2019.03.27
Final OK by SGLee
<질문> - 김윤수
에드워드 솔프의 '블랙잭 필승 전략'은 무엇일까?
만약 그런 방법이 있다면, 카지노는 그에 대비해서 어떻게 대처했는가 하는 의문이 듭니다.
<답변> - 이상구
실제 카지노 가 솔프의 '블랙잭 필승 전략' 에 어떤 방식으로 대응했는지 ... 카지노 매니저에게 물어볼까요? 답: 규칙을 만들어 <블랙잭에서는 카운팅을 금지합니다.>
[일반상식] 블랙잭의 개념,이론,전략 : 네이버 블로그
http://blog.naver.com/PostView.nhn?blogId=sunfish5&logNo=20115454726
<답변> - 김기현
교수님께서 답변을 잘 주셨습니다. 이에 추가적으로 학문적인 자료는 아니지만 참고하면 좋을 영화 한 편을 소개합니다.
https://movie.naver.com/movie/bi/mi/basic.nhn?code=69040
위 영화는 교수님께서 답해주신 내용 중에 있는 카운팅이란 기술을 활용하여 라스베거스의 카지노에서 돈을 따는 내용으로 진행됩니다. 이에 카지노 측은 돈을 잃지 않기 위해 카운팅을 활용하는 배팅러들을 찾고 감시하는 감시관을 따로 고용합니다. 대략적인 답이 될 것이라 생각합니다.
<답변> - 김윤수
블랙잭은 카드카운팅 기술이 있고, 상황상황마다 적절한 판단을 한다면, 게이머에게 유리한 게임이라는 연구가 있다.(1956년 ,이기려고 블랙잭을 해라: 블랙잭의 새로운 전략)
단 0.1%의 차이지만, 유리한 것이 사실이고, 큰 수의 법칙을 생각해본다면, 이는 카지노측의 손해로 이어져야 하는데, 실상은 미숙한 운영때문에, 카지노의 수익중 많은 부분을 차지하고 있다고 한다.
카드 카운팅에 대해서 카지노 측이 금지한다고 하셨는데,
제가 찾아본 자료에 의하면, 카지노 측에서 금지하는 카운팅은 전자기기를 이용한 카운팅이고, 그것을 제외한 대부분의 것들은 금지할 수 없다고 한다.
[ 기본적으로 카드카운팅은 카지노에서 전혀 금지하지 않는다. 카드카운팅은 플레이어의 머리속에서만 이루어지는 것이기에, 애초에 금지할 수도 없다.
도구를 이용하는 경우도 있는 데 이 경우는 케바케. 대부분의 카지노에서 종이와 연필을 이용해서 메모하는 것을 제지하진 않지만, 그렇다고 대놓고 카드카운팅을 하면 제지당할 수는 있다. 스마트폰 같은 전자기기를 이용하는 것은 당연히 금지된다. 그런데, 카지노에서 자유롭게 사용할 수 있는 도구가 하나 있는데 바로 '칩' 이다. 칩을 이용해서 한다면, 제지하는 경우는 드물다. ]
물론 카드 카운팅에 대한 방지 대책으로 매 경기 카드를 셔플한다던가, 여러덱을 한번에 넣는다던가 하는 방법이 있다고 하긴 하지만, 카드카운팅이 의미가 없는 것은 아닐 것이다.
추후에 기회가 된다면 좀더 공부하고 카지노에서 블랙잭을 즐겨보고 싶은 생각이 든다.
<답변> - 최민수
https://www.quora.com/Does-the-strategy-of-Ed-Thorp-in-Beat-the-Dealer-still-work-today
https://www.amazon.com/Beat-Dealer-Winning-Strategy-Twenty-One/dp/0394703103
실제로 솔프가 쓴 책인 비결이 적힌 Beat the dealer 라는 책이고 히트, 스탠드 ,더블다운, 스플릿 페어 등을 언제 사용할 것이냐 라는 것이 적혀 있습니다.
하지만 카지노에서는 이것에 대처하기 위해서 Single Deck 이였던 것을 7에서 8덱으로 늘리고 세부 룰을 계속 바꿔나가는 식으로 대처해나갔습니다.
또한 이러한 책이 잘 팔려도 카지노가 꾸준히 수익을 낼수 있는 이유는
1. 이 전략을 완벽하게 실행할 수 있는 사람은 적다
2. 카운팅이나 돈을 확연하게 버는 것이 티가 나면 쫓아낼 수 있다.
3. 룰과 덱을 수정해서 더이상 전략이 통하지 않도록 한다.
등이 있습니다.
하지만 카드 카운팅과 전략의 기본 틀은 블랙잭의 기본뼈대가 변하지 않는 한 언제나 유용한 기술입니다
반원과 직선의 점의 개수
Ask by 최민수, Date: 2019.03.27
Finalized by 최민수, Date: 2019.03.27
Final OK by SGLee
<질문> - 최민수
제가 집합론을 공부했었는데 그때 배우기로는
Aleph alpha + n = Aleph alpha (for every natural number n, for every alpha) 이라고 배웠습니다
따라서 반원의 양 끝 두 점을 일단 제외하면 직선과 일대일 대응관계가 성립하고
직선은 실수의 Cardinality와 동일하며 아마 Aleph1 로 예상되는데 그러면 갯수가 더 많은 것이 아니라 점의 갯수는 동일하지 않는가 라고 생각됩니다
<추가질문> - 이현수
수업시간에 반원과 직선의 점의 갯수 설명 들을 때 일대일대응 설명하시며 말씀해주셨는데,
사실 직관적으로는 그런 개념이 혼란스럽습니다... 그러면 아무리 작은 원이라도 아주아주 긴 어떤 직선보다도 항상 점의 갯수가 더 많다 이것도 맞는 말이 되는건가요?
또 예를들어
ㅡ
ㅡㅡ
ㅡㅡㅡ
이렇게 길이가 각각 1,2,3인 직선이 있을 때에는 일대일 세가지 직선 모두 일대일 대응이 가능해 보이는데 그럼 이 세 직선의 점의 갯수는 같은건가요...?
<추가질문 답변> - 강민석
무한론을 가장 일반적인 언어로 풀어 쓴 예로는 힐베르트의 '무한 호텔'이 있습니다.
간단하게 찾을 수 있으니 찾아보세요.
이에 따르면 무한 + n(자연수) = 무한, 무한 X n(자연수) = 무한이라는 명제를 쉽게 이해할 수 있습니다.
그냥 말장난 같기만 할 수도 있으니 머릿속으로 받아들여지느냐의 차이는 있지만...
슈뢰딩거의 고양이나 디렉 델타 함수도 발표 당시에는 학계의 수많은 사람들에겐 말장난으로 받아들여졌으니 결국 받아들이기 나름입니다.
이를 응용하면 자연수의 개수는 2의 배수의 개수와 같습니다. 1대 1 대응하니까요.
유리수의 개수와도 같습니다. 분모를 p로 고정시키면 분자 q와 1대 1 대응하니까요.
무리수의 개수는 더 많습니다. 칸토어의 대각화 논증으로 증명되는데,
그 대각화 논증에서 보이는 논리는 결국 무리수는 유리수와 1대1 대응이 불가능하다는 증명입니다.
물론 1대 1로 대응시킬 수 없어도 같다고 보는 경우도 있습니다.
반원의 점의 개수는 직선의 점의 개수보다 2개 더 많아 완벽한 1대1 대응이 불가능하지만 같은 무한으로 칩니다.
무한보다 작은 실수 범위의 차이이므로 같게 보는 겁니다.
이제 직선의 길이가 달라도 점의 개수가 같나는 물음에 답하자면,
1대 1 대응하므로 같습니다.
<추가질문 답변> - 김윤수
부피가 없는 직선이기 때문에, 이를 늘리거나 줄여도 사실 개수는 같다고 생각하면 이해가 될 것 같습니다.
고무를 늘였다가 뭉쳤다가 하는 것을 생각하시면 될 듯 하네요.
비슷한 예시로 원과 타원의 점의 개수도 같지 않을 까 생각합니다.
여기서 말하는 개수가 같다는것은
고등학교에서 극한을 배울 때 느꼈던 것처럼
(n^2+1)/n^2 이 n이 무한대로 갈 때 1 로 수렴하는 것과 같은 이치라고 생각하시면 됩니다.
<답변> - 이상구
맞습니다. 책의 설명은 무한에서는 2개 더 많은 것이 큰의미가 없다는 것을 설명하려고 그 예를 그런 식으로 들어 준 것입니다.
맞습니다. Aleph 알레프 0 + n = Aleph 알레프 0 (for every natural number n, for every alpha) 입니다.
그러나 직관적으로는 적어도 2 점이 더 많은 것을 느낄 수 있습니다.
맞는 말입니다.
책의 설명은 무한에서는 2개 더 많은 것이 큰의미가 없다는 것을 설명하려고 그 예를 그런 식으로 들어 준 것입니다.
<정리> - 최민수
Aleph alpha + n = Aleph alpha (for every natural number n, for every alpha)
는 성립한다
책에서는 무한의 레벨에서는 별 차이가 없다는 것을 강조하려고 했을 뿐이다.
실제로는 반원쪽이 일대일 대응보다 점이 많긴 하지만 무한의 레벨에서는 차이가 없다고 봐도 무방하다
베르트랑의 역설
Ask by 최민수, Date: 2019.03.27
Finalized by 최민수, Date: 2019.03.27
Final OK by SGLee
<질문> - 최민수
"주어진 원에 현을 하나 임의로 그었을때 그 현의 내접 정삼각형의 한 변보다 길어지는 확률을 구하라" 가 우리가 수업시간때 배웠던 문제였습니다
그런데 이 답에 대해서 논란이 있습니다
그래서 그것을 소개하고 같이 생각해보고자 합니다
Q1. 여러분들은 세가지 해법중에 어떤것이 타당하다고 생각하시나요?
Q2. 확률의 고전적정의에 의해서 생긴 난제라고 하는데 그렇다면 좀더 엄밀한 새로운 정의는 어떤 것들이 가능할까요?
<답변> - 김윤수
직관적으로는 도저히 알 수 없는 내용들이네요.
고적적인 정의에서의 확률은 = 사건이 일어날 수 있는 경우의 수 / 가능한 모든 경우의 수
인데 각각의 확률변수를 길이, 각도, 넓이로 다르게 생각한 것이 흥미로웠습니다.
답은 1/3이라고 하는데
1/2로 나오는 경우 : 반지름을 적분하는 방법으로 원의 넓이를 구할 수 없기 때문에 모순
1/4로 나오는 경우 : 원의 넓이는 무한한 공간이 아니기 때문에 모순
이라고 합니다.
bertrands-paradox.com
<답변> - 이상구
맞습니다. 쉽게 증명되는 내용이 아닙니다.
아래 논문이 답의 일부를 설명하고 있습니다.
https://aip.scitation.org/doi/10.1063/1.4890291
<답변> - 조영은
시각화한 그림상에서 문제의 조건인 '임의로 현을 긋는다'에 가장 적합한 해법은 무엇일까요?
저는 원 내부에 현이 최대한 비편재되어야 '원에 임의로 현을 긋는다'라는 조건을 만족할것이라고 생각했습니다.
그래서 처음에는 두번째 줄의 가운데 그림이 원에 현이 가장 골고루 배치되었다고 생각했는데
다시 생각해보니 첫번째 줄의 3번째 그림이 현의 중심이 가장 비편재된것 같기도 해서 뭐가 적합한 해답인지 잘 모르겠습니다.. 혹시 다른 분들은 어떻게 생각하시나요?
<답변> - 이현수
임의의 현 이라는 정확하게 정의되지 않은 말 때문에 이런 역설이 발생하다니 신기하네요
제 생각에는 3번이 가장 방법인 것 같습니다 원 안의 모든점을 고려해서 생각한것이 가장 합리적이어 보입니다..
<답변> - 최민수
저랑 같은 의미로 하시는 말씀일수도 있지만
점의 갯수는 현에서도 Aleph 1 만큼 많고 면적에서도 Aleph 1 만큼 많기 때문에
갯수/갯수 등의 확률로 접근하면 원하는 확률값이 안나올 가능성이 높기 때문입니다
<답변> - 조영은
답변해주신 내용 중에 'Aleph1'이라는 말이 무슨뜻인지 모르겠어서 여쭤봅니다.
<답변> - 최민수
수학자들은 무한집합에서도 원소의 갯수가 크기가 다르단 점을 인식했고
그것에 단위를 부여해서 구별해주기 위해서 수를 도입했습니다
Aleph null 은 자연수집합의 원소의 갯수입니다
Aleph 1 은 실수집합의 원소의 갯수입니다
자연수집합과 실수집합은 1대1 대응이 안됩니다
그래서 aleph 1 이 더 큽니다
1. 제논의 역설 (Finalized by 최민수, Final OK by SGLee)
Question by 최민수
제가 고등학생때 제논의 역설을 처음 들었을때는
그리스 시대에는 무한에 대한 개념과 무한급수에 대한 개념이 정립이 되어 있지 않았기 때문에, 그 개념들이 도입된 이후에는 해결이 되었다고 배웠습니다.
하지만 제가 '두뇌는 최강의 실험실' 이라는 책을 읽어본 뒤에는 생각이 좀 달라졌습니다.
제논은 단순히 무한급수와 무한의 개념을 몰랐던 것이 아니였고, 제논 역설의 전제와 결론은 전제: 시간과 공간은 무한히 분할 가능하다.
-> 결론: 아킬레스는 거북을 따라잡을수 없다.인데 배리법으로 전제자체가 오류이며 시간과 공간은 무한분할할 수 없다고 주장을 했을 가능성도 높다는 것입니다.
즉, 우리는 머릿속으로 무한의 개념을 이상화해서 수학적으로 유용하게 쓰고 있는데 과연 이것은 괜찮을지,
질문: 무한이 수학에 정식으로 받아들여지던 과정이 궁금합니다.
Reply by 강민석
아는 이야기들을 쭉 늘어놓아보겠습니다. 하나하나 레퍼런스를 찾긴 힘들지만 뒤지다 보면 많이 나올 겁니다.
제논의 역설이 말도 안 되는 이야기인 건 다 압니다. 문제는 이를 제논의 방식에 맞춰 논파해야 한다는 점입니다.
아킬레우스와 거북이 논쟁을 중심으로 말하겠습니다. 다른 문제들도 비슷한 논리로 다 풀립니다.
경기장의 역설은 내용조차 못 찾겠습니다. 영어는 서툽니다...
일단 역설의 내용은 다 아실 것이니 넘어가고, 이 역설의 주요 논점은 다들 이 부분이라 알고 계실 겁니다.
"아킬레우가 거북이를 따라잡는 동안 거북이는 좀 더 나아간다. 이를 무한히 반복하면 따라잡을 수 없다.
이는 즉, 0보다 큰 수를 무한히 더하면 무한이 된다는 것을 암시한다.
0보다 큰 수를 무한히 더해 유한하다는 걸 증명하면, 제논의 역설은 논파된다."
무한급수라는 개념의 탄생입니다.
논점은 정해졌으니 반박할 일만 남았습니다. 최초로 "수학적으로 유의미한" 반박을 한 사람은 뉴턴입니다.
어떻게 무한급수를 학계에 내놓고 인정받았는지를 언급한 답글이 있으므로 굳이 추가 언급은 않겠습니다.
하지만 완전한 반박은 아니었고, 차후 코시가 극한을 정의하고 무한급수의 성질을 완전히 규명함으로서 이 논점은 완파됩니다.
(무한급수도 합의 극한이니 극한의 개념이 없고서는 논하기 힘들었던 것이 큽니다.)
하지만 제논의 역설에서 또 다른 흠을 찾아낸 사람도 있습니다.
버틀런드 러셀은 게오르그 칸토어야말로 제논의 역설을 완전히 논파했다고 보았습니다.
해석학 Chapter.1에 나오는 개념입니다. 이 개념을 만든 사람이 칸토어입니다.
대부분 쉽다고 넘어갈 수 있을 개념입니다.
가산집합과 불가산집합입니다. (Countable/Uncountable)
제논의 역설을 따르자면, 아킬레우스는 거북이를 따라잡고 거북이는 그 동안 더 나아가는 과정을 반복합니다.
무한히 이 과정이 이어지는 중간에 아킬레우스는 어느 순간에든 자기가 어느 지점에서 몇 번 거북이를 따라잡았는지 "셀 수 있습니다."
즉, 아킬레우스가 거북이를 제치기 직전까지 아킬레우스가 나아간 거리는, 거북이를 따라잡은 횟수, 즉 가산집합으로 치환할 수 있습니다.
이는 직선상의 거리 = 가산집합이라는 가정을 내포합니다.
하지만 실제로 직선 위의 점의 집합은 불가산집합입니다. 칸토어는 직접 이를 증명합니다.
가산집합 = 불가산집합이라는 성립할 수 없는 명제가 탄생하므로 모순. 이것이 가장 제논의 역설을 올바르게 논파하는 방법입니다.
칸토어는 이 개념을 시작으로 "무한론"을 창시합니다. 무한론에 따르면 자연수와 2의 배수, 유리수의 집합은 서로 동등합니다. 가산집합이니까.
하지만 무리수의 집합은 유리수의 집합보다 큽니다. 실수의 집합은 무리수의 집합과 크기가 같습니다. 대각화 논증으로 증명됩니다.
불가산집합은 가산집합보다 얼마나 큰가, 이는 칸토어도 몰랐습니다.
하지만 얼마나 클지 대충 가설은 세워보았습니다. 이것이 연속체 가설입니다.
이후 이 연속체 가설이 현대 수학사의 핵폭탄이 되어 여러 수학자와 수학 체계를 파탄내고 좌절시키지만 이는 또 다른 이야기로 남겨두고,
여하간 무한이 수학사에 받아들여진 과정은 대충 이러합니다.
2. 정리인가 추측인가 (Finalized by 최민수)
Question by 이헌수
페르마의 마지막 정리와 푸앵카레의 추측을 보며 의문점이 들어 질문드립니다. 다들 어떻게 생각하시는지
페르마의 마지막 정리는 왜 정리 라고 이름이 붙었고 푸앵카레의 추측은 왜 추측이라 이름붙은걸까요?
엄밀히 말하면 페르마도 증명 없이 (본인은 증명했다 주장했다하지만) 내용만 말한것이고 푸앵카레도 증명없이 내용을 말한것인데 페르마는 정리라고 이름붙이고 푸앵카레는 추측이라고 할까요?
Reply by 이상구
<--- 맞습니다. 증명이 된 것은 정리, 아직 증명도 반예도 못 찾은 것은 추측입니다.
Question by 이헌수
푸앵카레의 추측도 증명되었으니 그럼 이제 푸앵카레의 정리가 되는건가요? 증명한지 20년이 다 되어가는데 아직 모두 푸앵카레의 추측이라 부르는 것 같습니다
Reply by 이상구
<-- 맞습니다. 이제는 정리입니다. In mathematics, the Poincaré conjecture (/ˌpwæ̃kɑːˈreɪ/; French: [pwɛ̃kaʁe]) is a theorem about the characterization of the 3-sphere.
Reply by 강민석
참고로 페르마의 마지막 정리를 정리라 부르는 이유는
자기는 증명했다고 했던 페르마를 존중하는 의미에서지 큰 뜻은 없습니다.
완전히 증명된 지금은 페르마-와일스 정리라고 부르기도 합니다.
Finalize by 최민수
증명이 되었을 경우 정리라고 하고 꽤나 설득력 있고 의미있는 명제인 경우에 추측이라고 한다.
페르마의 마지막 정리의 경우에는 최고의 수학자 중 하나인 페르마가 증명을 했다고 주장하고 다른 많은 사람들도 풀었다고 주장을 했었던 바가 있었으므로 추측이였던 시절이 길었음에도 정리라고 명명했었다.
푸앙카레 추측의 경우에는 추측이였던 기간이 또한 길어서 요즘에도 추측이라고 부르는 사람들이 많지만 이제는 정리이다.
3. 클라인 병에 관한 질문 (Finalized by 조영은)
Question by 김기헌
수업에서 위상수학에 대한 수업 때
뫼비우스의 띠와 클라인 병에 대해 잠깐 언급 됐습니다.
뫼비우스의 띠는 공장 컨베이어 벨트에 활용이 되고 있습니다.
그럼 클라인 병은 어떤 곳에서 활용되고 있으며 어느 분야에서 활용 가능성을 찾을 수 있을까요??
Reply by 조영은
저도 클라인 병이 어디에 활용될지 궁금해서 찾아보았는데
시각적으로 눈에 띄고 독특한 클라인 병의 구조를 본뜬 건축물이 있더라고요.
클라인병주택, [건축디자인][건축이야기] 클라인 병 주택, MCR Architecture
주말주택으로 사용되는 Klein Bottle House 주변에는 나무가 둘러싸여 있고, 해변에서 그리 멀지 않은 모래 언덕 위에 자리 잡았다. 호주 멜버른에서 한 시간 가량 차로 이동해야 한다. 클라인 병. 이는 독일 수학자 클라인의 이름을 딴 면이 하나밖에 없는 도형으로, 내외부의 구분이 모호하다. 또한 안과 바깥의 구별이 없어 방향을 정할 수 없는 4차원 도형이다. MCR 아키텍츠는 이 병을 기본 콘셉트로 한 주택을 설계한다. 그들은 그 동안 구상적인 방법이든, 추상적인 방법이든 다양한 형태로 그들이 의도한 테마와 소재를 건축 언어로 무난하게 소화시키며 즉각적인 형태를 완곡하게 새로운 어휘로 전환시켜 왔다. 실내에 위치한 거대한 계단은 다양한 레벨의 공간을 연결해주고 있고, 서로 접근하기 용이하게 적당한 거리를 유지시켜 공간을 독식하지 않는다. <이경택 에디터>
Reply by 최민수
클라인병은 뫼비우스의 띠와는 다르게 활용가능성이 좀 낮습니다.
뫼비우스의 띠는 안과 밖이 구분이 되지 않는 띠라는 성질이 있고 어떤 바깥면이나 안쪽 면에서 시작해서 면을 따라 이동하다 보면 결국 안과 밖을 따라서 원래 위치로 돌아오는 성질이 있습니다.
이를 이용해서 에스컬레이터 손잡이나 컨베이어 벨트등 내구도가 중요한 부분에서 사용기한을 두배로 늘릴 수 있는 장점이 있어서 사용하는 것입니다.
그와 달리 클라인 병은 애초에 4차원 존재이기 때문에 현실에 존재할 수 조차 없고 우리가 알고있는 클라인 병의 모양은 3차원으로 본 모습일 뿐입니다.
즉 애초에 존재조차 못하기 때문에 학습용 외에는 사용하기가 무리입니다
아마존에서 클라인 병 모양의 물병을 팔고있기는 합니다.
Question by 조영은
그럼 클라인병은 현실에서 존재하지 않는것인가요?
클라인병은 4차원에서만 존재할 수 있다고 하셨는데
그러면 교재에 나와있는 클라인 병의 구조가 실제 클라인 병의 이론적인 모습과 다르다는 말씀이신가요?
교재에 나와있는 클라인병의 사진을 보면 '클라인 병에 액체를 넣으면 흘러나온다'라고 나와있는데
그림으로는 이해가 되지 않아서 여쭤봅니다.
Reply by 최민수
어느정도는 반영되어 있지만 실제와는 다릅니다.
예를 들자면
우리가 책에있는 삽화등을 볼때 3d물체를 본다고 생각하지만 실제로는 2d에 불과합니다. 우리가 경험등과 원래 어떻게 생겼는지를 기억해서 입체감을 스스로 부여해서 보고 있을 뿐이죠,
클라인 병도 실제로는 뚫고 들어가는 듯한 부분이 존재하지 않고 안과밖이 구분이 되지 않으므로 안같이 보이는 부분에 물을 부을수도 없습니다.
실제로 어떻게 생겼는 지는 3d에서 완벽히 나타낼수 없기 때문에 추상적으로 받아들이시는게 이해하기 좋을 것 같습니다
Reply by 강민석
이해가 안 되는 게 당연합니다.
이걸 보시면 좀 납득이 되실 겁니다.
Finalize by 조영은
클라인 병은 4차원 구조로 현실에서는 존재하지 않는 구조이다.
그래서 '안과 밖의 구분이 없다'는 성질을 지닌 뫼비우스의 띠를 활용한 특허가 수백 건(ex. 공장 컨베이어 벨트나 에스컬레이터 손잡이, 양쪽 면에 녹음할 수 있는 테이프 등)인 반면, 클라인 병이 활용되는 예시는 거의 찾아보기 어렵다.
클라인 병의 모습을 본따 '클라인 병 유사 구조'로 건축물을 만들거나 물병과 같은 상품을 만들 수는 있으나
이들 역시 '클라인 병 유사 구조'이지 클라인 병의 구조는 아니다.
<클라인 병의 특징>
- 클라인 병 내에는 뫼비우스의 띠가 포함되어 있다.(클라인 병은 뫼비우스의 띠를 닫아 만든 곡면이다.)
- 뫼비우스의 띠 같이 안과 밖이 구분되지 않으므로 이론적으로는 클라인 병에 물이 담기지 않는다.
-클라인 병의 안쪽과 안쪽을 맞붙이면 토러스가 된다.
4. 문학의 분석과 데이터화
Question by 이헌수
문학을 분석하고 데이터화 한다면 사람들이 생각하는 완벽에 가까운 작품들이 많이 나올 것 같다는 생각이 드네요.
제 생각에는 바둑의 알파고와 비슷한 경우가 될 것 같아요. 알파고가 처음에는 바둑기사들의 바둑을 분석한 방대한 데이터를 토대로 점점 발전하여 인간을 따라잡고 딥러닝으로 더욱 발전하여 새로운 창의적인 수들마저 엄청 만들어내어 이제는 인간이 알파고에게 바둑을 배우고있죠
그것처럼 문학도 사람들이 명작이라고 생각하는 혹은 좋아하는 작품을 만드는데 단어,구조,감정,진행 등 여러가지 방면을 분석하고 그 데이터를 바탕으로 글을 쓴다면 좋은 작품들이 많이 나올 것이라 생각이 드네요. 또 창의성 문제도 과학에서 많은 변수를 고려하며 여러가지 실험을 하듯 문학에서도 여러가지 실험을 하며 그 반응을 또 분석하고 데이터화시키며 딥러닝을 한다면 창의성 마저 갖춘 작품이 나올 것 같다는 생각이 듭니다. 알파고가 바둑 세계랭킹 1등인것처럼, 문학에서의 베스트셀러 작가가 글쓰는로봇 혹은 그런 도움을 받은 작가가 될수도 있겠네요!
여러분들은 어떻게 생각하시나요??
Reply by 조영은
윗 글에서 밀씀해주신 것처럼 로봇이 문학작품을 쓰는 것도 데이터베이스를 이용한다면 충분히 가능하다고 생각합니다.
그렇지만 로봇이 쓰는 단어, 문장 형식, 어체 등은 과거에 어떤 사람이 사용했던 글을 토대로 한 것이므로,
새로운 아이디어나 독특한 어체, 아무도 생각해보지 못한 작품 소재를 사용한 글은 나오지 않을 것 같습니다.
예를 들어 ‘해리포터’나 ‘반지의 제왕’과 같이 사람의 상상력에서 탄생한 판타지 소설을 로봇이 만들어내기는 힘들 것입니다.
즉, 제 생각에 로봇이 쓴 문학작품은 완성도가 어느 정도 있긴 하지만,
누구나 예상할 수 있는 전개를 가진 다소 뻔하다는 평가를 받을 수 있는 작품들이 될 것 같습니다.
Reply by 이헌수
저도 영은님 말에 어느정도 동의합니다. 결론부터 말하자면 제 생각에는 아주 천재적인 인간 몇명과 대부분의 로봇이 주인공이 될 것 같네요. 소설 뿐만 아니라 다른 여러방면에서 그럴 것 같습니다.
독특한 어체, 단어, 구성 간단히 말해 '창의력' 이라고 불러도 될까요? 로봇 혹은 AI 는 인간과는 다른 방법으로 '창의력'을 길러나가는 듯 합니다. 한국에 불었던 AI열풍? AI에 대한 관심이 알파고로 인한것이니 알파고를 예를 들어보자면. 처음에는 인간을 흉내내던 알파고가 기존의 인간의 데이터를 학습하고 나서는 (기계이다 보니 방대한 테이터를 금방 학습해버렸습니다) 더 좋은 결과를 얻기위해 스스로 딥러닝 하여 (가능한 변수들을 바꿔가며 엄청난 양을 실험) 인간이 하지못했던 새로운 발상들을 해냈습니다. 새로운 경향성? 직관? 이라고 해야할까요.. 인간은 한번에 엄청난 양의 데이터를 처리하지 못하기때문에 천재들이 직관적으로 혹은 기발한 아이디어로 새로운 것을 만들어간다면 기계는 정말 방대한 양의 데이터 처리로 우리가 '창의성'이라고 불리는 것을 해내는것이죠
물론 알파고는 바둑AI 였고 목적은 상대방을 이기는 것 이니 다른 얘기일 수고 있겠죠
이것을 문학작품에 적용시켜본다면, 목적을 인간이 좋아하는 기존에 없었던 새로운 문학작품으로 설정한다면
로봇은 여러가지 단어,어체,구성 등 다양한 변수들을 바꿔가며 작품을 써보는 것이죠. 그리고 이것에 대한 반응을 데이터화 시켜서 경향성을 얻는 것이죠 즉 새로운 직관을 가지는 거네요. 새로운 작품을 쓰는 작가 탄생!
제 생각에는 로봇의 빠른데이터 처리속도와 빅데이터의 힘이라면 가능할 것 같습니다. 물론 천재적인 사람들은 이런것을 직관적으로 혹은 새로운 아이디어로 해결하겠죠
다들 어떻게 생각하시나요
인간의 직관 혹은 창의성 vs 로봇의 엄청난 양의 데이터 처리로인한 창의성
그래서 저는 뛰어난 일부의 인간과 대부분의 로봇이 미래를 주도할 것 같습니다
Reply by 김윤수
https://www.seoul.co.kr/news/newsView.php?id=20190322500044
벌써 실용화 되있는 기술이었네요.
이미 단편소설까지 제출해서 공모전 예심을 통과한 수준의 글이라면,
미래에 정말로 AI가 쓰는 글을 읽는 시대가 올 수 있을 것 같습니다.
Reply by 최민수
장편 소설같은 경우에 긴 플롯이나 복선, 설정 등을 다 고려하고 쓰는 경우가 많은데 로봇이 그걸 대체하는 것이 과연 가능할까요.
시의 경우에는 일단 시적인 문체를 쓰는 것만으로 거의 다 해결이 되니 크게 상관이 없지만.
물론 머신러닝을 어떻게 하느냐에 따라서 그것은 모르겠습니다.
프로그램을 짜는 사람의 능력에 달린것 같기도 하구요
Reply by 강민석
오히려 소설이 시보다 더 쉽지 않을까요.
소설 뺨 치게 긴 시도 많습니다. 중요한 건 분량이 아닙니다.
그럼 작품의 '생산'에 기계가 개입할 만한 중요한 요점이 무엇일까. 저는 '형식'이라고 봅니다.
시는 그 출발은 정형시로써 형식이 있었지만 지금 와서는 거의 형식이란 단어를 초월한 상태입니다.
심지어는 그냥 아무 것도 쓰지 않은 백지 한 장조차도 시가 됩니다.
중요한 건 거기에 어떤 의미를 담았느냐인데, 이게 참 추상적인 말입니다.
이런 추상성을 기계가 감당할 수 있을까. 전 회의적입니다.
반면 소설은 그나마 형식이란 걸 잡기 쉽습니다.
기본은 기승전결, 혹은 발단/전개/위기/절정/결말 식이고,
순문학을 전문적으로 다루는 게 아니고서야, 오히려 이런 형식을 벗어난 글은 너무 낯설게 받아들여져서 수요가 없습니다.
즉 어느 정도는 형식이 정해진 게 소설입니다.
그렇다면 형식에 맞춰서 적절한 문장을 서사에 맞게 끼워넣기만 한다면 소설이 될 것입니다.
기계가 이 정도로 치밀해질 수 있을지는 의문이지만, 저는 적어도 시보다는 가능성이 높다고 생각합니다.
Finalize by 강민석
문학을 데이터화하고 플롯을 정형화하여 로봇이 소설을 창작하는 것은 가능하다.
당장 공모전 예심에 당선된 사례가 있다.
하지만 경험적 프로세스에 의한 산물로서 어느 정도 뻔한 작품이 나오는 것은 감안할 수밖에 없다.
완전히 새로운 것을 창작하는 것은 불가능할 것으로 보인다.
추상성을 중시하는 시 같은 경우는 로봇이 창작하는 것이 힘들 수 있겠으나,
적어도 소설에 한해서라면 로봇이 충분히 양산 가능할 수 있다.
5. 한국어 수학 교재의 필요성
Review by 조영은
문학과 수학
-니콜라스 부르바키: 프랑스의 10여명의 수학자 집단
프랑스에서 어려운 독일 원서를 읽기 어려웠기 때문에 이해하기 쉬운 교재를 집필해서 학생들이 편하게 공부하는데 큰 기여를 했다. 참고로 교재의 뒷부분에는 현대 수학까지 포함되어 미제문제에 대한 학생들의 호기심을 자극하기도 한다.
이를 통해 프랑스 수학이 성장하였고, 이 때문에 프랑스의 유명 수학자 중에는 스승이 없는 경우도 꽤 있다.
-문문법: 문장의 특징을 찾아 유사성 발견하는 것.
토마스 메리엄은 문문법을 활용하여 작가불명의 작품 몇 개가 셰익스피어의 작품임을 밝혔다.
-소설<이상한 나라의 앨리스>의 작가(Lewis Carroll, 본명: Charles Lutwidge Dodgson)는 실제로 옥스퍼드 수학과 교수(기호논리학)이다. 이를 통해서도 수학과 문학이 관련이 있음을 알 수 있다.
Reply by 최민수
제가 기억하기론 서울대에서 쓰이는 수학교재중 선형대수와 군, 미적분학(김홍종 지음), 해석개론 등은 한글로 쓰였지만 다양하고 깊이 있는 내용을 다루고 수준도 높다고 알고 있습니다. 우리 성균관대도 저런 교재를 사용하거나 한글 교재를 출판한다면 훨씬 질좋은 강의를 들을 수 도 있을 것 같네요
Reply by 강민석
수준이 높다거나, 다루는 주제가 다양하다거나, 최신 수학까지 다룬다거나 하는 이전에,
일단 일정 교육 수준을 받은 사람이라면 누구든 혼자서도 읽을 수 있을 만큼 한국어로써 자연스럽고 쉬워야 한다고 생각합니다.
수학의 발전에 있어서 가장 필요한 부분은 수학에 관심을 가지는 사람이 많아지도록 하는 것이라 생각하고,
그를 위해서는 수학이 마냥 어렵다는 편견을 낮추고 접근성을 높여야 할 필요가 있을 것입니다.
어려운 건 접근성을 높인 다음 들이밀어도 됩니다.
우리나라에서 수학이라는 학문은 '마냥 어려운 것'으로 받아들여지기 십상이고,
우리말로 된 교재들 역시 이런 편견을 그다지 버리게 하지 못하고 있습니다.
대부분은 강의용으로 만든 것이라 잘 모르는 사람이 접하기에는 강의 없이는 섣불리 감당하기가 힘들고,
심지어 교양서적조차도 전문용어와 딱딱한 어투로 인해 이게 무슨 교양이냐며 기피당하기 일쑤입니다.
수학도 충분히 '즐길 수 있는' 것임을 가르치는 것이 지금의 한국어 교재의 가장 중요한 점이라고 개인적으로 생각합니다.
이를 만족시킬 만한 교재는 지금껏 몇 가지 접해본 적이 없습니다.
제가 봐본 것들 중 이런 이상에 가장 맞을 만한 책은 경문사에서 출판한 '친절한 수론 길라잡이'(정수론을 다룹니다) 같은 경우가 있겠네요. 아마 교양서적으로 분류될 겁니다.
수학에 대한 언어의 장벽을 허물기 위해서는 수학을 우리말로 할 줄 하는 사람이 많아져야 한다고 생각하기에, 그런 사람을 늘릴 수 있는 방안으로 접근해야 한다고 생각합니다.
Relpy by 이헌수
서울대에서 좋은 교재들을 집필하고있었군요 문제는 모두가 말씁하셨듯이 그것이 여기저기 보급되어 공유되지않고 어떤 집단 안에서만 돌고 있어서 사회에 영향을 끼치지 못하는 것 같습니다. 그런 책들이 널리 퍼졌으면 좋겠네요
새로운 책을 쓰는것과는 또 다른 문제로 저는 번역서들에 대한 문제도 해결되었으면 좋겠습니다
전공공부를 하다보면 원서를 사용하기는 하지만 아무래도 영어이다 보니 읽는 속도 자체가 느리고 한글로 된 책은 공부하는 속도가 훨씬 빠르다보니 (영어를 원어민 처럼 하시는 분 부럽네요) 한글 책으로 공부하고 영어로 된 단어들을 익히는 식으로 했는데 (저 뿐만 아니라 많은 분이 이런식으로 공부하시는 것 같습니다)
한글 번역이 자연스럽게 안된 구절, 이해하기 힘들게 적은 구절들이 많아 오히려 그 문장을 이해하기 위해 영어책을 찾아보게 되는 경우가 많았습니다. 한글 설명이 더 어려운 아이러니가...
다들 어떻게 생각하실지 모르겠지만 저는 번역서를 만들때에도 이런 것들을 좀 더 고려해서 만들어 줬으면 좋겠습니다. 한국인이 쉽게 읽을 수 있게 번역만 잘해줘도 한국도 학문적으로 더 발전할 수 있을것 같습니다. 혹은 번역서에 저자가 자신의 의견을 보태어 만든 책들도 좋을 것 같습니다. 이건 저작권 문제때문에 안되겠지만..
Reply by 조영은
서울대에서 직접 쓴 수학교재가 있다는 건 몰랐는데 덕분에 새로운 사실을 알게됐습니다!
한국에서도 번역 과정을 거쳐 공부하는 것이 아니라 우리말로 바로 이해할 수 있는 교재의 보급이 활성화되어야 한다고 생각합니다.
또한 니콜라스 부르바키의 교재처럼 현대수학의 미제 문제와 미래 수학의 방향성에 대해 서술한다면,
학생들의 지적 호기심을 끌어올려 한국 수학의 발전도 기대할 수 있지 않을까요??
Finalize by 최민수
-니콜라스 부르바키: 프랑스의 10여명의 수학자 집단
프랑스에서 어려운 독일 원서를 읽기 어려웠기 때문에 이해하기 쉬운 교재를 집필해서 학생들이 편하게 공부하는데 큰 기여를 했다. 참고로 교재의 뒷부분에는 현대 수학까지 포함되어 미제문제에 대한 학생들의 호기심을 자극하기도 한다.
이를 통해 프랑스 수학이 성장하였고, 이 때문에 프랑스의 유명 수학자 중에는 스승이 없는 경우도 꽤 있다.
-문문법: 문장의 특징을 찾아 유사성 발견하는 것.
토마스 메리엄은 문문법을 활용하여 작가불명의 작품 몇 개가 셰익스피어의 작품임을 밝혔다.
-소설<이상한 나라의 앨리스>의 작가(Lewis Carroll, 본명: Charles Lutwidge Dodgson)는 실제로 옥스퍼드 수학과 교수(기호논리학)이다. 이를 통해서도 수학과 문학이 관련이 있음을 알 수 있다.
한국에서도 서울대에서는 한국어 교재로 김홍종 저자의 미적분학, 이인석 저자의 선형대수와 군 김성기 김도한 계승혁 저자의 해석개론 등을 이용해서 질높고 언어의 장벽이 없는 수업을 해나가고 있다.
또한 경문사의 친절한 수론 길잡이 라는 책은 쉽게 정수론을 가르쳐 주는 좋은 한국어 교재이다.
또한 번역서는 교재마다 쓰는 단어가 다르고 문맥의 흐름이 이상한 경우가 많아서 용어의 통일과 섬세한 번역이 필요해 보인다.
결론적으로, 언어의 장벽없이 생각을 자유롭게 해나갈 수 있도록 한국어 교재의 출판은 진흥되어야 할 것이다
이 아래로는 Final 처리를 받지 못했던 것들입니다. 제가 쓴 답변만 기재합니다.
4. 원과 점의 개수에 관한 관련질문 (Question by 이헌수)
무한론을 가장 일반적인 언어로 풀어 쓴 예로는 힐베르트의 '무한 호텔'이 있습니다.
간단하게 찾을 수 있으니 찾아보세요.
이에 따르면 무한 + n(자연수) = 무한, 무한 X n(자연수) = 무한이라는 명제를 쉽게 이해할 수 있습니다.
그냥 말장난 같기만 할 수도 있으니 머릿속으로 받아들여지느냐의 차이는 있지만...
슈뢰딩거의 고양이나 디렉 델타 함수도 발표 당시에는 학계의 수많은 사람들에겐 말장난으로 받아들여졌으니 결국 받아들이기 나름입니다.
이를 응용하면 자연수의 개수는 2의 배수의 개수와 같습니다. 1대 1 대응하니까요.
유리수의 개수와도 같습니다. 분모를 p로 고정시키면 분자 q와 1대 1 대응하니까요.
무리수의 개수는 더 많습니다. 칸토어의 대각화 논증으로 증명되는데,
그 대각화 논증에서 보이는 논리는 결국 무리수는 유리수와 1대1 대응이 불가능하다는 증명입니다.
물론 1대 1로 대응시킬 수 없어도 같다고 보는 경우도 있습니다.
반원의 점의 개수는 직선의 점의 개수보다 2개 더 많아 완벽한 1대1 대응이 불가능하지만 같은 무한으로 칩니다.
무한보다 작은 실수 범위의 차이이므로 같게 보는 겁니다.
이제 직선의 길이가 달라도 점의 개수가 같나는 물음에 답하자면,
1대 1 대응하므로 같습니다.
5. 정수론과 암호학 (Question by 이헌수)
정수론의 실생활에서의 사용에 관한 질문 올렸던 질문자입니다.
정수론이 어떻게 암호학에서 사용되는지는 저도 아주 대략적으로만 알고 있습니다만, 일단 아는 내용으로 설명드리겠습니다.
현재 수학의 발전 상태로는 거대한 어떤 수를 주고 그 수가 합성수라고 했을 때 그 수가 도대체 몇 곱하기 몇인가, 그걸 특정지어 단박에 알아낼 수 없는 방법은 없다시피 합니다.
3의 배수인가? 아니고... 그럼 7의 배수인가? 그것도 아니고... 식으로 깜깜이식으로 찾아갈 수밖에 없습니다.
물론 곱해서 거대한 수가 될 정도로 충분히 큰 두 수를 주고 곱해봐라 그런다면, 연산 속도에 따라 단박에 됩니다.
즉 거대한 수를 주고 이건 소수가 아닌데(소수인지 아닌지 가려내는 것도 사실 큰일입니다) 그럼 몇 곱하기 몇인가 하고 묻는다면, 이에 대한 답은 굉장히 구하기 어려운 반면, 일단 답을 구했다면 그게 맞는지 틀린지는 쉽게 검산 가능합니다. 그냥 곱해보면 되니까요.
이는 P대 NP 문제로 이어지지만, 자잘한 거니까 생략합니다.
좋은 암호의 조건은 외부인은 굉장히 풀기 어렵지만(풀기 불가능할수록 좋겠죠) , 암호의 규칙을 알기만 하면 굉장히 풀기 쉬워야 합니다.
좋은 컴퓨터가 있다는 가정 하에 이 소수/합성수 문제는 굉장히 좋은 조건을 충족합니다.
그래서 실제로도 쓰입니다.
여기서 대답인데, 인류의 수학 발전 상태가 현 상태에서 답보가 없다면 암만 컴퓨터 성능이 좋아도 그 성능값이란 게 유한한 이상은 이런 체계를 이용한 암호는 풀기 어려울 것입니다.
정확히 말하면, 풀기 불가능한 암호란 건 존재하지 않는다고 쳐버리고, 그럼 풀리긴 풀리는데 풀 때까지 평생의 한 100배쯤 걸리는 암호를 만들어버리면 된다는 식입니다.
이런 접근방식으로 다가가면 컴퓨터의 성능은 더 이상 암호를 풀 수 있는가의 여부에 대한 변수가 되지 못합니다.
6. 주위환경과 수학자 (Question by 이헌수)
쌍둥이 소수 이론의 증명 가능성에 획기적인 한 획을 그은 수학자 이탕 장은 시간제 강사로 벌어먹고 살던 사람이었습니다. 시간제 강사 자리도 서브웨이(그 샌드위치 말하는 거 맞습니다) 아르바이트 하면서 여기저기 이력서 넣고 아무 지원 못 받고 개인논문 쓰다가 겨우 잡은 자리였습니다. 쌍둥이 소수 이론의 증명 가능성을 높인 업적으로 교수직을 얻고 테렌스 타오 등의 수학자와 수학 커뮤니티에서 협업하고 있다고 압니다.
그레고리 페렐만은 푸앵카레 추측의 증명으로 유명해졌는데, 이 사람은 수학계의 협잡질(중국계 수학자 야우싱퉁이 아주 지독하게 굴었습니다)에 신물이 나서 필즈상이고 상금이고 다 거절하고 때려치운 채 평생의 은사와도 연락을 끊고 잠적해버린 것으로 유명합니다. 필즈상도 상금도 거절하고서도 정부에서 주는 저소득층 지원금 타먹으면서 홀어머니와 살았습니다. 상술했듯이 잠적해버려서 현재 근황은 모르겠습니다.
7. 정수론의 발전방향에 대한 의문 (Question by 김윤수)
사실 정수론 들어가자마자 배우지만 많은 분들이 일단 한 귀로 흘리고 시작하는 부분입니다.
'정수의 모든 성질을 소수의 성질만으로 다 알아낼 수 있다'
이것이 정수론에서 소수에 그렇게 집착하는 이유입니다.
실제 거의 모든 정수론의 연구결과는 이를 뒷받침하곤 합니다.
최대공약수/최소공배수는 일단 소인수분해부터 하고 접근하면 이 이상 쉬울 수가 없고,
2차 잉여 같은 조금 복잡한 것들도 결국 소인수분해해서 따로따로 떼어내어 계산하게 됩니다.
정수론의 연구결과가 다른 학문과 어우러지면서 온갖 정수론이 탄생했지만, 결국 정수론이 소수만 연구하게 되는 이유는 이 이유일 것입니다.
정수의 모든 성질을 소수의 성질만으로 알아낼 수 있는 것 같긴 한데, 그게 재밌어서 소수를 열심히 파봤더니 이게 생각 외로 도저히 풀리지가 않더라, 그래서 소수에 계속 집착하게 되는 것이라고 이해하면 됩니다.
일종의 희망고문 같은 이야기네요.
결국, 소수의 분포와 그 성질이 완전히 규명되지 않는 한은 정수론은 소수를 연구할 수밖에 없을 겁니다.
다른 분야들에 비해 이해는 쉬운 편이라 널리 알려진 것일 테고요.
마냥 소수만 연구하진 않는다지만, 어느 분야에서든 정수론이라면 결국은 무시할 수 없게 되곤 합니다.
8. 위상동형 (Question by 최민수)
위상동형은 정의만 가지고는 쉽게 이해되지 않는 편입니다.
이런 방식의 이해는 어떨까 합니다.
일단 말해두는데, 이 설명 내에서의 모든 f'은 f의 미분 형태가 아니라 그냥 좀 다른 함수입니다.
어떤 두 집합 X와 Y가 위상적으로 같은 모양인지 어떤지 판단하고 싶다고 합시다.
가장 간단한 방법은 X에 있는 모든 점들 간의 성질이 Y에서도 성질이 비슷한지 판단하는 겁니다.
위상적인 성질 말이죠. 뭐 Connected라든지 Compact라든지 Convex라든지 여러 가지 있습니다.
어찌됐건 X에 있는 점으로 Y에 있는 점을 판단해야 합니다. 그렇다면 X에서 Y로 가는 함수 f : X→Y를 정의해서 f를 분석하는 게 좋습니다.
X에 있는 모든 점으로 Y에 있는 모든 점을 판단해야 하므로 f는 전단사함수여야겠네요.
X에 있는 점 x와 Y에 있는 점 y가 y = f(x)라고 합시다. x 근처에 있는 x'를 통해 f(x') = y'을 분석하고 싶다면 아무래도 y'가 y 근처에 있는 게 또 좋습니다. 이를 보장하는 게 연속함수라는 조건입니다.
X와 Y가 위상적으로 같다면 Y와 X 역시 위상적으로 같아야 하므로 역함수에서도 연속이 보장되어야겠고요.
뭐 대강 이런 식으로 정의됩니다. 더욱 더 엄밀하게 이쪽을 파고 싶으시면 아예 이쪽으로 전공을 잡으시는 게.
이런 게 왜 필요한지 싶겠는데, 미지수화와 관련이 좀 있습니다.
복잡한 함수 f(x)가 있을 때 일부분을 g(x) = y 형태로 바꿔서 f(x) = f'(g(x)) = f'(y)로 만들어버립니다.
치환적분 할 때처럼요.
이 조금 더 간단하게 만들어본 f'(y)가 f(x)와 얼마나 비슷한가, 얼마나 같은 성질을 공유하느냐를 논할 때 이런 위상동형 개념을 이용합니다.
이런 걸 이용해서 Parametrization이라든가 치환적분이라든가 뭐 여러 가지 방법으로 쓰입니다.
일단 숙달이 되면, 함수를 분석하는 상당히 강력한 무기가 됩니다.
9. 4색 정리 (Question by 김기헌)
4색 정리 문제는 최초의 컴퓨터를 이용한 증명으로 알려져 있습니다.
그냥 모든 경우의 수를 컴퓨터로 짜서 모든 경우의 수를 돌려봤더니 안 되는 거 없더라 식으로 증명했습니다.
어떻게 증명했느냐고 물어도 그냥 다 해 봤더니 되던데? 라고밖에는 요약이 불가능합니다.
다만 대충 돌렸더니 나왔다, 정도로 요약할 수 있는 건 아니고, 경우의 수를 최소화하고 컴퓨터의 연산을 최소화하는(안 그러면 돌리다가 맛이 갈 테니까) 데에는 충분한 논리가 들어간 것으로 압니다.
에르되시 팔 등의 저명한 수학자들 중 일부는 좀 더 논리적이고 아름다운 증명이 있을 것이라 생각했습니다.
이미 식어버린 떡밥 같겠지만 그렇지도 않습니다.
수학의 왕 가우스는 '어떤 문제에 대한 다른 증명을 발견하는 것은 그 문제에 대한 최초의 증명을 발견하는 것만큼이나 중요하다'고 했습니다. 이를 통해 완전히 새로운 관점에서의 접근이 이루어지고, 이 발상의 영감이 수학 발전에 이바지할 수 있다고 생각했기 때문입니다.
하지만 실용성은 그저 지도를 출판할 때 사용할 잉크 색을 하나 아낀 정도로 알고 있습니다.
위상수학과 그래프 이론에 큰 이바지가 되었다고는 합니다만 전 그 쪽은 어두운 편이라 잘 모르겠네요.
참고로, 가정이 하나 있습니다.
평면공간이나 구면공간에서만 성립하는 정리입니다.
+ 추가질문(컴퓨터를 이용한 증명은 수학계에서 받아들여졌나요?) by 이헌수
일단 경우의 수를 하나하나 나누고 그걸 모조리 적용시켜본 거니까,
겹치는 데 없이 모두 4색으로 칠해졌는지 검토만 잘 해보면 됐을 겁니다.
왜 이런 식으로 모든 경우의 수를 표현할 수 있는가, 이렇게 따진다면 이건 논리에 의한 게 맞기도 하고요.
10. 프로그램을 통한 표절 색출 가능 여부 (Question by 김윤수)
작자 미상의 작품이 작가가 누구더라, 혹은 이 작품은 이 작가 작품이라 알려졌는데 실은 아니더라,
이런 경우는 설령 기계와 프로그램을 동원하더라도 논란이 있을 수 있을 겁니다.
예술가라면 매 순간 변화를 시도하는 것이 당연하고 작풍이 변화하여 이 작품이 정녕 전에 이런 작품을 썼던 사람이 쓴 게 맞나 싶은 경우도 꽤 있을 텐데, 이런 경우를 기계가 판별해냈다고 의심이 다 가시진 않을 것입니다.
하지만 표절은 이야기가 조금 다릅니다. 기계적으로 판단해도 크게 유사할 정도면 사람이 보면 얼마나 똑같겠냐 식의 논리가 됩니다만,
예를 들어 논문의 표절 같은 경우는 기본적으로 원칙이 두 가지, '여섯 단어 이상의 연쇄적인 표현의 일치'와 '출처 미표기'입니다.
충분히 기계적으로 찾아낼 수 있는 범주 안에 드는 가이드라인입니다.
결국은, 기준의 문제입니다.
이런 생각으로 접근할 수 있겠습니다.
사람이 언제나 같은 글자를 같은 글씨로 쓸 수가 없는데 필적 감정에 의미가 있겠느냐 같은 의문과 본질적으로 다르지 않습니다.
사람은 결국 경험에 의해 지성을 확립하기 때문에, 지성을 표현하는 '글을 쓴다'는 행위를 할 때는 그 사람만의 경험적인 부분이 들어갈 수밖에 없습니다. 이는 비단 필체만이 아니고, 단어 선택이나 자주 틀리는 띄어쓰기 등의 오류 등등 많은 곳에서 힌트가 될 요소가 있습니다.
이런 것들을 중점적으로 잡아내면 합리적으로 이 글은 누구의 글이다, 라고 말할 만한 근거가 되는 것이겠죠.
11. 천경자 화백 사건에서의 진위 논란의 양측 근거 (Question by 이상구)
이 법정공방이 살펴보면 꽤 재밌습니다.
일단 내 그림이 아니라고 했던 천경자 화백의 입장은 제외합니다.
미인도가 위작이라고 주장했던 다른 곳이 있습니다.
프랑스의 르미에르 테크놀로지 연구 팀입니다.
작품 검증 방법은 사진 이미지 분석을 통한 수학적 수식 산출 방법입니다.
특수 카메라로 진품을 스캔하고 각 사진 이미지를 수치화하는 방법입니다.
추상적으로 들릴 수 있겠습니다만, 즉, 화가의 '버릇'을 수학적으로 계산하고자 하는 시도입니다.
명암대조를 보통 이 화가는 어느 정도 하는데 이 그림은 어느 정도더라, 이걸 평균과 표준편차를 만들어 대입한다거나,
색채와 색조의 선택 문제, 붓질 문제, 등등 그림을 구성하는 곳곳을 모조리 수치화하는 겁니다.
문제는 정확도였습니다.
당시 미인도의 진품 가능성은 0.00002%. 이것만 보면 확실히 위작이다 싶으시겠지만,
문제는 이 시스템을 그대로 도입해서 다른 화가를 골라 진품임이 입증된 그 화가의 작품을 적용해봤더니 진품 가능성이 4% 미만으로 뜨거나 했던 점입니다.
0.00002%에 비해서야 높은 수치지만 이 시스템 자체의 신뢰도가 뚝 떨어지는 것은 어쩔 수 없을 것입니다.
반면 미인도가 진품이라고 주장했던 검찰 측 증거는 다분히 과학적인 접근입니다.
천경자 화백 특유의 작품 제작 방법 그대로였고, 육안으로 관찰되지 않는 안입선(날카로운 필기구 등으로 사물 외곽을 그린 선)이 다른 진품들과 같다,
즉 천경자 화백이 그린 그대로 그려진 그림이다, 이렇게 판정했던 것입니다.
심지어 이 주장은 '내가 그린 위작이다'라고 자백한 권춘식 화가의 등장에도 굽혀지지 않았습니다.
물론 이 주장도 근거가 다분히 부족하기는 합니다.
하여 법정공방까지 갔던 거고요.
지금의 위작을 가려내는 수학적 시도는 이 시절보다 현격히 발전한 상태이지만,
지금의 기술로 미인도를 판단했을 때 미인도는 진품일지 위작일지, 흥미롭지 않을 수 없습니다.
12. 전문용어의 번역 문제 (Question by 최민수)
보통은 그냥 되는대로 온갖 단어들을 있는 대로 갖다 쓰는 경우가 대부분입니다.
전 생명과학 번역 책을 두 개 갖고 있는데,
한 책에서는 전두엽이라고 부르는 대뇌의 일정 부위를 다른 책에서는 이마엽이라고 번역해놓았습니다.
이런 용어들을 제대로 번역하려면 다른 나라는 모르겠어도, 우리나라는 일단 협회를 세웁니다.
대한수학회 같은 곳에서 주관해서 용어를 통일시킨다거나 하는 그런 식입니다.
하지만 적어도 제가 아는 한에선 이런 시도는 없네요. 혹시 있을지도 모르지만 전 모릅니다.
사물의 모든 것은 이름에서 시작한다는 말이 있듯이 참 중요한 문제인데,
영어로 수학을 하는 게 하도 당연시되다 보니 잘 신경쓰지 못하는 부분이라고 봅니다.
수학 번역서도 몇 개 갖고 있는데, 용어 선택 문제 이전에 번역투가 심해서 어려운 공부 하는 사람들이 한글도 모르나 하고 투덜댔던 기억이 있어서 심정을 이해합니다.
13. 타르탈리아의 3차 방정식 풀이에 대한 공방 (Question by 이헌수)
당시의 수학에 대한 분위기를 알아야 합니다.
수학이 주로 사용되는 분야가 돈계산이던 시절이라,
얼마나 쉽고 정확하게 계산을 하느냐가 중요했습니다.
이것이 곧 '스펙'이던 시절이고,
실제로 누가 얼마나 정확하고 빠르냐를 두고 수학 대결을 하기도 했습니다.
서로의 목숨이 걸려있다는 점에선 칼 들고 하는 결투와 별다를 바 없기도 했습니다.
타르탈리아는 찢어지게 가난했다가 셈계산 하나로 신세를 고친 인물이었던 걸로 압니다.
과연 자기만의 비술을 쉽게 풀 수 있었을까요?
오히려 이걸 꼬드겨서 통째로 훔쳐간 카르다노가 나쁜 놈인 건 아니었을까요?
이런 식의 분쟁은 뉴턴과 라이프니츠 간의 미적분 발견자가 누구냐는 논란과도 흡사합니다.
뉴턴은 일찍이 미적분의 기본원리를 스스로 깨쳤지만 자기만의 비기로 놔두고 발표를 하지 않았었습니다.
혹은 그냥 대수롭지 않게 생각해서 굳이 발표까지 할 것 있나 상태로 놔뒀다는 이야기도 있습니다.
실제로 뉴턴은 그런 식의 듬성듬성 일처리가 제법 있었던 인물이라고도 합니다.
그러다 라이프니츠가 미적분을 발표하자 난리가 난 겁니다.
앞서 발표한 뉴턴의 논문들 중엔 미적분을 알지 못하고는 있을 수 없는 논문들도 있었으니까요.
영국과 유럽 대륙 간의 자존심 싸움으로까지 번졌던 어마어마한 논란이었습니다.
결국 따로 연구하여 동시에 발표로 서로 원만히 합의를 본 것이었고요.
학자들의 격언에는 이런 말이 있습니다.
'발견했다면, 발표해야 한다.'라는 격언입니다.
나중에라도 이 원칙이 세워졌기에 학계라는 것이 존재할 수 있었던 것입니다.
이런 지식들을 원만히 공유하지 못했던 우리나라는 맥이 끊긴 것들이 수없이 많은 것이고요. 고려청자 제법이라든가 등등.
+
타르탈리아는 별명입니다. 말더듬이라는 뜻입니다.
본명은 니콜라 폰타나입니다.
14. 컴퓨터 과목의 중요성 대두 (Question by 김기헌)
중학교 시절에 컴퓨터 관련 과목이 있었던 경험이 있는 분들이 있을 겁니다.
당장 제가 그랬습니다.
1주일이나 2주일에 한 번씩 컴퓨터실에 한 시간 정도 박혀 있었던 기억이 납니다.
당시엔 그냥 워드프로세서나 한글, 타자연습이나 하고 맨 뒷자리 애들은 몰래 게임 깔아서 하던 그런 수업이었습니다.
자연히 중요성은 떨어지고, 왜 하는지도 모를 그런 수업이 되곤 했었습니다.
컴퓨터 없는 학문의 발전을 오히려 상상하기 힘든 지금 시대인 만큼 이런 과목의 대대적인 개혁이 필요하다 봅니다.
워드프로세서나 Microsoft 관련 프로그램이나 다루다 마는 수업이 아니라, 코딩을 가르치는 수업으로 충분히 변모할 수 있다고 봅니다.
코딩 교육을 통한 미래 인재의 육성 외에도, 국영수사과에 쓸데없다 싶을 만큼 너무 목매는 지금의 교육 체계에서 중요성이 큰 다른 과목이 대두되어 경직된 주입식 교육 환경에 새로운 충격을 안겨줄 수 있다는 점에도 주목할 가치가 있다고 보입니다.
15. 수학 연표에 동아시아 수학을 끼워넣을 필요가 있는가 (Question by 이상구)
차라리 동아시아의 수학 연표 항목을 따로 개설하는 게 나을 겁니다.
이러니저러니 해도, 결국 동아시아의 수학이 현대수학의 기틀이 된 서양 수학과 별다른 접점 없이 끝나버린 건 사실이니까요.
현대수학의 모태를 알아보기 위한 목적으로 작성되었을 세계 수학 연표에 들어가기엔 좀 힘들 것 같습니다.
굳이 넣는다면 중국인의 나머지 정리에 관한 내용 정도밖에는 생각이 안 납니다.
+
그 일단 모아두자는 미봉책이 개별적인 연표 작성입니다.
서기 기준으로 작성만 해두면 합치기도 쉬울 것이고,
서양 수학과 동아시아 수학이 따로 발전한 것만큼은 사실인 만큼,
합쳐놓으면 서로 섞여서 헷갈릴 만한 것들이 따로 보면 각각의 발전 과정을 알 수 있어 더 나을 거라 봅니다.
아주 없었던 것으로 하자는 게 아니라 일단은 별개로 봐두자는 것입니다.
불분명한 요소가 너무 많아서(작자미상이나 연도미상이 너무 많은고로) 어디다 끼워넣기도 참 애매한 구석이 많다보니 명료하게 정리하기도 힘들고요.
16. 동아시아 수학에서의 조선 수학을 내세울 수 있을까 (Question by 이상구)
앞으로도 없다면 영원히 없지 않을까요.
조선 수학이 위대했다고 한국 사람이 주장하면 그걸 누가 알아줄까요,
자기 나라 감싸고 도는 것으로밖에 안 보이지...
하다못해 누구나 인정할 수 있는 역사적 사료라도 있으면 혹 모르는데,
만약 있더라도 중국과 일본에선 절대로 인정하지 않을 겁니다.
실제로도 있는데 인정하지 않는 경우를 수없이 접합니다. 독도는 뭐 근거가 있어서 누구 땅이냐 싸우나요...
쓸데없이 나라의 자존심 같은 거나 내세우니까 동아시아 사학이 엉망이라는 투의 역사 교양서를 본 적이 있습니다만,
저 역시 공감하는 내용입니다.
동북공정이 다름아닌 이런 논리로 진행된 프로젝트이고,
역사 왜곡 오염은 한중일 가릴 것 없이 어느 나라나 심한 편이니까요.
역사학에 대한 학문 윤리의 정립부터가 시급하다고 생각합니다.
내가 원조다 하는 투의 주장도 자제할 필요가 있다고 봅니다.
세계 최초에 제일 환장한 나라다 투의 빈정거리는 여론도 수없이 접해본 바 있는 입장으로선 해야 할 말이라고 생각합니다...
그리고 무엇보다도, 조선 시대 산학이 발전하였고 여러 산학책이 있다고 자랑할 것이 아니라,
그 산학책을 처음부터 끝까지 온전히 번역해내는 것이 더 의미 있는 시도라고 봅니다.
문제 몇 가지는 번역된 바 있는 걸로 아는데, 책 자체가 온전히 번역되었는지 여부는 전 모릅니다.
혹 그런 번역서가 있나요?
17. 주입식 교육과 창의성을 강조하는 교육의 방향성 (Question by 이헌수)
간단히 말하자면 주입식 교육은 범재를 양산하기 위한 교육.
창의성을 중시하는 교육은 특별한 소수를 더 위대하게 키워내기 위한 교육.
전 이렇게 봅니다.
어느 것이 더 낫다고 하긴 어렵다 봅니다.
주입식 교육으로 인해 망가져버린 천재들은 셀 수 없이 많지만,
기본도 없는 창의성은 오히려 일반적으로 독이 됩니다.
학문을 공부하는 대학이라면 창의성을 강조하는 교육이 틀리지 않다고 생각하지만 그건 일반론입니다.
대한민국은 사정이 좀 특수한 나라인지라...
요즘 교육의 트렌드는 창의성과 리더십이라고 하지만 솔직히 헛소리고,
주입식 교육도 버거운 사람들한테 창의성 강조해봐야 뭘 할 것이며,
누구나가 리더십에 목매기 시작하면 배가 산으로 가기 십상이니 굳이 모두가 이런 교육을 받을 이유는 없을 것입니다.
이런 차별화된 교육을 시스템화 하기 위해 특성화고니 특수학과니 하는 게 있는 것일 텐데,
이마저도 지금은 주입식 교육에 특화되어 시험점수만 잘 뽑는 학생들이 바글거리니 이미 의미가 없습니다.
대대적인 교육개혁이 필요한 이유일 것입니다.
+ Question by 이헌수
고등학교까지 주입식교육 대학교초반에도 지식의 습득을위한 주입식과 사회로 나갈때엔 다른 교육방식 좋은 것 같습니다.
그런데 그것은 고등학교 후 대학을다니고 그 후 사회로 나가는사람에게만 해당되는 것 같습니다.
고등학교 이후 바로 사회로 가는 학생들은 다르게 교육해야한다 생각합니다. 한 학교내에서는 어려우니 애초에 고등학교를 목적에 맞게 따로 설립하면 좋을 것 같습니다.
저희학교 학생들은 그래도 그나마 전공과 관련있는 일을 하고 있는 것 같은데, 사실 대부분의 대학졸업생들은 전공과무관한 일을 하고 대학 졸업 후 따로 직업학교를 다니고있습니다. 주입식 교육이 다른것을 창조하기위해선 당연히 기존의 것들을 알아야한다 라는 생각으로 한다면 이 학생들은 어떻게 하는것이 좋을까요...? 여러분의 의견이 궁금합니다..
+ Reply by 강민석
대학의 의도 자체가 왜곡된 문제라고 봅니다.
대학 초입까지 주입식 교육을 고수할 것이 아니라,
아예 창의적인 교육에 적응하지 못한다면 대학을 오지 말아야 한다고 봅니다.
대한민국의 대학 진학률이 지나치게 높은 것이지,
전 지금의 1/4로 대학 진학률이 낮아져도 괜찮다, 오히려 낮아져야 한다는 입장입니다.
대학 진학률의 과다로 인해 사회로의 진출 연령이 과하게 늦어지고,
이것이 노동력의 손실에 직결된다고 보기 때문입니다.
대학은 인재를 길러내기 위해 운영되는 곳이지,
누구나가 가기 위해서, 혹은 등록금 벌어들이려 운영되는 곳이 아니라는 게 제 생각입니다.
대한민국에 현존하는 직업들 중 절대 다수는 대학 수준의 학력까지 요구될 이유가 없다고 봅니다.
다만 그 놈의 스펙이란 게 문제일 뿐이라고 보니다.
학문을 하기 위해, 혹은 더 배워서 뭔가 이뤄내고자 하는 게 있기 때문에,
이런 목표를 가지고 창의성을 발휘하고자 하는 사람이 대학을 가는 게 대학의 원래 취지에 맞다고 봅니다.
이런 목표를 키워주는 곳이 고등학교고요.
현실은 고등학교는 수능공부하러 거치는 곳이고 대학은 수능 점수 맞춰 원서 내고 어영부영 졸업해버리고,
정작 일자리는 자기 학과랑 전혀 관계없는 곳에 넣는 경우가 상당수인데,
특히 문과가 이런 경향이 두드러집니다.
심각한 국가적 낭비라고 봅니다.
정수론의 발전
Ask by 김윤수, Date: 2019.03.27
Finalized by 최민수, Date: 2019.04.01
Final OK by SGLee
<질문> - 김윤수
정수론은 접근성이 좋아서 오래 전부터 연구되던 학문이었는데,
그 쉬운 접근성 때문에 이미 너무 많은 것이 밝혀져
더 이상 발전하기 힘든 상태에 와 있다는 생각이 드는데, 어떻게 생각하시는지
소수에 관련된 문제를 제외하고는 어떠한 연구가 진행되고있는지 궁금합니다.
<답변> 강민석
사실 정수론 들어가자마자 배우지만 많은 분들이 일단 한귀로 흘리고 시작하는 부분입니다.
'정수의 모든 성질을 소수의 성질만으로 다 알아낼 수 있다'
이것이 정수론에서 소수에 그렇게 집착하는 이유입니다.
실제 거의 모든 정수론의 연구결과는 이를 뒷받침하곤 합니다.
최대공약수/최소공배수는 일단 소인수분해부터 하고 접근하면 이 이상 쉬울 수가 없고,
2차잉여 같은 조금 복잡한 것들도 결국 소인수분해해서 따로따로 떼내어 계산하게 됩니다.
정수론의 연구결과가 다른 학문과 어우러지면서 온갖 정수론이 탄생했지만, 결국 정수론이 소수만 연구하게 되는 이유는 이 이유일 것입니다.
정수의 모든 성질을 소수의 성질만으로 알아낼 수 있는 것 같긴 한데, 그게 재밌어서 소수를 열심히 파봤더니 이게 생각 외로 도저히 풀리지가 않더라, 그래서 소수에 계속 집착하게 되는 것이라고 이해하면 됩니다.
일종의 희망고문 같은 이야기네요.
결국, 소수의 분포와 그 성질이 완전히 규명되지 않는 한은 정수론은 소수를 연구할 수밖에 없을 겁니다.
다른 분야들에 비해 이해는 쉬운 편이라 널리 알려진 것일 테고요.
마냥 소수만 연구하진 않는다지만, 어느 분야에서든 정수론이라면 결국은 무시할 수 없게 되곤 합니다.
<답변> - 최민수
물론 우리가 기초 정수론 교재에서 배우는 것처럼 폭발적인 발전은 힘들겠지만 연구는 꾸준히 이루어 지고 있습니다
https://ko.wikipedia.org/wiki/정수론#현대_수론
<답변> - 이상구
지금은 전통적인 정수론에서 더 나아가 대수적 정수론, 해석적 정수론, 기하적 정수론, 확률론적 정수론(Probabilistic number theory), computational number theory 등
수학의 다른 분야와 마찬가지로 수학의 다른 분야의 지식을 이용하여 새로운 분야들이 지속적으로 발전하고 있지 않나 생각합니다.
<정리> - 최민수
정수론 뿐만 아니라 요즘 디지털 시대의 대부분의 과학은 발전할 만큼 발전해있고, 박사나 석사등의 석학들은 그 커져있는 테두리에 티눈을 추가하는 식으로 서서히 발전해나가고 있다고 생각합니다.
matt might 라는 유타대 교수가 각화해서 보여주었습니다
http://matt.might.net/articles/phd-school-in-pictures/
또한 정수론에서 소수를 지독하게 파는 이유는 정수론의 많은 기반들이 소수의 성질과 관련이 많고 소수를 이해하면 정말 많은 부분을 이해할 수 있게 되기 때문입니다.
더 나아가서 사실 고급 내용의 정수론에서는 해석적 정수론, 대수적 정수론, 계산 정수론, 기하학적 정수론, 확률론적 정수론 등 우리가 정수론 하면 일반적으로 생각하는 소수와 모듈로 연산 등과는 다르고 수준이 높은 것들이 많습니다
보험료 인상 인하
Ask by 김윤수, Date: 2019.04.10
Finalized by 최민수, Date: 2019.04.13
Final OK by SGLee
<질문> - 김윤수
자동차 보험 같은경우
사고이후 보혐료가 인상된다던가,
무사고 10년이후 보험료가 인하된다던가 하는 조항이 있는데,
이러한 조항들이 초기 화재보험법에도 적용이 되어있었을까?
하는 의문이 듭니다.
<답변> - 이상구
제일 처음에는 없다가 기간이 지나면서 무사고 운전자에 대한 보험료 디스카운트로 반영되기 시작하였겠지요^^
<답변> 최민수
초기를 어느정도의 기간으로 설정하느냐에 따라 다르겠지만
보험료 산출의 기반은 통계이고
10년 동안 무사고 같은 경우에는 앞으로도 사고를 일으킬 가능성이 모든 사람들의 경우의 가능성보다 낮고
이러한 통계가 쌓이려면 시간이 필요했을 것이므로
초기의 화재보험법에는 없었을 것 같습니다.
사실 보험이 맨처음 생겼을때는 경쟁자가 적으므로 가격을 낮출 필요가 없지만, 경쟁자가 생긴 뒤에는 추가로 고객을 유치하기 위해서 보험료를 인하해 주던 것이기 때문에 초기에는 낮출 필요성도 근거 자료도 부족했다고 생각됩니다
<답변> - 김윤수
교재에 간단한 화재보험 계산법을 설명할 때,
'실제는 집마다 위치나 가치, 건축자재의 종류 등 복잡한 조건이 더해져서 같은 도시 내에도 집 마다 보험료에는 차이가 있게 된다. ' 라는 구절이 있어서
건축자재의 종류에 따라 보험비가 바뀌는 것과 10년 무사고의 경우와 비슷하다고 느껴서 의문이 생긴 것이었는데,
건축자재에 대한 화재확률을 계산한 것이나,
10년 무사고 운전자의 경우 사고 확률이 낮다는 것이나
비슷한 맥락 아닐까요?
<답변> - 이상구
그보다 예나 지금이나 사고가 났을 때 물어 줄 금액과 사고 확률로 보험료를 산ㅊㄹ하는 것이 맞을 것입니다.
<답변> - 김윤수
런던 대화재 이후 보험이 처음 생겨난 줄 알았는데,
확률론의 발전의 시작이라고 할 수 있는 대항해시대에
실패할 확률을 계산하여 기댓값을 통해 투자자가 항해에 투자를 하였다고만 생각했는데,
실패에 대한 대비인 보험또한 함께 생겨났다고 합니다.
이른바 '해상보험' 으로
https://www.lifentalk.com/570
<답변> - 이헌수
https://www.lifentalk.com/860
대항해 시대에는 배, 런던은 대화제였다면
우리나라는 농경사회 답게 소 !
각 나라들의 최초의 보험이 궁금해지네요 특이한 보험들이 많이 있을 것 같습니다
<답변> - 이상구
http://www.wikitree.co.kr/main/news_view.php?id=204754
계나 두레 와 같은 부조를 넘어서서, 생명표와 수학을 이용하는 수학 학문으로의 보험학 의 시작을 의미하는 것입니다.
<답변> - 김윤수
보험은 있었으나
확률이나 통계를 적용한 보험이 아니라서
학문으로서의 보험의 시작을 말씀하셨던 것이군요.
확실히 알겠습니다. ㅎ
<정리> - 최민수
보험료 인하는 처음에는 없다가 자료가 쌓이면서 인하를 해주기 시작했다/
또한 화재보험은 집의 재료가 어떤 종류이냐 건축법이 무엇이냐에 따라서 보험료를 책정하는 시스템이 옛날에도 있었다.
또한 계나 두레같이 위험분산은 고대에도 있었지만 학문으로의 시작은 영국 대화재때 부터 시작했다고 보는 것이 맞다
한붓 그리기 증명
Ask by 최민수, Date: 2019.04.10
Finalized by 최민수, Date: 2019.04.13
Final OK by SGLee
<질문> - 최민수
연결된 변이 홀수인 점의 갯수가 0개일때, 2개일때, 4개 이상일때에 따라서 한붓그리기 가능 유무와 형태가 달라지는데
그에 관한 수학적 증명은 어떻게 하는 걸까요?
<답변> - 김윤수
https://m.blog.naver.com/PostView.nhn?blogId=holybody017&logNo=221171593105&proxyReferer=https%3A%2F%2Fwww.google.com%2F
정확히 일치하는 글을 네이버 블로그에서 찾을 수 있었는데,
주 개념은 수학적 귀납법이라고 합니다.
<답변> - 이상구
일단은 각 경우에 대한 수학적 증명을 가능한 다양한 방법을 사용하여 주고
그런 증명 들에서 일반성을 찾으면서
일반이론을 만들어 갑니다..
<정리> - 최민수
각 케이스마다 증명을 찾아가고 그것의 일반성을 유추해가면서 증명을 하는 경우가 많다
한붓그리기의 경우에는 수학적 귀납법을 사용해서 증명을 한다.
k개의 점을 가진 그래프를 가정한 뒤에 점을 하나 추가해서 그 경우에 대해서 증명을 하는 것이다
최적화의 해를 구하는 과정
Ask by 김윤수, Date: 2019.04.29
Finalized by 이상구, Date: 2019.04.29
Final OK by SGLee
<질문> - 김윤수
수백개의 연립방정식의 해를 통해 최적화된 해를 구하여
최대한의 이익을 보는 일은 사실 사업이나 전쟁에만 국한된 내용은 아니다.
사실 우리일상에서도 그러한 경험들을 자주 겪을 수 있는데,
'오늘 밥을 무엇을 먹을까?' 라는 간단한 결정사항에도
가격, 이동거리, 맛, 시간 등 여러가지 결정요인들이 있을 수 있다.
이런 상황에서 결정하는데 중요한 역할을 하는 것은
그 사람의 가치관이다.
예를 들어 맛집을 좋아하는 사람이라면 이동거리나 가격은 크게 고려치 않는 사람들이 있다.
모든 연립부등식을 만족하는 해가 없을 때, 최 우선 고려 대상을 계산으로 어떻게 적용시킬 수 있을까?
<답변> - 최민수
QFD란 Quality Fuction Deployment의 준말로 신제품 개념정립, 설계, 부품계획, 공정계획 그리고 생산계획과 판매까지 모든 단계를 통해 고객의 요구가 최종 제품과 서비스에 충실히 반영되도록 하여 고객의 만족도를 극대화하는데 초점을 맞추고 있는 품질경영의 방법론 중 하나
아니면 가장 우선순위가 낮은 것을 제거하는 방법 또한 존재하고
수학적으로는 Projection을 통해서 반영시키는 것이 일반적입니다
<답변> - 김기현
질문을 읽으면서 최적화문제는
몇 년 전에 들었던 창의적 공학 설계 수업의 QFD와 상당히 비슷하다는 생각이 들었습니다.
QFD란 Quality Fuction Deployment의 준말로 신제품 개념정립, 설계, 부품계획, 공정계획 그리고 생산계획과 판매까지 모든 단계를 통해 고객의 요구가 최종 제품과 서비스에 충실히 반영되도록 하여 고객의 만족도를 극대화하는데 초점을 맞추고 있는 품질경영의 방법론 중 하나입니다. 이를 이용하면 최적화 문제에 도움이 될 것이라 생각합니다.
<답변> - 최민수
제가 선형대수나 통계학 시간때 배운 최적해라는 개념이 있긴 합니다.
여러개로 흩어져 있는 데이터들을 가장 잘 설명할수 있는 그래프(=그래프로부터 떨어진 점들의 거리의 합이 최소가 되게 하는) 였었는데
가중치를 줘야하는 경우에는 과연 어떻게 될지 저도 궁금합니다
<답변> - 조영은
최적화의 해가 존재하지 않을 경우, 최우선고려대상을 계산으로 결정하는 방법에 대해 질문하셨는데
우선 순위가 가장 낮은 조건을 제외하면서 해를 구하는 방식을 이용하는 것 또한 하나의 방법이 될 것 같습니다.
만약 사업에 관련된 최적화의 해를 구하는 경우라면,
수백 가지의 조건 중에 사업적 이득에 영향을 덜 미치는 순서대로 조건들을 나열하여
하나씩 제거하면서 최적화의 해를 구하는 것처럼 말이죠!
<정리> - 이상구
이 때 찾는 답이 Projection 입니다^^
모든 사람이 요구하는 방향성은 유지하면서 ... best poosible 답을 제시하는 Projection
법과 의료기관에서의 빅데이터 활용
Ask by 김윤수, Date: 2019.05.15
Finalized by 김윤수, Date: 2019.05.22
Final OK by SGLee
<질문> - 김윤수
법의 판결은 수학에서 처럼 모든 상황을 법으로 표명하기 어렵다는 것은 이해?습니다. 근데 결국 판례를 기반으로 판결을 내린다면 이는 데이터화하여 프로그래밍을 이용하여 판결할 수 있지 않을까? 결국 판사는 몇몇 특이케이스에 대해서나 헌법재판이 아니라면 쓸모없는 직업이 되지 않을까 라는 생각이 들었습니다. 다른 분들은 어떻게 생각하시는지 궁금합니다.
<답변> - 조영은
저도 이에 대해 동의합니다. 간단한 사건들은 데이터베이스를 충분히 구축하면 판사, 변호사가 없어도 재판이 진행될 수 있을거 같습니다. 비슷한 예시로 병원에서 환자에 대한 진찰 또한 수술방법의 결정도 구축해놓은 데이터베이스를 이용할 수 있지 않을까요? 이에 대해서는 어떻게 생각하시나요?
<답변> - 이헌수
만약 데이터를 이용해서 프로그래밍을 한다해도 결국은 인간이 필요할 것 같다는 생각이 듭니다.
100퍼센트 프로그래밍만으로 판결이 이루어진다면 그결과를 예상할 수 있게됩니다.
본인이 어떤 행동을 하면 어떤 판결이 나올것이다를 알고 계획적으로 범죄를 저지를 수도 있고, 또 법의 헛점을 이용하는 것처럼 프로그래밍의 헛점을 이용할 수 있을 것 같습니다.
요즘 스포츠에서 VAR 의 도움을 받듯 프로그래밍의 도움을 받는 것은 좋아 보이네요
<답변> - 최민수
수술도 데이터만 쌓이면 상관 없을것 같긴 합니다. 사실 의사들이 수술을 해나가는 과정도 이론과 지금까지의 수술등을 고려해서 수술을 해나가는 것인데 충분한 추가 데이터만 쌓이면 기계가 의사보다 섬세하게 수술을 할 수 있다고 생각됩니다.;
하나 걸리는 것은 만약 수술이 잘못되었을때의 책임문제입니다.
물론 요즘 수술에서도 각서나 동의서등을 제공받긴 하지만 의료사고가 일어났을때 수술을 시행한 병원이 책임을 질 것인지 기계의 제작사가 질 것인지 동의서를 제출한 환자가 질 것인지 등이 꽤나 쟁점일것 같습니다
<정리> - 김윤수
https://www.mk.co.kr/news/society/view/2018/02/137456/
"미국의 대형 법무법인 베이커앤드호스테틀러는 2016년 `인공지능(AI) 변호사`를 채용했다. 대륙아주는 현재 변호사 업무의 70% 정도를 AI가 대체할 수 있을 것이라고 예상했다." 고 합니다. 실제로 연구되고 있는 분야였네요.
대부분 AI 도입을 환영하지만, 모든 판결을 AI가 내리는 것은 불가능하다는게 전문가들의 의견이라고 합니다.
계인국 사법정책연구원 박사는 "기술이 발전하면서 AI의 판단과 실제 판결 간 격차가 좁혀들 수는 있지만, 단순 법률 적용의 문제를 넘어 `법적 가치평가`가 개입되는 사건에서는 AI가 법관을 완전히 대체하기는 힘들 것으로 보인다"고 내다봤다.
의료 부분도 실제로 AI 상품이 많이 개발이 되었지만, 검증을 제대로 하지 못한 상태라고 합니다.
원인이라면 아무래도, 사람의 생명을 다루는 일이기 때문에, 그렇다고 하네요.
<보충자료> - 이상구
세계 최고 '의료 빅데이터' 썩히는 한국
https://www.hankyung.com/it/article/2019052199581?fbclid=IwAR15UIT82bGyWV0qami8s_g3pUqahc89ReJTiYDJMK6HMTqiCvlvdWElaE4
위 자료의 내용은 의료 빅데이터가 개인정보 유출의 우려 때문에, 활용할 수 없다는 것입니다.
의료 빅데이터 활용에 가장 큰 걸림돌은 시민단체와 정치권이다. 경제정의실천시민연합 등 시민단체들은 의료 개인정보 유출우려가 있다며 민간의 공공 의료 빅데이터 활용을 반대하고 있다. 송승재 한국디지털헬스산업협회장은 “의료 데이터를 보는 편협한 시각을 버리고 개인정보를 보호하면서도 의료 빅데이터를 국민 건강에 이롭게 쓸 방법을 찾아야 한다”고 말했다.
문학과 로봇
Ask by 이상구, Date: 2019.05.18
Finalized by 강민석, Date: 2019.05.30
Final OK by SGLee
<질문> - 이헌수
문학을 분석하고 데이터화 한다면 사람들이 생각하는 완벽에 가까운 작품들이 많이 나올 것 같다는 생각이 드네요.
제 생각에는 바둑의 알파고와 비슷한 경우가 될 것 같아요. 알파고가 처음에는 바둑기사들의 바둑을 분석한 방대한 데이터를 토대로 점점 발전하여 인간을 따라잡고 딥러닝으로 더욱 발전하여 새로운 창의적인 수들마저 엄청 만들어내어 이제는 인간이 알파고에게 바둑을 배우고있죠
그것처럼 문학도 사람들이 명작이라고 생각하는 혹은 좋아하는 작품을 만드는데 단어,구조,감정,진행 등 여러가지 방면을 분석하고 그 데이터를 바탕으로 글을 쓴다면 좋은 작품들이 많이 나올 것이라 생각이 드네요. 또 창의성 문제도 과학에서 많은 변수를 고려하며 여러가지 실험을 하듯 문학에서도 여러가지 실험을 하며 그 반응을 또 분석하고 데이터화시키며 딥러닝을 한다면 창의성 마저 갖춘 작품이 나올 것 같다는 생각이 듭니다. 알파고가 바둑 세계랭킹 1등인것처럼, 문학에서의 베스트셀러 작가가 글쓰는로봇 혹은 그런 도움을 받은 작가가 될수도 있겠네요!
여러분들은 어떻게 생각하시나요??
<답변> - 조영은
윗 글에서 밀씀해주신 것처럼 로봇이 문학작품을 쓰는 것도 데이터베이스를 이용한다면 충분히 가능하다고 생각합니다. 그렇지만 로봇이 쓰는 단어, 문장 형식, 어체 등은 과거에 어떤 사람이 사용했던 글을 토대로 한 것이므로, 새로운 아이디어나 독특한 어체, 아무도 생각해보지 못한 작품 소재를 사용한 글은 나오지 않을 것 같습니다. 예를 들어 ‘해리포터’나 ‘반지의 제왕’과 같이 사람의 상상력에서 탄생한 판타지 소설을 로봇이 만들어내기는 힘들 것입니다.
즉, 제 생각에 로봇이 쓴 문학작품은 완성도가 어느 정도 있긴 하지만,
누구나 예상할 수 있는 전개를 가진 다소 뻔 하다는 평가를 받을 수 있는 작품들이 될 것 같습니다.
<답변> - 김윤수
https://www.seoul.co.kr/news/newsView.php?id=20190322500044
벌써 실용화 되있는 기술이었네요.
이미 단편소설까지 제출해서 공모전 예심을 통과한 수준의 글이라면,
미래에 정말로 AI가 쓰는 글을 읽는 시대가 올 수 있을 것 같습니다.
<답변> - 이헌수
저도 영은님 말에 어느 정도 동의합니다. 결론부터 말하자면 제 생각에는 아주 천재적인 인간 몇 명과 대부분의 로봇이 주인공이 될 것 같네요. 소설뿐만 아니라 다른 여러 방면에서 그럴 것 같습니다.
독특한 어체, 단어, 구성 간단히 말해 '창의력' 이라고 불러도 될까요? 로봇 혹은 AI 는 인간과는 다른 방법으로 '창의력'을 길러나가는 듯하다. 한국에 불었던 AI열풍? AI에 대한 관심이 알파고로 인한 것이니 알파고를 예를 들어보자면. 처음에는 인간을 흉내 내던 알파고가 기존의 인간의 데이터를 학습하고 나서는 (기계이다 보니 방대한 데이터를 금방 학습해버렸습니다) 더 좋은 결과를 얻기 위해 스스로 딥러닝 하여 (가능한 변수들을 바꿔가며 엄청난 양을 실험) 인간이 하지 못했던 새로운 발상들을 해냈습니다. 새로운 경향성? 직관? 이라고 해야 할까요.. 인간은 한 번에 엄청난 양의 데이터를 처리하지 못하기 때문에 천재들이 직관적으로 혹은 기발한 아이디어로 새로운 것을 만들어간다면 기계는 정말 방대한 양의 데이터 처리로 우리가 '창의성'이라고 불리는 것을 해내는 것이죠.
물론 알파고는 바둑AI 였고, 목적은 상대방을 이기는 것 이니 다른 얘기일 수고 있겠죠
이것을 문학작품에 적용시켜본다면, 목적을 인간이 좋아하는 기존에 없었던 새로운 문학작품으로 설정한다면 로봇은 여러 가지 단어, 어체, 구성 등 다양한 변수들을 바꿔가며 작품을 써보는 것이죠. 그리고 이것에 대한 반응을 데이터화 시켜서 경향성을 얻는 것이죠. 즉 새로운 직관을 가지는 거네요. 새로운 작품을 쓰는 작가 탄생!
제 생각에는 로봇의 빠른 데이터 처리속도와 빅데이터의 힘이라면 가능할 것 같습니다. 물론 천재적인 사람들은 이런 것을 직관적으로 혹은 새로운 아이디어로 해결하겠죠.
다들 어떻게 생각하시나요
인간의 직관 혹은 창의성 vs 로봇의 엄청난 양의 데이터 처리로인한 창의성
그래서 저는 뛰어난 일부의 인간과 대부분의 로봇이 미래를 주도할 것 같습니다.
<답변> - 최민수
장편 소설 같은 경우에 긴 플롯이나 복선, 설정 등을 다 고려하고 쓰는 경우가 많은데 로봇이 그걸 대체하는 것이 과연 가능할까요.
시의 경우에는 일단 시적인 문체를 쓰는 것만으로 거의 다 해결이 되니 크게 상관이 없지만.
물론 머신러닝을 어떻게 하느냐에 따라서 그것은 모르겠습니다.
프로그램을 짜는 사람의 능력에 달린 것 같기도 하구요
<답변> - 강민석
소설 뺨 치게 긴 시도 많습니다. 중요한 건 분량이 아닙니다. 그럼 작품의 '생산'에 기계가 개입할 만한 중요한 요점이 무엇일까. 저는 '형식'이라고 봅니다. 시는 그 출발은 정형시로써 형식이 있었지만 지금 와서는 거의 형식이란 단어를 초월한 상태입니다. 심지어는 그냥 아무 것도 쓰지 않은 백지 한 장조차도 시가 됩니다.
중요한 건 거기에 어떤 의미를 담았느냐 인데, 이게 참 추상적인 말입니다. 이런 추상성을 기계가 감당할 수 있을까. 전 회의적입니다.
반면 소설은 그나마 형식이란 걸 잡기 쉽습니다.
기본은 기승전결, 혹은 발단/전개/위기/절정/결말 식이고, 순문학을 전문적으로 다루는 게 아니고서야, 오히려 이런 형식을 벗어난 글은 너무 낯설게 받아들여져서 수요가 없습니다.
즉 어느 정도는 형식이 정해진 게 소설입니다. 그렇다면 형식에 맞춰서 적절한 문장을 서사에 맞게 끼워 넣기만 한다면 소설이 될 것입니다. 기계가 이 정도로 치밀해질 수 있을지는 의문이지만, 저는 적어도 시보다는 가능성이 높다고 생각합니다.
<정리> - 강민석
문학을 데이터화하고 플롯을 정형화하여 로봇이 소설을 창작하는 것은 가능하다. 당장 공모전 예심에 당선된 사례가 있다. 하지만 경험적 프로세스에 의한 산물로서 어느 정도 뻔한 작품이 나오는 것은 감안할 수밖에 없다. 완전히 새로운 것을 창작하는 것은 불가능할 것으로 보인다. 추상성과 중의성을 중시하는 시 같은 경우는 로봇이 창작하는 것이 힘들 수 있겠으나, 적어도 소설에 한해서라면 로봇이 충분히 양산 가능할 수 있다.
나이팅게일과 통계
Share by 조영은, Date: 2019.04.10
Finalized by 최민수, Date: 2019.04.13
Final OK by SGLee
<정보공유> - 조영은
<통계로 세상을 구한 나이팅게일>
위생의 중요성을 통계를 통해 증명한 나이팅게일의 선구자적 모습에 감명을 받게 됩니다. '법률과 정부의 부적절함, 정치 체계의 무능함, 사회를 이끄는 자들의 답답한 몽매함’을 오직 통계 연구로만 바르게 이끌 수 있다고 여겼다는 대목에서 그녀가 얼마나 통계학을 중요하게 생각했는지를 알 수 있습니다. 모든 사람은 통계학을 이해해야 한다고 말하는 그녀의 생각에 크게 공감이 갑니다.
<답변> - 김윤수
나이팅게일과 통계라니 전혀 예상치 못했고 알지 못했던 조합이네요. 좋은 정보 감사합니다.
<정리> - 최민수
나이팅게일의 이미지는 롱펠로가 읊은 것처럼 전쟁터에서 부상을 당한 병사를 밤새 보살핀 ‘백의의 천사’다.
그렇지만 나이팅게일은 영국 군대와 도시의 위생문제를 개선하기 위해 통계를 적극적으로 사용한 인물이기도 하다
나이팅 게일이 통계에 주목한 이유
나이팅게일을 포함한 대부분의 사람들이 질병은 더럽고 바람이 통하지 않는 병실이나 방에서 자연스럽게 생긴다고 믿었다. 그래서 나이팅게일은 좋은 간호를 하기 위해선 우선 환경을 청결하게 유지해야 한다고 생각했다.
그래서 나이팅게일은 야전병원의 위생을 개선하기 위해 숫자로 야전병원의 상황을 정확히 파악하기 위해 노력했다.
개선사업을 시작한지 한 달이 지나자 야전병원의 사망률이 급격히 떨어졌다. 42%에 달하던 환자의 사망률이 2%까지 떨어졌다.
전쟁후에도 나이팅게일은 간호론이라는 책을 내서 치유와 예방법을 전파했고, 영국을 발전시키기 위해서 통계를 적극 활용했다
보고서
팀원: 최민수, 김윤수
주제: 보험수학
목차
1. 주제 선정 이유
2. 보험이란 무엇인가
3.보험의 종류
4.보험의 역사
5. 초기 보험수학과 초기 보험계리사
6. 한국과 미국의 보험계리의 차이
7. 한국보험계리학과의 역사
8. 후기
1. 주제 선정 이유
2016년 기준으로 세계 평균 수명은 약 83세에 달한다. 정년 연령이 60살에서 65세로 올해 늘어나긴 했지만, 결국 18년 정도를 은퇴 후에 살아가야 하고, 사람들이 오래 산다고 해도 그것이 병에 걸리지 않는다거나 사고가 나지 않는 것은 아니다. 또한 사회가 갈수록 고도화하고 복잡해지면서 불확실성을 대비하기 위해서 어느 것이든 보험을 들면서 안전장치를 마련하고 있다. 그 예로 중국에는 배우자가 바람을 피우면 상대방이 돈을 받는 외도 보험이 존재하고, 외계인 납치됐을 때를 대비하는 보험, 응원하는 축구팀이 졌을 때를 대비한 축구 트라우마 보험 등이 있는 것처럼 말이다. 그리고 수학과에서 자신의 전공을 살려 취업하는 몇 안 되는 진로이기도 하며, 새로운 국제회계기준이 도입되면서 요즘 같은 취업난이 있는 시기에 사람이 없어서 못 구한다는 몇 안 되는 직종이기 때문에 보험수학을 주제로 선택하게 되었다.
2. 보험이란
보험: 보험계약은 당사자 일방이 약정한 보험료를 지급하고 재산 또는 생명이나 신체에 불확정한 사고가 발생할 경우에 상대방이 일정한 보험금이나 그 밖의 급여를 지급할 것을 약정함으로써 효력이 생긴다. 쉽게 말해서 큰 일이 생겼을 때 혼자서 감당하기 힘드니까 많은 사람이 조금씩 모아 큰돈을 만들고 그 중 큰 일이 생긴 사람에게 몰아주는 식으로 위기를 극복해나가는 것이라고 할 수 있다.
ㄱ. 생명보험이란: 넓은 의미로는 자연인을 대상으로 하여 사고나 재해를 보상해주는 것이고, 좁은 의미로는 피보험인(자연인)이 사망하였을 때에 소정의 금액을 지급하는 것이다. 생명보험계약은 보험자(보험 사업을 하는 사람)가 보험계약자 또는 제3자(피보험자)의 생사(生死)에 관하여 일정한 금액(보험료)을 지급할 것을 약정하고 보험계약자는 피보험자의 생명에 관한 보험사고가 생길 경우에 약정한 보험금액을 지급할 것을 약정하는 인 보험계약이다(대한민국 상법 제730조). 생명보험의 보험사고는 피보험자의 생사, 즉 생존 또는 사망이다.
ㄴ. 연금보험이란: 피보험자의 종신 또는 일정한 기간 동안 해마다 일정 금액을 지불할 것을 약속하는 생명 보험. 넓은 뜻의 연금보험은 의료보험·산업재해보상보험·고용보험과 더불어 사회보험의 4대 분야를 이룬다. 한국에서는 일반 국민에 대한 연금보험이 1994년부터 도입되어 실시되고 있다.
ㄷ. 손해보험이란: 보험자가 우연한 사고(보험사고)로 생기는 손해를 전보할 것을 약정하고, 보험계약자가 이에 보험료를 지불할 것을 약정하는 보험이다(상법 665조). 정해진 금액을 지급하는 생명보험과 다르게 손해의 정도에 따라서 다르게 지급한다.
3 보험의 역사
고대에는 오늘날과 같은 보험제도는 없지만 집단으로 살아가면서 장례비용을 나눠서 내는 경우가 있었다. 그리고 마을 사람들 중 천재지변, 도난 등으로 손해가 생기면 그것을 공동부담으로 돕는 제도 등이 있었다. 중세에는 길드 조직을 중심으로 서로 돕는 방식과 항해를 하다가 생긴 배나 적재한 화물의 손해를 공동으로 부담하는 제도가 있었는데 이로부터 보험제도가 생겨나게 된다.
생명보험(생명표)
통계학과 확률론에 기반하는 최초의 보험계리를 적용시킨 보험이 바로 생명보험이었다. 본래 해상보험이나 화재보험의 경우는 재화를 보상의 대상으로 하는 대물 보험이었기 때문에 보험금으로 지급해야 하는 액수가 명확하였고, 리스크와 무관하게 일정한 보험료 체계가 존재했다.
반면 생명보험은 다른 보험에 비해 리스크와 매우 밀접한 관련이 있었고, 이 리스크는 예측 불가능한 것이었다. 당시 보험업자의 입장에서 보면, 한 명의 보험 가입자로부터 예상되는 수익과 손실을 예측할 수 없었던 것이다. 즉 인간 수명에 대한 예측이 불가능했기 때문에, 많은 보험업자들이 높은 보험료율을 제시했다가 고사하거나, 혹은 지나친 보험금 지급으로 파산하기도 하는 등 기존의 생명보험은 도박으로 끝날 수밖에 없었다.
따라서 생명보험 업계에서 절실하게 필요했던 것은 바로 ‘사망률을 예측하는’ 방법이었다. 그런데 이것이 수리학의 발달에 의한 확률론의 등장과 사망 집계를 누적시켜 통계 처리한 사망표에 의해 해결된 것이다.
사망보험은 1706년에 설립된 아미카블 소사이어티를 시작으로 보고, 생명보험의 시초로 본다. 1764년 문을 연 에퀴터블 생명보험(Equitable Assurance)는 최초로 인간의 사망률과 보험 산업을 결합했다. 에퀴터블의 고안자인 제임스 도슨은 최초로 통계와 확률론을 도입해서 예상 수명을 예측하고 보험에 이것을 적용해서 적절한 보험료 및 수익성을 보장하였다. 최초의 보험계리인인 윌리엄 모스델이 나이에 따른 생존율을 기록한 표를 기반으로 보험료를 측정해서 장기 보장을 유지하면서 낮은 보험료로 많은 고객을 유치하였다.
윌리엄 모스델이 고안한 노샘프턴 생명표. 보험 계리를 도입한 생명보험으로 에퀴터블은 큰 수익을 얻을 수 있었다.
화재보험(런던 대화재)
1666년 9월 2일 새벽에 어느 빵집에서 치솟은 불길은 5일 동안 타오르면서 대부분이 목조 건축물로 이루어져 있었던 런던 시내 가옥의 80%를 태워버렸다. 이 화재로 타버린 교회나 관청 같은 공공건물만 87개에 달하며 그중에는 런던의 성 베드로 성당도 포함되어 있었다.1667년 치과의사 니콜라스 바본(Nicholas Barbon)은 국왕의 명을 받아 대화재를 대비하고 화재 피해자들을 구제하기 위한 사무실을 열었다. 혹시나 화재가 발생하여도 그 피해 금액을 보상해준다는 제안은 한창 도시 재건 공사에 열을 올리던 런던 시민들에겐 그야말로 솔깃한 소식이었다. 제대로 된 이름도 존재하지 않고 그저 화재사무소(fire office)로 시작했던 이 사무실은 폭발적으로 성장했고, 이와 유사한 기능을 하는 화재보험들이 급증하기 시작하였다. 바본의 화재보험 사무소는 1680년 합자회사로 성장하여 단기간에 정착한 대표적인 화재보험회사 가운데 하나가 되었고, 1705년 피닉스 화재 사무소(PheonixFire Office)로 개명하면서 약 100여년 간 영업을 지속했다.
해상보험
초기의 보험 중에서도 해상보험은 주로 금융업자나 상인 계층 중 많은 자본을 소유한 사람들을 중심으로 공증인을 통해 보험계약이 이루어진 것이었다. 전문적으로 보험업을 하는 보험업자들은 16세기 후반에나 등장하였다. 이후로도 해상보험은 꾸준히 로이드 커피하우스의 보험업자들과 같이 개인 보험업자들을 중심으로 발전했다.
최초의 해상보험 회사가 등장하게 된 것은 1720년으로, 왕의 특허장을 받은 로열 익스체인지 어슈어런스(Royal Exchange Assurance)와 런던 어슈어런스(London Assurance)가 설립된 것이 그 시작이었다. 당시 금융시장의 혼란을 잠재우기 위하여 이 두 회사를 설립한 조지 1세가 “화재보험, 생명보험, 해상보험 영업에 특허를 가진 유일한 회사들”로 특허장을 발부한 것이다. 당시의 법안은 이 두 회사를 제외한 어떠한 회사도 해상보험 영업을 할 수 없도록 규정하고 있었기에, 이들의 영향력은 그야말로 막강할 수밖에 없었다.
흥미로운 점은 18세기에 번성하던 보험회사들이 아직도 군림하고 있다고 볼 수 있고, 그 예로 당시의 에퀴터블 생명보험 회사가 AXA 그룹의 일부로 존재하고 있기도 하다는 것이다.
5. 초기 보험수학과 초기 보험계리사
5.1 기본 개념
보험수학이란 불확실한 미래의 리스크를 수학적 도구를 이용해서 측정하고 관리해서 보험사와 계약자의 이익을 보장하기 위한 학문이다. 보험수학은 그 특성 때문에 수학뿐만 아니라 통계학, 경제학, 그리고 컴퓨터 공학까지 연관되어 있다. 많은 대학들은 학부와 대학원에 보험수학 관련 커리큘럼을 짜놓았다. 이자율에 반대되는 개념으로 현가라는 개념이 있다. 이자는 미래에 돈이 얼마나 불어날 지를 예측하는 도구이다. 그에 반해서 현가는 미래의 그 돈이 적립되기 위해서 지금 있는 돈의 가치를 뜻한다. 보험은 필연적으로 미래의 일을 상정하기 때문에 현재의 그 가치가 어느 정도인지를 측정하는 것이 중요한데 그때 사용되는 것이 현가라는 개념이다.
5.2 초기 개발
17 세기는 독일, 프랑스, 영국에서 수학의 발전의 시기였다. 동시에 급속도로 발전했으며 개인적 위험에 대한 가치 평가를 보다 과학적으로 수행 할 필요가 있었다. 서로 독립적으로, 복리에 대한 관심이 연구되었고, 확률 이론은 잘 이해된 수학적 학문으로 나타났다. 또 다른 중요한 진전은 1662 년에 John Graunt 에게서 나왔다. 그는 어떤 한 사람의 사망 날짜의 불확실성에도 불구하고 같은 나이의 사람들의 집단 또는 코호트에서 장수와 사망의 예측 가능한 패턴이 있음을 봤다. 한 그룹의 사람들에게 생명 보험이나 연금을 제공하고 그룹의 각 개인이 일정한 이자율을 얻는 것으로 가정된 공통 기금에 어느 정도 기여할 수 있는지를 어느 정도 정확하게 계산할 수 있는 보험을 설정할 수 있었다. 이것이 어떻게 할 수 있었는지 공개적으로 시연한 최초의 사람은 에드몬드 할리였다. 할리(Halley)는 생명표를 세우고 그것이 주어진 연령의 누군가가 생명 보험료를 지불하기 위해 지불해야하는 보험료 금액을 계산하는 데 어떻게 사용될 수 있는지를 보여주었다.
5.3 초기 계리사, 유명한 보험수학자들
(주:The Equitable Life Assurance Society(1762)는 영국의 가장 오래된 보험회사이다.)
제임스 도슨(James Dodson)
영국의 수학자, 45세보다 나이가 많아서 보험 계리사 Amicable Life Assurance Society에 입사를 거부당했다. Dodson은 1693 년 에드먼드 할리 (Edmund Halley)가 개발한 통계적 사망률 표를 작성했다. Dodson이 소속된 Equitable Life는 장기 생애 보장 정책의 리스크를 상쇄하는 것을 목표로 한 청구액을 청구했다.
제임스 조셉 실베스터(James Joseph Sylvester)
1884년에 the Equity and Law Life Assurance Society 에 고용되었었고 성공적으로 보험 모델을 개발했습니다. 또한 아서 케일리와 함께 행렬의 발전에 지대한 공을 미쳤고, 행렬(matrix)라는 단어를 최초로 사용하였다. 또한 American Journal of Mathematics를 창간하였다.
바다 항해법으로 유명한 수학자.아버지의 강요로 일을 하기 위해서 10살 때 학교를 그만두어야 했지만 스스로 수학을 자습하고 라플라스의 책을 번역해서 출판해서 미국의 천문학의 발전에 지대한 공을 세웠다. 그 뒤에 첫 항해를 나가고 수학을 공부하기 위해서 돌아오고 보험업계로 진출한다. 1804년에 Bowditch는 the Essex Fire and Marine Insurance Company in Salem, Massachusetts 의 사장으로 미국의 첫 번째 보험계리사가 되었다.
5.4 금융 경제학과 보험 수학
제한 조건과 투자전략 등이 다르기 때문에 전통적인 보험수학과 현대 금융 경제학은 실제 활용이 다르다.
역사적으로 보험 통계 이론의 기초는 현대 금융 이론에 기반을 두었다. 20세기 초 보험 계리사는 현대 금융 이론에서 발견 할 수 있는 많은 기법을 개발하고 있었지만 이러한 개발은 많은 인정을 얻지 못했다 (Whelan 2002). 그 결과 보험 통계 과학은 현대 금융에서 사용 된 차익 거래없는 Risk-Neutral 개념과는 달리 가정에 더 의존하게되어 다른 경로를 따라 발전했다. Risk-Neutral이란 기댓값이 공정하도록 조정한 확률을 말하며 이 때문에 이자율을 따로 고려할 필요가 없어 계산이 편해진다. 금융 경제학과 보험수학의 차이는 데이터의 사용 및 현금 흐름의 통계적 예측과는 관련이 없고 대신 전통적인 보험 통계 방법이 이러한 수치로 시장 데이터를 적용하는 방식에 기인한다. 예를 들어, 하나의 전통적인 보험 수리적 방법은 투자의 자산 배분 혼합을 변경하면 (할인율 가정을 변경하여) 부채와 자산의 가치를 바꿀 수 있음을 암시한다. 이 개념은 금융 경제학과 일치하지 않는다.
6. 한국과 미국의 보험계리의 차이
ㄱ. 한국의 경우
1차시험으로 영어(토익점수 700점 이상 or 텝스 625점 이상 or 토플 iBT 71점이상 달성),보험계약법, 보험업법 및 근로자퇴직급여보장법, 경제학원론, 보험수학, 회계원리 등을 시험친다. 2차시험은 1차시험에 합격한 해를 포함하여 5년간 응시가 가능하고, 5년간은 과목별 부분합격이 가능하다. 합격자 인원 제한은 없고 응시해야하는 과목은 계리리스크 관리, 보험수리학, 연금수리학, 계리모형론, 재무관리 및 금융공학 등이 있다.
ㄴ. 미국의 경우
ASA(Associate)레벨까지는 7단계, Fellow(FSA)레벨 까지는 ASA이후 8~10단계의 과정을 거쳐 고난이도 시험이다. P(Probability), FM(Financial Mathematics), IFM(Investment and Financial Markets), LTAM(Long-Term Actuarial Mathematics), STAM(Short-Term Actuarial Mathematics), SRM (Statistics for Risk Modeling), PA (Predictive Analysis) 시험과 FAP Modules까지 합격하고 필수과목 이수사항을 인증해야 ASA자격(준계리사)이 주어진다. Fellow까지 따려면 10년정도 걸리는게 일반적이지만 계리업무와 병행하면서 따는 자격증이라고 생각하면 된다.①기초시험 5개를 통과하고, ②대학교에서 필수과목을 들은 게 인증이 된 상태에서, ③실무 시험을 봐야 하며, 이 모든 게 끝나고 ④세미나를 참석하면 ASA가 된다.
Exam P : 기초확률론이다. 대학교에서 확률론 수업을 들었으면 어렵지 않게 통과할 수 있을 것이다.
Exam FM : 이자론(Interest theory) + 파생상품 기초(financial economics)를 다루는 과목이다.
Exam MFE : 흔히 "금융공학"이라 불리는 과목이다. 블랙 숄즈 모형이나 이토렘마 모델 등 본격적인 수학의 향연이 펼쳐지며 확률미분방정식을 풀 수 있는 정도가 돼야 한다. 한국계리사 2차 과목인 "재무관리&금융공학"과 겹치기는 하나, 우리나라 시험은 "재무관리"쪽에 거의 집중돼있기 때문에, 사실상 겹치는 부분이 거의 없다고 봐도 된다.
Exam MLC : 우리나라 "보험수리학"과 대응되는 과목이다. 원래 보험수리학은 생명보험수리학의 약자로, 생존분포, 생명보험, 생명연금, 다중상태모형, 다중탈퇴모형 등을 배운다. 그러나 MLC의 범위가 한국계리사 범위에 비해 좀 더 넓고, 변액연금 등 공부할 게 더 많다.
Exam C : 우리나라 "계리모형론"에 대응되는 과목이다. 크게 손해모형론(loss model), 추정(estimation), 신뢰도(credibility), 시뮬레이션(simulation) 등을 배운다.
VEE (대학교에서 필수과목을 들은 것 인증)
미국계리사가 되기 위해선 특정 과목을 이수해야 한다. 대학교에서 해당 과목을 듣지 않고 졸업했다면 시험을 보는 것으로 대체 가능하다. FRM 같은 국제 시험을 통해 VEE를 면제받는 경우도 있다.
Economics : 경제학에 해당하는 과목이다.
Corporate Finance : 재무관리에 해당하는 과목이다.
Applied Statistics : 응용통계학에 해당하는 과목이다.
FAP (실무 시험)
FAP에는 8개의 Module이 있으며 Module 4를 끝나고 응시하는 중간고사 IA와 Module 8을 보고 응시하는 FA를 모두 통과해야 한다
APC (세미나 참석) FAP까지 통과했다면, SOA에서 주관하는 세미나에 참석하면 된다.
7. 한국 보험계리학과의 역사
금융강국과 동북아 금융중심국가의 선도적 역할을 수행할 국제적 보험 금융 전문 인재를 양성하기 위해 국내 최초로 성균관대학교의 일반대학원(협동과정)에 보험계리학과를 설립하였다. 한국보험계리사 시험과 국제적으로 인정받는 미국의 보험계리사(Actuary) 시험 과정인 생명보험(Society of Actuaries)과 손해보험(Casualty Actuarial Society)의 동일 부분인 P, FM, MLC, MFE, C 합격을 목표로 한다.
2005년 보험계리학과가 설립된 이후 졸업생들이 국내유수의 보험회사와 유관기관에 취직하여 업무를 수행하고 있다. 높은 취업률과 내실 있는 교육을 통하여 국내 최고 보험계리학과로 자리매김하고 있으며, 우수한 외국인 학생의 유치를 통하여 국제적인 보험계리학과로서 위상을 유지하고 있다. 또한 Society of Actuaries에서 승인한 VEE 인증이 가능한 교육기관이다.
성균관대 대학원 보험계리학과 연혁
2002 한국과학재단 전산수학 연구센터에서 “ Open School”을 개최하여 해외 강사를 초빙하고 미국보험계리사 시험 Course 3을 중심으로 강의를 개설
2003.12 한국과학재단 제 284회 학연산 교류회를 “금융위험관리 연구성과 발표 및 국제보험계리사 양성의 필요성”이란 주제로 서울 무역회관에서 개최
2003.12 포항공대-경북대-성균관대의 총장은 콘서시움을 구성하여 프로그램을 개발
2004 교육부의 인가를 받아, 2004년 가을 신입생을 모집
2005.3 강좌 개설
2006.3 Official Advisory Board meeting
한양대 ERICA캠퍼스에선 2013년에 보험계리학과를 설립하고 보험계리 인재를 양성하고 있는데 이 학과가 한국 최초의 보험계리학과라고 주장하고 있다. 학부만으로 따지면 그것이 맞는 말일 수는 있겠지만 대학원까지 포함하면 성균관대가 8년이나 앞섰다.
8.후기
최민수 : 프로젝트를 진행하고 자료를 찾아가면서 영어로 된 자료를 찾아서 해석하느라 고생도 많이 하고, 사람들이 어떤 내용을 알고 싶어하고 내가 어느 정도 까지 전달할 수 있을까 등을 고민했었습니다. 예전에 보험계리사가 되고 싶다는 생각이 들어서 조사를 해보던 기억이 있어서 아예 맨땅에 헤딩을 하는 것처럼 힘들진 않았습니다. 하지만 자료를 찾다보니 찾으면 찾을 수록 자료가 많이 나와서 어느 정도 요약하고 정리하느라 정말 중요한 자료들도 보고서에 넣지 못한 경우가 필연적으로 생겼는데 아쉽다고 생각이 들면서도 선택과 집중 즉 편집의 중요성을 절감했습니다. 사회에 나가서도 우리는 프로젝트를 시행하고 다른 사람들에게 보여줘야 하는 보고서를 작성해야 할 것입니다. 그때에도 부족하지만 최선을 다해서 자료를 찾고 남들이 이해할 수 있고 가치있는 정보를 생산해 낼 때에 수학사 시간때 했던 경험들이 큰 재산이 될거라고 느껴졌습니다. 또한 결국 전달하는 사람도 중요하지만 결국 듣는 사람도 정말 중요합니다. 그런 의미에서 QnA에 자신의 프로젝트 진행방향등을 공유하고 전문가인 교수님의 조언을 받아서 수정하고, 발표를 들을 동료 학우분들이 궁금해 하는 부분을 체크했었던 경험은 나중에도 좋게 작용할 것 같습니다. 결국 이 프로젝트를 진행하면서 힘들었던 일도 많았지만 미래의 제가 성과를 내기위한 성장통이었다는 생각이 들었었고 앞으로도 잘 해나갈 수 있다는 자신감이 들었습니다.
김윤수 : 요즘 보험가입을 권유하는 전화를 굉장히 많이 받았었다. 하지만 난 보험에 가입하는 것에 대해서 굉장히 회의적인 생각을 갖고 있었다. 보험을 설계하는 것이 수학적으로 정밀한 계산을 통해서 이루어진다는 것을 막연히 알고 있었고, 보험사는 이익을 추구하는 영리단체이므로, 대부분의 상황에서 피보험자는 손해를 볼 것이라 생각했기 때문이다. 확률론의 기댓값을 생각해 보면 보험에 가입하는 것은 손해 보는 일이라고 생각했다.
아마 과거의 런던 사람들도 나와 같은 생각이 아니었을까 생각했다. 직접 대화재를 겪기 전까지 화재보험의 필요성을 못 느꼈던 것이다. 나는 아직도 많은 보험들의 필요성을 느끼지 못하지만, 고통을 나눈다는 취지에서 보험이 굉장히 인간적이고, 매력적이라고 느껴졌다. 또 이를 설계하는 보험계리사 또한 매력적으로 느껴졌다.
수학사 Final project
Spring, 2019
Final Project : 오일러, 가우스, 힐베르트 & 컴퓨터 수학
Name 이름 : 이헌수 , 강민석
Intro
가장 위대한 수학자가 누구일까요? 피타고라스? 오일러? 수많은 위대한 수학자들이 역사에 이름을 남겨왔고 각각 뛰어난 업적들을 남겨 누가 가장 위대하다고 말하기는 힘듭니다. 수학은 고대로부터 계속 발전해오고 있었지만 중세 이후 근대부터 현대에 이르기까지 엄청난 속도로 발전하였고 우리는 그 기간 동안 위대한 업적을 남긴 수학자들을 알아보려합니다. 18세기, 19세기, 그리고 20세기를 대표할 만한 수학자를 각각 한 명씩만 꼽았고, 그 결과 오일러, 가우스, 힐베르트 이렇게 세 수학자가 각 세기를 대표할만한 위대한 업적들을 남겼다고 판단하였고 이들이 수학계에 남긴 업적을 통해 그 업적 수학을 또 세상을 어떻게 바꾸었나를 중심으로 알아보고자 합니다. 또한 21세기 수학에 큰 업적을 남기고있는 컴퓨터 수학에 대해서도 알아 보았습니다. 전문적인 수학 지식을 나열하기 보다는 전문성을 줄이고 접근성을 높여 수학에 관심이 있다면 누구나 알아들을 수 있는 것을 목표로 이 프로젝트를 진행하고자 합니다.
Ch. 1 레온하르트 오일러 (18c)
1)<사진1.레온하르트 오일러 초상화>
1. 오일러의 업적
1) 수학 용어 표기
현대의 수학의 거의 모든 기틀이 오일러의 손에서 세워졌다 해도 과언이 아닐 정도로 오일러의 영향은 막대합니다. 당장 수학 용어의 표기에서부터 그 영향력이 드러납니다. 원주율을 ݝ¿라 부르는 것을 상용화시킨 사람이 오일러입니다. 허수, √-1을 ݑ로 최초로 표기한 사람 역시 오일러이고, 자연상수 ݑ를 처음으로 발견하고 표기한 사람 역시 오일러입니다. 함수를 가리키는 f(x)라는 표기 역시 오일러의 작품입니다. 사실상 중학교 수학에 들어서는 순간 이미 우리는 오일러의 영향력을 벗어날 수 없게 되는 것입니다.
2) 오일러 방정식
오일러의 가장 큰 물리학적 성과는 현실의 물리 법칙을 미적분을 사용하여 수학적으로 표기하고 분석적으로 해결한 데에 있습니다. 적분의 근사값을 구하는 오일러의 방법이나 유체의 움직임을 미분방정식으로 기술한 오일러 방정식 등이 그 예입니다. 오일러 방정식은 현재도 풀리지 않는 비점성 유동에 관한 미분방정식이며, 여기에 점성에 관한 항 하나만 더 붙이면 익히 알려진 밀레니엄 문제 중 하나인 내비어-스톡스 방정식이 됩니다. 현대수학의 최고 난제인 밀레니엄 문제에도 오일러의 자취가 남아있는 것입니다.
3) , 오일러의 공식, 오일러의 황금열쇠
함수의 해석에 관해서도 오일러의 업적을 빼놓을 수 없습니다.
우선적으로 언급할 만한 것은 함수를 분석함에 있어 급수로 변환시켜 생각하는 (ex. ) 창의적인 발상입니다. 자연수의 제곱수의 역수의 합이 ݜ²/6이라는 증명이 이런 발상으로 이루어지고, 이런 함수의 해석을 통해 정수의 성질을 파악하려는 시도가 지금의 해석적 정수론의 모태가 됩니다. 미분방정식과 복소해석학의 강력한 무기가 되는 오일러의 공식(
) 역시 이런 발상의 유산이고, 모든 소수의 역수의 합은 발산한다는 발견과 리만 제타 함수를 효과적으로 변형시키는 오일러의 황금열쇠(
) 또한 빼놓을 수 없습니다.
4) FLT n=3, 오일러 정리, 골드바흐 추측, 2차잉여의 상호법칙
정수론에서의 오일러의 업적은 페르마의 영향을 많이 받았습니다. 페르마의 마지막 정리가 n=3일 때 성립함을 증명하였고, 페르마의 소정리를 일반화한 오일러 정리가 유명합니다. 골드바흐의 추측을 수학계에 널리 알린 사람 역시 오일러이며, 자연수 n과 서로소인 n보다 작은 수의 합을 ݚ½(n)(totient function, 파이함수)라 이름붙이고 이를 분석하여 짝수 완전수의 성질을 완벽하게 규명한 사람 역시 오일러입니다. 2차잉여의 상호법칙을 처음으로 추측한 사람이기도 하며, 이를 결국 증명해내지는 못했으나 훗날 가우스가 증명에 성공합니다.
5) 쾨니히스베르크 다리 그림, 한붓그리기, V-E+F=2
쾨니히스베르크 다리 문제 역시 빼놓을 수 없는 업적입니다. 이 증명방식은 도형과 영역을 다루는 완전히 다른 접근법으로 크게 알려져 훗날 위상수학의 모태가 됩니다. 한붓그리기의 가능 여부를 일반화하기도 하였으며, 평면기하에서의 오일러 정리 역시 추상적인 도형의 분석에 큰 기여를 한 공식입니다.
2. 그 외 업적 및 Comment
해를 구하기 힘든 식에 초기값을 대입하고 그 결과값을 대입하는 것을 반복하는 수치해석학의 알고리즘을 처음으로 개발한 사람 역시 오일러이며, 그 외에도 오일러 각도, 오일러-매클로린 공식, 오일러-마스케로니 상수, 오일러-베르누이 빔 방정식, 논리학의 오일러 다이어그램, 광학이나 음악의 수학적 분석 등등 오일러의 업적은 수학계 여러 분야에 셀 수 없이 많이 스며있습니다. 오일러가 수학사에 미치는 진정한 의의는 폭넓고 창의적인 발상과 직관이 수학의 하위 학문의 모태가 되어 결과적으로 수학이라는 학문의 지평을 크게 넓혔다는 데 있습니다.
Ch. 2 카를 프리드리히 가우스 (19c)
1. 가우스의 업적
1) 가우스 소거법, 최소제곱법
가우스의 업적은 대학 수학을 공부하기 시작하고 나서야 실감하는 경우가 대부분입니다. 아마 가우스의 업적을 가장 쉽게 접할 수 있는 분야는 선형대수학일 것입니다. 가우스 소거법은 기본 행 연산을 통해 행렬을 분석하고 변형하는 가장 기초적인 방법입니다. 변형된 이차곡선인 이차형식을 연구하고 그래프 상에 흩뿌려진 수많은 데이터들을 분석하기 위한 추세선을 구하는 최소제곱법을 개발하는 등 행렬에 있어서 가우스의 업적은 절대적이라 하여도 과언이 아닙니다.
2) 대수학의 기본정리 증명
대수학에서 가우스의 이름을 가장 크게 떨친 업적은 대수학의 기본정리의 증명일 것입니다. 모든 n차 방정식은 하나 이상 n개 이하의 복소근을 무조건 갖는다고 요약할 수 있는 이 정리는 오일러가 증명하였다고 결론지어졌던 정리였지만, 가우스는 그 증명에서 오류를 발견하고 새로 증명해내었습니다. 지금에 와서는 가우스의 증명에서도 오류가 발견되었지만, 당시에는 큰 반향을 일으키는 데 성공했습니다.
3) 정수론에서의 업적
정수론에서의 가우스의 업적은 합동식, 모드(mod)에서 가장 크게 드러납니다. 합동방정식의 등식 기호를 상용화시키고, 합동방정식의 해법을 연구하고 발표하여 당시까지와는 완전히 다른 정수론을 개척하는 데 성공하였습니다. 오일러가 추측하고 끝내 증명하지 못했던 이차잉여의 상호법칙을 처음으로 증명해낸 것 역시 정수론의 큰 업적으로 꼽힙니다. 모든 자연수는 세 개 이하의 삼각수로 표현될 수 있다는 페르마의 삼각수 추측을 증명한 것 역시 특별한 업적입니다.
4) 기하학에서의 업적
기하학에서의 가우스의 업적을 꼽으라면 곡면의 연구에 있습니다. 곡면을 함수로 생각하여 유클리드 공간에 놓고 생각하는 미분기하학의 기본 정리 중 하나가 가우스의 빼어난 정리입니다. 비유클리드 기하학에 대한 발상 역시 일찍이 가지고 있었지만 이를 정식으로 발표하지는 못했습니다. 가우스가 남긴 말에 따르면 ‘아둔한 사람들이 얼마나 악을 쓰며 덤빌지 겁난다.’는 것이 이유였습니다. 훗날 비유클리드 기하학을 정식으로 발표하는 데 성공하는 우리가 잘 아는 리만이 바로 가우스의 제자 중 하나입니다.
2. 그 외 업적 및 Comment
이외에도 가우스의 업적은 수학과 수리물리학 요소요소에 녹아들어 있습니다. 통계학에서 ∫e^(x^2)dx = √ݝ¿를 간단히 증명하고 표준정규분포함수를 정의하여 적분을 이용한 수리통계학의 근간을 마련한 사람이 가우스이며, 다중적분에서 적분기호를 하나 줄여주는 큰 무기가 되는 발산 정리 역시 가우스가 발견하였고, 복소평면이란 개념을 도입한 사람 역시 가우스이며, 그 복소평면에서 정수를 연구하는 색다른 방법인 가우스 정수, 전자기학에서 맥스웰 방정식의 한 축을 구성하는 가우스 법칙, 천문학에서 외행성 세레스의 궤도를 계산하는 데 쓰인 가우스 방법, 17각형의 작도 가능 여부 문제 해결, n=5일 때의 페르마의 마지막 정리의 증명, 산술평균과 기하평균 간의 관계성 규명 등 가우스의 업적은 찾아볼수록 그 끝을 알 수가 없습니다. 수학이라는 넓은 학문의 어디를 가도 그 이름이 빠지는 곳이 없기에 가우스는 가장 위대한 수학자로 대중들에게 깊이 각인되어 있습니다..
Ch. 3 다비트 힐베르트 (20c)
1. 힐베르트의 업적
1) 기하학의 공리화
다비드 힐베르트의 업적을 간략히 설명하자면 우선 기하학의 공리화를 들 수 있습니다. 다소 약점이 있었던 유클리드의 공리를 보완하여 더 엄밀하고 논리적인 공리를 내세우고 점, 선, 평면 등등 기하학의 기둥뿌리를 더욱 체계적으로 하나하나 정의해내었습니다.
2) 아인슈타인-힐베르트 작용
아인슈타인의 일반 상대성 이론을 수학적으로 정의하는 핵심적인 역할을 하여 현대물리학과 현대수학을 잇는 중요한 업적을 이루었습니다.
3) 힐베르트 공간
유클리드 공간을 일반화 한 힐베르트 공간을 정의하여 지금까지의 함수론을 통합하여 함수해석학의 장을 열었습니다.
4) 힐베르트의 23문제
1800년대 후반에 최초로 전 유럽과 미국의 내로라하는 수학자들을 모아놓고 열린 세계 수학자 대회에서 수학의 발전방향이란 주제로 연설을 하게 된 힐베르트는 이 ‘발전방향’이라는 애매한 주제를 ‘현대 수학이 가장 시급히 해결할 필요가 있는 문제들’의 관점으로 다가가기로 하고 10개의 문제를 준비해갑니다. 이후 차차 불어나 23가지로 늘었는데, 그 중에는 수학적으로 너무 약하게 정의되었거나 수학의 영역을 벗어났다거나 하는 옥의 티가 있기도 했지만, 적어도 수학계가 지향해야 할 미래의 발전방향이라는 주제에는 들어맞았기에 큰 반향을 일으키는 데 성공합니다. 정수론, 미분방정식, 집합론, 군론, 수리물리학, 다변수함수론 등 수학의 거의 모든 분야에 걸친 23문제들 중에는 수학에 관심이 있다면 한 번쯤 들어보았을 ‘연속체 가설’, ‘리만 가설, 골드바흐 추측, 쌍둥이 소수 추측 등 소수 관련 난제’, 수학과 논리학을 결부시키려는 힐베르트의 성향을 반영하는 ‘물리학 공리의 수학적 표현’, ‘산술 공리의 무모순성’ 등이 있었고, 지금에 와서는 상당수가 해결된 상태입니다.
2. 그 외 업적 및 Comment
비록 힐베르트 자신의 학문적 목표였던 ‘모순 없이 완전한 공리계’의 존재성의 증명은 쿠르트 괴델의 불완전성 정리로 인해 산산이 깨졌지만, 힐베르트가 당대에 존재했던 수학에 대한 거의 모든 지식을 가지고 큰 논란 없이 중요성의 경중을 따질 수 있었다는 점과, 수학계의 미래를 성공적으로 제시했다는 점으로 미루어 판단할 때, 힐베르트가 20세기 수학계에 미친 영향은 그야말로 거대했다는 점을 알 수 있습니다. 무엇보다도 21세기의 1/5을 넘기는 지금 시점에서도, 수학계에 힐베르트와 같은 수준의 영향력을 발휘하고 있는 학자가 없다시피 하다는 점이 이를 더 부각시킵니다.
Ch. 4 컴퓨터 수학 (21c)
1. 컴퓨터 수학
1976년에 처음으로 시도된 컴퓨터를 사용한 증명, 4색 정리는 수학계에 일대 파란을 몰고 왔습니다. 대부분은 부정적인 반응이었지만, 또 다른 방법론의 발견이기도 했습니다. 경우의 수가 어쨌든 유한하기만 하면, 다 프로그램 돌려버리면 된다! 식의 접근법은 이후 그다지 답보가 없던 여러 난제들에 대한 폭넓은 새로운 시도들을 가능하게 했습니다.
2. 컴퓨터 수학의 활용과 Comment
2019년 지금까지 컴퓨터를 동반한 증명은 차차 늘어가고 있습니다. 힐베르트의 23문제 중 하나이기도 한 300년짜리 난제인 케플러 추측은 21세기를 목전에 두고 1998년에 컴퓨터를 통해 증명되었습니다. 일본식 수학 퍼즐 스도쿠의 해답이 유일하기 위한 최소 조건으로 17개의 숫자가 제시되어야 한다는 스도쿠 문제가 2012년에 컴퓨터를 동반하여 해결되었고, 7 이상의 모든 홀수는 세 개 이상의 소수의 합으로 표현된다는 골드바흐의 약한 추측은 약 10의 30승 이상의 수에 대해서는 논리적으로, 그 이하의 수에 대해서는 전적으로 컴퓨터로 증명되었습니다.
컴퓨터를 통한 증명이 점점 많아질수록 왜 이 증명이 옳은지 이해하기 위해서 컴퓨터를 알아야 할 필요성이 커지고 있습니다. 직관적 통찰력이나 새로운 개념을 창조하는 창의성의 감퇴, 혹은 성능적 결함이나 프로그램 오류로 인한 증명의 불확실성 등 컴퓨터를 이용한 증명의 신뢰성이나 수학 발전의 가능성에 대해 의문과 우려를 표하는 목소리가 버젓이 현존하는 것은 사실이지만, 압도적인 연산력이란 확연한 장점은 컴퓨터와 수학의 관계를 더욱 확고히 해나가고 있습니다.
Reference
1. https://en.wikipedia.org/wiki/Leonhard_Euler
2. https://en.wikipedia.org/wiki/Carl_Friedrich_Gauss
3. https://en.wikipedia.org/wiki/David_Hilbert
4. https://en.wikipedia.org/wiki/Gaussian_integral
5. https://en.wikipedia.org/wiki/Quadratic_reciprocity
6. https://en.wikipedia.org/wiki/Least_squares
7. https://en.wikipedia.org/wiki/Hilbert%27s_twenty-third_problem
8. https://en.wikipedia.org/wiki/Computer-assisted_proof
9. https://www.mathpages.com/home/kmath056/kmath056.htm
10. https://www.britannica.com/science/modular-arithmetic
11. https://www.storyofmathematics.com/20th_hilbert.html
Ch. 5 Final Commnet
- 강민석
오일러가 추측했던 이차잉여의 상호정리가 가우스에 의해 증명되거나, 오일러가 발표했던 대수학의 기본정리의 오류가 가우스에 의해 지적되거나, 오일러가 증명에 실패해 유명해진 골드바흐 추측이 힐베르트 문제에 언급되고, 가우스가 생각했던 비유클리드 기하학 때문에 유클리드의 공리를 더욱 엄밀히 할 필요가 생겨 힐베르트가 기하학을 더 엄밀히 공리화하는 등, 세 수학자는 각각의 위치에서 다음 시대의 수학에 영향을 주었고, 업적을 계승하고 보완해 왔습니다. 그리고 다음 세대인 지금의 수학계가 계승해야 하는 업적이 지금의 밀레니엄 문제로 확연히 주어졌습니다. 근대 이래로 수학은 이런 식으로 발전해 왔고, 앞으로도 이렇게 발전하리라 생각합니다.
- 이헌수
오일러, 가우스, 힐베르트에 대해 알아보며 그들은 많은 공통점을 가지고 있다는 것을 발견했습니다. 그들은 각각 그 시대에 없던 새로운 학문을 창조하는데 큰 공헌을 했습니다. 이전까지와는 다른 방식으로 수학에 접근하여 새로운 수학의 장을 열었을 뿐 아니라 수학이 아닌 다른 학문에도 새로운 접근법을 제시해주었습니다. 이는 당대 학문에 엄청난 영향을 끼치고 트렌드가 되어 다른 학자들이 관련 연구를 하며 더 깊은 지식을 생산할 수 있게 했습니다. 이런 큰 업적을 남기면서도 죽을 때 까지 연구를 손에서 놓지 않았다는 점도 그들의 공통점입니다. 가우스의 경우 중간에 암흑기가 있었지만 결국 다시 연구를 지속했습니다. 힐베르트 이후 한동안 엄청난 수학자라고 불릴만한 인물이 나오고 있지는 않지만 학문은 한번 씩 천재들이 등장하여 비약적으로 발전해왔었고 이번에 나올 위대한 인물은 컴퓨터의 도움까지 받아 더 큰 발견을 할 수 있지 않을까 생각합니다.
2019 Spring 수학사 프로젝트
Title: Sage를 이용한 고등수학교육
김기헌, 조영은
2019.06.09.
■ 목차
Ⅰ. 연구동기
Ⅱ. 연구 목적
Ⅲ. 연구 내용과 방법
Ⅳ. 도출한 결론 (예상)
Ⅴ. Sage 소개 및 기본 명령어
Ⅵ. 고등학교 수학에의 Sage 적용
1. 이차곡선- 타원
2. 이차곡선- 쌍곡선
3. 공간도형, 방정식
Ⅶ. 결론/후기
Ⅰ. 연구동기
평소에 기계적인 반복과 암기의 수학 학습방법에 부정적인 인식을 가지고 있었습니다. 그런데 때마침 수학사 수업시간에 교수님이 올려주시는 자료 중 하나인 ⑴Sage 프로그램이 눈에 들어왔고, 이를 고등학교 수학에 적용하면 좋은 결과를 도출 할 거라 예상하였습니다. 또한 수학사 수업시간에 활용한 ⑵PBL(Problem-based learning)보고서를 접목하면 수학교습 방법에 긍정적인 반향을 일으킬 거라 생각하였습니다. 그 이유는 PBL보고서와 Sage 프로그램을 통한 수업은 능동적이고 자기주도적인 학습방법이 가능하도록 도와주며, 질문과 궁금한 점을 항시 물어볼 수 있기 때문입니다. 뿐만 아니라 답을 도출 하는 것보단 다른 학생들과의 토론과 문제에 초점을 맞추게 되기 때문에 학업에 대한 부담감이 줄어 들 거라 생각합니다.
이러한 21세기 수학교육 모델을 학습에 적용한다면 학생들은 실습을 통해 수학 교과목과 코딩 지식울 습득하여 4차 산업혁명을 대비한 인재로 성장할 수 있을 것입니다.
(3) [표 1] 기존의 수학교육과 21세기 수학교육 모델 비교
항목 |
기존의 수학교육 모델 |
21세기 수학교육 모델 |
교재 |
종이(서책형) 교과서 |
전자책(e-book), 웹 링크・코딩・시뮬레이션・계산 기능 등이 자유로운 디지털 교과서 |
실습실 |
PC가 설치된 전산실 |
Wi-Fi가 가능한 모든 장소, 본인의 노트북・태블릿・스마트폰 등을 기자재로 이용 |
강의실 / 강의 |
칠판이 있는 강의실 / 판서로 강의 |
인터넷 PC와 스크린 및 칠판이 있는 강의실 / 인터넷 자료와 클라우드 컴퓨팅을 통한 시뮬레이션 및 수학계산 과정이 포함된 강의(녹화하여 언제든지 다시 볼 수 있으며 학생과 공유 가능) |
학습활동 |
교과서의 과제풀이 / 조교(TA)의 튜터링 |
디지털 교과서와 LMS를 활용하여 예습・질문・토론・과제 모두를 포함한 과제풀이 / 개인별로 자신의 학습기록을 모아 PBL 보고서로 작성・제출 및 발표 |
평가 |
출석・과제・지필고사 합산 |
온/오프라인 출석・디지털활동참여・지필고사・PBL보고서 등 모든 활동의 결과물에 가중치를 주어 합산 |
⑴http://matrix.skku.ac.kr/Cal-Book/index.html
⑵https://ko.wikipedia.org/wiki/%EB%AC%B8%EC%A0%9C%EC%A4%91%EC%8B%AC%ED%95%99%EC%8A%B5
(3)Trend of Technology in Math Education-코딩교육과 수학
Ⅱ. 연구 목적
1. 21세기 디지털 첨단 모바일/인터넷 시대와 함께 4차 산업혁명 시대의 도래는 대학수학교육에서 기존과 다른 혁신적인 교육을 하도록 요청하고 있으며, 코딩교육에 대한 관심 또한 날로 증가하고 있습니다. 21세기 수학교육 모델을 수업 현장에 적용한다면 기존의 수학 교과목을 배우면서 자연스럽게 파이썬과 R등 코딩에 필요한 지식을 습득하게 됩니다.
2. 이런 경험을 가진 학생들은 실제로 프로젝트 기반 학습을 통해 다양한 현실 문제들을 다 루어 본 경험을 갖게 되어 차별화된 인재로 성장할 수 있습니다. 이는 4차 산업혁명을 대비한 인재 양성에 부합합니다.
3. Sage 프로그램을 사용함으로써 계산 및 코딩과정에서 시간이 절약되어 학생들이 교사 및 다른 학생들과 질문, 답변, 토론하며 개념에 대해 깊게 이해하고 학습할 수 있습니다.
4. 수학공부도 실습을 통해 진행할 수 있습니다. 이를 통해 수학이 지루하다고 생각하여 수학을 포기한 학생들, 일명 ‘수포자’를 감소시킬 수 있습니다.
(1)http://www.edaily.co.kr/news/NewsRead.edy?SCD=JG21&newsid=02742086609437144&DCD=A00702&OutLnkChk=Y
Ⅲ. 연구 내용과 방법
기존에 이상구 교수님께서 개발하신 ‘Sage를 통한 미적분학’을 참고하여 고등학교 수학과목 수준에 적합한 Sage 프로그램을 구상하였습니다. 또한 고등학교 수학교육에 PBL 방식을 적용하는 것에 대해서도 논의할 것입니다.
고등학생 때 배우는 미적분1,2, 확률과 통계, 기하와 벡터 중에 어느 부분을 Sage프로그램에 적용할지 고민을 했습니다. 확률과 통계는 비교적 학생들이 쉽게 생각하는 영역이고, 미적분학과 기하와 벡터를 어려워하는 학생들이 많으므로 이 둘 중에 기하와 벡터를 선택했습니다. 기하 영역을 프로그램화한다면 공간지각능력이 부족한 학생들이 시각화한 공식을 접함으로써 이전보다 훨씬 빠르게 개념을 이해할 수 있을 것이라고 생각하였습니다. 기하와 벡터 영역 중에서는 시각화의 효과가 두드러지는 이차곡선과 공간도형을 프로그램화 할 것입니다.
고등 수학교육에 PBL 방식을 적용할 때, 교사가 문제를 제시하거나 학생이 질문을 하면 다른 학생들이 그에 대한 자신의 생각을 추가적으로 이어나가는 것이 핵심입니다. 교사는 학생들의 토론방향이 주제와 빗나가지 않도록 유도하고, 학생들이 참여에 흥미를 가질 수 있도록 동기를 만들어주는 역할을 합니다. 동기로는 학생들이 흥미를 가지는 일상소재를 토론에 끌어온다던가, 수행평가 점수를 토론에 적용할 수 있습니다.
Ⅳ. 도출한 결론(예상)
1. 수학공부도 실습을 적용할 수 있다는 점에서 학생들의 지적호기심을 불러일으킬 수 있습니다.
2. Sage를 활용함으로써, 손으로 그리기 어려운 복잡한 그래프라도 어떤 개형을 가질지 한눈에 확인할 수 있습니다.
3. Sage를 활용하면 여러 숫자만 바뀐 비슷한 문제들을 반복적으로 풀어야 하는 지루함에서 벗어날 수 있습니다.
4. 공간지각능력이 부족하여 도형이 어떻게 보일지, 어떻게 변화할지 상상이 안가는 학생들이 큰 도움을 받을 수 있고, 나중에는 공간지각능력이 향상되어 프로그램이 없더라도 쉽게 상상할 수 있을 것입니다.
5. QnA를 활용하면 일방적인 주입식 교육이 아니라, 양방향적인 교육이 진행되므로 학생들의 자기 주도적 학습능력이 향상되고 자신의 의견을 말하는 능력이 향상될 것으로 예상됩니다.
Ⅴ. Sage 소개 및 기본 명령어
수학 도구는 오랫동안 교실에서 중요한 위치를 차지하고 있었다. 정보통신기술(ICT)의 혁신으로 많은 도구들이 등장하고 교육 목적으로 채택되었다. 세이지(Sage)는 2005년에 출시된 인기 있는 수학 소프트웨어다. 이 소프트웨어는 인터넷을 통해 활용할 수 있는 효율적인 기능을 갖추고 있으며 선형대수학, 추상대수학, 조합론, 숫자론, 기호 연산, 숫자 수학, 미적분학 등 대부분의 수학 문제를 다룰 수 있다. 이번 프로젝트를 통해 우리는 이 강력한 소프트웨어를 소개하고 그것이 수업에서 어떻게 사용될 수 있는 지에 대해 이야기할 것이다.
세이지(Sage)는 온라인에서 쉽게 사용할 수 있는 수학 컴퓨터 대수 체계(CAS)이다. 세이지 노트북은 2008년 4월 미국 워싱턴 대학교에서 발매되었다. 자유롭고, Mathematica, Maple 또는 Matlab과 같은 값비싼 상용 소프트웨어와 견주어도 손색없는 강력한 기능을 가지고 있다. 단, 프로그램을 별도로 설치할 필요는 없다. 이것은 웹 Mathematic에 가깝지만 몇 가지 더 나은 특징을 가지고 있다. 어떤 웹브라우저에 접속하는 것만으로 거의 모든 미적분 문제를 해결할 수 있다. 우리는 문제를 해결하기 위해 필요한 명령어를 쉽게 찾을 수 있다. Sage 노트북은 많은 오픈 소스 및 상업용 프로그램과 접점을 만들 수 있는 플랫폼이다.
Sage는 서버용 무료 수학연산 프로그램으로서, 사용자의 PC에 직접 설치할 필요가 없이 인터넷 환경만 제공되면 언제 어디서나 사용이 가능하다. 특히 Sage를 이용하여 워크시트(worksheet)를 제작하고, 그 워크시트에서 Sage명령과 수학연산에 적합한 파이선(python) 명령어들을 직접 실행해볼 수 있다.
Sage는 기본적으로 웹을 통해 서비스되기 때문에, Sage를 이용하여 제작된 워크시트 또한 웹을 통해 다른 사람들과 공유할 수 있다. 따라서 새로운 수학연산을 시도할 때 그 수학연산에 대한 Sage 명령어의 지식이 부족하다 하더라도, 웹을 통해 이미 공개된 다양한 자료들을 통해 Sage는 그 해법을 제시할 수 있다.
Sage의 수많은 명령어 및 새롭게 개발된 함수들은 구글(Google)과 같은 웹 검색 사이트를 이용하여 관련된 검색이 가능하다. 그 명령어들을 단순히 복사하여 붙여 넣는 방식만으로도 새로운 수학연산을 할 수 있다. 또한 이와 같이 다양하게 공개된 워크시트의 자료들을 <표 1>의 주소에서 한 번에 얻을 수도 있다.
Sage의 대부분의 명령어 알고리즘들은 새롭게 개발된 것들이 아니라 <표 2>와 같이 이미 지난 40여 년 동안 수학의 각 분야에서 유용하다고 검증된 오픈소스 프로그램들의 함수들이 파이선 형식의 명령어로 다시 종합되었다. 따라서, 우리는 이와 같이 강력한 수학연산을 제공하는 Sage를 이용하여 제작된 공학적 도구를 사용하여 대수적인 답을 찾을 수 있고 Sage의 상호작용 함수(interact function)를 활용하여 다양한 시뮬레이션을 실행할 수도 있다. 이렇듯 Sage는 한국형 CAS 도구로써 공학적 도구 도입의 단점을 모두 극복하여 새로운 도구의 대안이 될 수 있다. 게다가 이미 Sage의 한글화에 성공하여 언어의 제약도 따르지 않는다. 또한, 새롭게 Sage 명령어를 익히지 않고서도 기존의 CAS 도구(Mathematica, Matlab, Maple 등)들의 언어를 그대로 Sage에서 사용할 수도 있다.
<Sage 기본적인 명령어>
1) 정의
● 변수 x var(‘x’)
● 벡터 <1, 2, 3> vector([1, 2, 3])
● 벡터함수 vector([2-5*t, 4*t, 1+3*t])
● 행렬 matrix([[1, 2], [3, 4]])
● 값 정의(예: a=3) a=3
● 함수의 정의 f(x)=x^2
2) 벡터와 행렬
● 벡터의 크기 a.norm()
● 벡터의 내적 b.dot_product(a)
● 벡터의 외적 b.cross_product(a)
◆ 행렬식 det(A)
◆ 행렬의 크기 len(A)
◆ 행렬의 고유값 A.eigenvalues()
◆ 행렬의 고유벡터 A.eigenvectors_left()
3) 미분과 적분
● 도함수 diff(f(x), x)
◆ 부정적분 integral(f(x), x)
◆ [a, b]에서 적분 integral(f(x), x, a, b)
◆ 이중적분 integral(integral(f(x), (x, 0, 1)), (y, 0, 1))
◆ 삼중적분 integral(integral(integral(f(x), (x, 0, 1)), (y, 0, 1)), (z, 0, 1))
4) 그래프 그리기
● 일반함수(2D) plot(f(x), (x, -4, 4))
● 매개변수함수(2D) parametric_plot((f(x), g(x)), (x, 0, 2*pi))
● 음함수(2D) implicit_plot(f(x)==0, (x, -2, 2), (y, -2, 2))
● 선분(2D) line([(1, 1), (2, 2)], color='red')
● 일반함수(3D) plot3d(f(x, y), (x, -2, 2), (y, -2, 2))
● 매개변수함수(3D) parametric_plot3d((f(x), g(x), h(x)), (x, 0, 2*pi))
● 음함수(3D) implicit_plot3d(f(x, y, z)==5, (x, -3,3), (y,-3,3), (z,-3,3))
● 선분(3D) line([(1, 1, 1), (2, 2, 2)], color='red')
● Contour Plot contour_plot(f(x, y), (x, -1, 1), (y, -1, 1), cmap='hsv', labels=True)
● 벡터필드(2D) plot_vector_field((x+y, x), (x, -3, 3), (y, -3, 3))
● 벡터필드(3D) plot_vector_field3d((0, 0, 1), (x, -3, 3), (y,-3,3), (z,-3,3))
5) 극한
● a에서 극한 limit(f(x), x=a)
● a에서 우극한 limit(f(x), x=a, dir='+')
● a에서 좌극한 limit(f(x), x=a, dir='-')
● 에서의 극한 limit(f(x), x=+oo)
● 에서의 극한 limit(f(x), x=-oo)
6) 기타
● 방정식 풀이 solve(f(x)==0, x)
● 급수 sum((p(n), n, 1, +oo)
● 변수 a의 값 보이기 print a
● 정의된 함수 f 보이기 show(f)
● 조건문 if 조건:
statement(s) # 조건이 True이면 실행할 명령어
else:
statement(s) # 조건이 False이면 실행할 명령어
● 반복문 for i in range(10):
print i # i=0~9 까지 i를 출력
(1)http://matrix.skku.ac.kr/Lab-Book/Sage-Lab-Manual.htm
Ⅵ. 고등학교 수학에의 Sage 적용
1. 이차곡선 - 타원
1) 타원의 정의
평면 위의 서로 다른 두 점 에서의 거리의 합이 일정한 점들의 집합을 타원이라 하고, 두 점
을 타원의 초점이라 한다. 그림과 같이 두 초점
을 지나는 직선이 타원과 마나는 점을 각각
이라 하고, 선분
의 수직이등분선이 타원과 만나는 점을 각각
이라 할 때, 네 점
을 타원의 꼭짓점, 선분
을 타원 장축, 선분
을 타원의 단축, 장축과 단축이 만나는 점을 타원의 중심이라 한다.
2) 타원의 방정식
(1) 두 초점 ,
에서의 거리의 합이 2a (a>c>0)인 타원의 방정식은
(단,
, b>0)
① 장축의 길이 : 2b, 단축의 길이 : 2a
② 초점의 좌표 : ,
③ 꼭짓점의 좌표 : (a, 0), (-a, 0), (0, b), (0, -b)
(2) 두 초점 ,
에서의 거리의 합이 2b (b>c>0)인 타원의 방정식은
(단,
, a>0)
① 장축의 길이 : 2b, 단축의 길이 : 2a
② 초점의 좌표 : ,
③ 꼭짓점의 좌표 : (a, 0), (-a, 0), (0, b), (0, -b)
<Sage를 이용해 타원의 방정식 그려보기>
<SKKU Sage Grapher를 이용한 타원 그리기>
2. 이차곡선 – 쌍곡선
1) 쌍곡선의 정의
평면 위의 서로 다른 두 점 에서의 거리의 차가 일정한 점들의 집합을 쌍곡선이라 하고, 두 점
을 쌍곡선의 초점이라 한다.
그림과 같이 두 초점 을 지나는 직선이 쌍곡선과 만나는 점을 각각
이라 할 때, 두 점
을 쌍곡선의 꼭짓점, 선분
을 쌍곡선의 주축, 선분
의 중점을 쌍곡선의 중심이라 한다.
2) 쌍곡선의 방정식
(1) 두 초점 ,
에서의 거리의 차가 2a (c>a>0)인 쌍곡선의 방정식은
(단,
)
① 주축의 길이 : 2a
② 초점의 좌표 : ,
③ 꼭짓점의 좌표 : (a, 0), (-a, 0)
(2) 두 초점 ,
에서의 거리가 차가 2b (c>b>0)인 쌍곡선의 방정식은
(단,
)
① 주축의 길이 : 2b
② 초점의 좌표 : ,
③ 꼭짓점의 좌표 : (0, b), (0, -b)
<Sage를 이용해 쌍곡선의 방정식 그려보기>
<SKKU Sage Grapher를 이용한 쌍곡선 그리기>
<SKKU Sage Grapher를 이용한 타원과 쌍곡선 그리기>
3. 공간도형 방정식
1. 평면의 방정식
점 을 지나고 영벡터가 아닌 벡터
에 수직인 평면의 방정식은
이다. 이때 벡터 를 이 평면의 법선벡터라 한다.
2. 구의 방정식
1) 구
공간에서 한 정점으로부터 일정한 거리에 있는 점 전체의 집합을 구라 한다. 이때 정점을 구의 중심, 구의 중심과 구 위의 한 점을 이은 선분을 구의 반지름이라 한다.
2) 구의 방정식
좌표공간에서 중심이 점 C(a, b, c)이고 반지름의 길이가 r인 구의 방정식은
특히 중심이 원점이고 반지름의 길이가 r인 구의 방정식은
<Sage를 이용해 면과 구의 방정식 그려보기>
(1)http://matrix.skku.ac.kr/Mobile-Sage-G/sage-grapher-imp.html
(2)http://matrix.skku.ac.kr/Lab-Book/Sage-Lab-Manual-1.htm
Ⅶ. 결론/후기
조영은
이번 프로젝트를 준비하면서 고등학생 때로 돌아간다면 어떤 공부방식이 도움이 될까라는 생각을 자주 했습니다. 고등학생 때 수학공부는 공식을 외우고, 문제만 많이 푸는 게 전부였습니다. 대학에 입학하여 이산수학, 수학사 강의를 수강하면서 알게 된 PBL 기반의 학습을 고등학교에 적용하면 어떨까라는 생각이 들었습니다. 문제의 답을 맞추는 것에 초점을 두는 게 아니라 문제에 대해 교사, 다른 학생들과 함께 다양한 풀이에 대해 생각하는 과정이 평가기준이 된다면 자연스레 사교육의 열기 또한 감소할 것입니다.
김기헌 학우님께서 Sage와 명령어에 대한 설명을 정리하셨고 이차곡선(타원, 쌍곡선)과 공간도형에 대한 내용을 Sage 포로그램에 적용하는 역할을 맡으셨습니다. 저는 연구동기와 목적, 연구 내용과 방법, 도출결론을 정리하였습니다. 이상구 교수님께서 프로젝트에 필요한 자료를 공유해주셔서 올바른 방향으로 보고서를 작성할 수 있었습니다.
보고서를 마무리하면서 고등교육 방식이 하루빨리 바뀌어야 한다는 생각이 점점 확고해졌습니다. 교육방식이 바뀌려면 입시체제가 먼저 바뀌어야 할 것입니다. 지금 학교현장에 Sage 프로그램과 PBL교육방식을 도입한다면, 수능점수와 내신점수 챙기기에 조급한 학생들은 ‘진정한 학습’보다는 점수를 잘 받기 위한 최소한의 노력을 투자하기에 바쁠 것입니다. 이 방식이 완전히 자리를 잡기에는 상당한 시간이 걸리겠지만, 21세기 교육방식이 제대로 적용된다면 학생들의 학습효율과 사고의 확장에 큰 도움이 될 것이 분명합니다.
김기헌
프로그래밍 능력이 갈수록 중요시되고 있는 시대입니다. 아이들은 초등학교에 입학하기 무섭게 코딩 학원을 찾으며 일반 기업에서도 사원에게 기본적인 코딩 능력을 요구하고 있습니다. 그런 면에서 Sage란 도구는 교육적인 측면에서 여러 장점을 갖고 있습니다. 수학과 코딩의 두 가지 성격을 모두 갖고 있기에 하나를 학습하면 두 개의 학문을 모두 습득할 수 있는 것입니다. 이를 기초 교육에 적용한다면 학생들이 느낌 부담을 줄이면서도 학습 효과는 극대화할 수 있겠다는 생각이 들어 이번 PBL Project 주제로 선정해 연구를 진행했습니다.
2인 1조로 프로젝트를 진행함에 있어 어려움이 없었다고 말할 수는 없지만 앞서 조영은 학우님께서 언급했듯이 적절한 역할 분배와 지속적인 의사소통 및 의견 교환을 통해 큰 갈등없이 이번 Project를 마무리할 수 있었습니다. 이 보고서를 성공적으로 진행하고 완성할 수 있게 언제나 많은 관심과 도움을 주신 이상구 교수님께도 감사의 인사를 드리고 싶습니다.
한 가지 아쉬운 점이 있다면 교육자의 입장이 아닌 수업을 듣는 당사자, 즉 학생들의 입장이나 반응을 살필 수 없었다는 것에 있습니다. 대학생인 우리가 의견을 물을 적절한 대상을 찾기엔 무리가 있었기에 학생들의 의견을 수렴할 수는 없었습니다. 하지만 단언컨대 학생들은 기존의 지루한 교육방식을 탈피한 Sage의 사용을 환영할 것이라 생각합니다.
세상은 더욱 더 빠르게 변화하고 있으며 우리는 그 변화에 적응해야 합니다. 그렇지 않으면 구시대에 도태될 수밖에 없기 때문입니다. 비록 정규 교과 과정에서 Sage가 활용될 수 없더라도 Sage의 가장 큰 장점인 오픈 플랫폼을 이용해 보다 많은 학생들이 이 강력한 도구를 활용할 수 있게 되는 날이 오기를 바라봅니다.
◆ Final comment
PBL보고서를 마치고 (5점)
3월 : 내용들은 복습하도록 하겠습니다.
이렇게 질문은 많이하고 답변을 많이 하는 적극적으로 참여하는 수업을 마지막 학기가 되어서야 처음 듣게되었다. 전문지식 부족으로 높은 퀄리티의 대화와 토론은 힘들지만 수업이 진행되면 중간중간 의문점을 제시하고 비전문가 입장에서 바라보는 새로운 시각과 의견을 제시하며 다른 학생들과 함께 하였다. 아쉬웠던 것은 적극성이 부족했다는 점이다. 깊이 있는 내용이나 정답이 아니더라도 적극적으로 의견을 제시하고 질문했던 것들에 대한 Final이 필요하다.
4월 : 수학사를 들으며 점점 느껴지는 것이 수학이라는 학문이 실생활에도 많은 영향을 주고 반대로 영향을 받기도 하는 유동적인 학문이라는 것이다. 마냥 수와 식과 형태 같은 것의 딱딱한 연구라고만 생각했었는데 점점 그 편견이 깨지고 있다. 이 수업은 마치 여행과 같다고 느낀다. 어떤 나라를 여행할 때 그 나라의 특징, 역사, 문화 등을 미리 알아보고 가서 직접 느껴보며 많은 것들을 배워가는 것처럼, 교수님과 또 학우들과 함께 수학사를 알아보고 토론하고 느껴보며 많은 것들을 배우고 더 넓은 시야를 가지게 되는 것 같다. 앞으로도 같이 수업을 듣는 학우들과 더 많은 주제, 많은 수학자들에 대해 알아가며 더 넓은 시야로 수학사를 바라보고 싶다.
5월 : 수학사 과목을 듣기로 했을때가 생각이난다. 교수님께서 수학사 과목은 부담스럽지 않고 즐거운 수업이 될 것이다 라고 말씀하셨다. 하지만 막상 수업을 처음 듣기 시작했을 때 익숙하지 않은 수업과 학습방법이 부담스럽고 그렇게 즐겁지도 않았다. 하지만 이젠 이 수업방식이 적응되어 가는 것 같다. 새로운 것을 스스로 찾아보고 타인과 의견을 교환하며 배우는 것이 교수님 말처럼 즐거워지고 learn how to learn을 조금씩 알아가는 것 같다.
Final 6월 : 3달이란 시간이 금방 지나갔다. 하지만 이 짧은 시간에 많은 것이 변했다. 수업시간에 얻은 것은 단순한 수학사 지식 뿐만이 아니다. 물론 수업을 듣기 전과 비교했을 때 수학사에 대해서도 시간에 따라 이야기 할 것도 많아졌고 그에 파생된 새로운 지식들도 많아졌다. 하지만 가장 큰 소득은 나의 변화이다. 어떤 문제를 바라볼 때 지금까지 내가 알고 들어왔던 것만을 기준으로 판단했는데, 그러다 보니 편협한 생각을 가진 것들이 많았다 예를들어 내가 생각하던 것들이 비정상인 것이었는데 계속 그 기준에서 생각하려다 보니 제대로 된 된 생각을 하기 힘들었다. 다른사람들과 의사소통도 하고 우리나라나 우리의 환경에 국한되지 말고 폭넓은 시야로 생각하고 행동하는 것이 좋다는 것을 알게되었다. 학습에 있어서도 지금까지의 수동적인 면을 버리고 능동적으로 하려도 노력하는 계기가 된 것이 이 수업이다.
3월 :
세계 수학사 뿐만 아니라 세계사에도 무관심했던 터라 아랍이라는 곳에 대해 다시 생각하게 되었다. 수학의 중요성을 미리 깨닫고, 그것을 발전시킨 문명이 있는데, 우리나라는 그에 준하는 발전된 수학을 갖고 있었음에도 서양에 뒤처지게 된 역사가 아쉽다.
중국의 범국가적 프로젝트를 보며 우리나라 또한 계획적으로 수학의 발전을 준비해야 한다고 느낀다. 정책적으로 관여하기는 힘들겠지만, 내 위치에서 최선을 다해야겠다.
4월 :
고대의 수학이 아니라 현대의 실생활에 관련되어있는 수학의 역사를 배우니 좀 더 흥미로웠다. 특히 확률의 발전에서 지금 중학생도 알 수 있는 기본적인 확률의 개념도 없었다는 것이 충격적이었다. 또한 확률을 배울 때, 큰 수의 법칙이 무엇인지 이해는 하면서도, 이게 법칙이라고 할 만한 것인가 의문이 들었었는데, 그 의문이 해소되었다.
바쁘다는 핑계로 질문과 토론을 소홀히 했던 내 자신을 반성한다.
5월, 6월 :
많이 참여하지 못했지만, 그때마다 흥미로운 주제가 있었다. 통계학을 통한 빅데이터의 활용과 문문법이 특히 인상적이었는데, 앞으로 우리가 살아가게 될 세상의 변화를 미리 경험한 것 같다는 생각이 든다. 사실 이러한 세상의 변화는 모두 기술의 발달이라고만 생각했었는데, 기술발전만큼 커다란 역할을 하고 있는 것이 수학 이라는 것을 깨달았다.
또한 프로젝트를 진행하면서, 넘치는 자료를 취합하고, 정리하여 새로운 지식을 생산해내는 경험은 앞으로 내게 꼭 필요할 것 같다.
더 성실히 참여하지 못한 것이 아쉽지만, 이 수업에서 정말 많은 것을 얻어가는 것 같다. ■
3월
수학사 강의를 통해 수학적 개념과 세계 역사를 동시에 배우는 점은 정말 좋은 것 같습니다. 분명히 수학과목이지만 인문학 강의를 듣는 기분이 들어서 재밌고, 교수님이 수업 중간 중간에 보여주시는 각 나라의 사진들을 볼 때면 ‘나도 나중에 저 나라들을 꼭 여행해봐야지’라는 생각이 들기도 했습니다. 6번의 수업을 듣고 나니 3월이 끝나버렸는데, 앞으로의 수업도 기대되고, 더욱 재밌는 수학사에 대해 배우고 싶습니다.
4월
5주차, 6주차 수업은 흥미로운 주제가 많았습니다. 그래서 4월에는 이전보다 흥미로운 QnA 주제가 많이 나온 것 같습니다. 1차 PBL 레포트를 제출한 뒤, 앞으로는 QnA 활동을 꾸준히 해야겠다고 다짐했던 것과는 달리, 4월 초반에 활동을 자주 못한 것이 아쉽습니다. 그래도 두 번째 온라인 토론은 이전보다 활발한 토론이 오고간 듯해서 앞으로 남은 수학사 시간도 기대가 됩니다.
5월
5월 첫 수업은 야외 수업이 있었습니다. 대학에 입학하여 야외 수업은 처음이라 신기했고, 일월 호를 거닐며 교수님께 수학사 이야기를 듣는 것이 재밌었습니다. ‘실베스터’, ‘나이팅게일’ 등의 수학사 이야기들은 산책을 하면서 들어서 그런지, 지금도 기억에 잘 남아있습니다.
4월 PBL레포트를 발표할 때 ‘다음에는 더 성실히 QnA활동에 참여하겠다.’고 다짐했었는데, 이 약속이 이번에도 잘 지켜지지는 않은 것 같습니다. 교수님께서 수업시간을 이용해 온라인 토론을 하게 해주셔서 해당 시간을 활용해 몰아서 참여하곤 했습니다. 그래도 이 시간 동안 학우님들의 다양한 생각을 들을 수 있었고 확실히 처음 온라인 토론을 할 때보다 더 많은 빈도로 질문과 답글이 오고 간다는 것을 느낄 수 있었습니다.
6월
프로젝트 활동을 본격적으로 진행하였습니다. 처음에 생각했던 프로젝트 주제는 수학이 쓰이는 분야와 그 예시에 대해 알아보는 것이었는데 어느 분야를 얼만 큼 조사해야 할지 감이 안 잡혀 교수님의 조언을 얻었습니다. 교수님의 말씀을 통해 주제의 방향을 잡을 수 있었고 프로젝트도 원활히 마쳤습니다. 개강한 날이 엊그제 같은데 벌써 종강을 마주하고 있네요. 이번학기에는 같이 수강하는 학우 분들의 수도 적어서 매 수업시간마다 과외를 받는 것 같은 기분이 들었고, 대학 입학 후 처음으로 일월 호 공원에서 산책하면서 수업도 들을 수 있었습니다. 동양수학사와 서양수학사에 대한 이야기들도 무척 재밌었습니다. 특히 조선 후기나, 일제강점기 그리고 현재 교육의 방향에 대해 많은 얘기를 해주셨는데 교수님의 말씀대로 우리나라가 현재어느정도의 선에서 더 이상 못 올라가고 있는 이유에 대해 생각해볼 수 있었고 이 문제를 해결하려면 많은 것이 바뀌어야 한다는 것도 인식하였습니다. 역사와 현재의 연관성에 대해 따져볼 수 있는 수업이었습니다. ‘역사는 암기과목’이라고 생각해서 좋아하지 않았던 저였지만, 이번 수학사 강의에서 이야기 형태의 수업을 들으며 재미를 느낄 수 있었고 더불어 세상을 더 크게 바라보게 되었습니다. ■
3월 : 지금까지 묻고 답한 것들을 정리하는 느낌으로 썼습니다. 의외로 배운 것들이 많았습니다. 질문에 서툰 점은 고쳐나가야겠다고 생각합니다. 답을 달 때도 간단히 알아온 사실만 짤막하게 다는 것보다는 알아볼 만큼 알아보고 한 번에 정리해서 다는 게 성미에 맞다보니 참여가 한 박자씩 계속 늦는 느낌입니다. 그래도 나름 만족할 만한 답을 달 수 있었고, 지금 와서 읽어보면 그 답을 달기 위해 알아보면서 배운 것들이 느껴져서 좋았습니다. 양은 적지만 개중 몇 가지는 나름 만족할 만한 질이 나왔다고 생각합니다.
4월 : 확률론, 통계학, 보험수학, 위상수학 모두 굉장히 생소한 내용이라 배우기 바빠 이번 달에는 질문 자체는 그다지 없었습니다. 보통은 의문이 생기면 책으로 해결하는 편이라 스스로 의문을 해결하는 데 너무 오랜 시간 익숙해진 듯도 합니다. 궁금한 게 생기면 질문을 하기 전에 먼저 책을 펼치다보니 토론에 참여가 적었고 쓴 글도 몇 개 되지 않습니다. 되돌아보면 주로 제 지식을 검증하는 용도로 Q&A를 이용하는 것 같습니다. 같은 질문에 제가 단 답변과 다른 사람이 단 답변이 어떻게 다른지 살펴보고, 어떤 부분을 내가 몰랐는지, 어떤 부분에서 차이가 있는지 비교하는 것은 매우 유의미했다고 생각합니다.
5월 : 중간에 아파서 앓아눕고 다른 공부에 이리저리 채이다 잠깐 Q&A에 소홀했던 게 정신 차리고 보니 2주가량 공백이 있었던 게 뼈아팠습니다. 질문이 적었던 것, 즉 먼저 이야기를 꺼내들고 오지 못했다는 점이 가장 큰 단점이었고, 답변은 양으로나 질로나 나름대로 괜찮게 달았다고 생각합니다. 수학이 암호, 법, 위작 판별, 예술 창작 등등 생각지도 못했던 영역으로 자꾸자꾸 넓어져가는 것이 경이롭게 느껴지기도 했습니다. 그리고 무엇보다도, Q&A를 통해 우리나라 수학 교육의 실태와 수학의 접근성을 높일 방법에 관해 깊이 생각해볼 기회가 되었습니다. ■
1st Month : 아직은 부족한 점이 많다는 것을 느꼈습니다. 수업을 정리하고 요약하는 것에 있어서는 지금까지도 많이 해왔던 부분이라 익숙했지만, 누군가와 의견을 나누고 토론을 하는 등의 활동은 경험이 많지 않기 때문입니다. 특히나 on-line 토론의 경우 이러한 경험이 전무했던지라 적응하는데 시간이 조금 걸렸습니다. 지금까지 지식을 전수받기만 하는 수업만 해왔던 저로서는 함께 공유하는 수업을 준비함에 있어서 모자란 부분이 많았다고 생각합니다. 지금까지 해온 내용을 보면 다른 학우들에 비해 그렇게 나은 점이 없다는 것이 제 결론입니다. 다음 4월부터는 조금 더 수업을 주도적으로 이끌어가고 싶습니다. 토론의 주제를 직접적으로 제시하고 필요한 자료를 능숙하게 찾아 학우분들과 함께 나누고 싶습니다.
Mid Term : 중간고사 대비 기간입니다. 변명에 변명을 더하는 것 같습니다만 솔직히 다른 전공 과목 준비에 정신이 팔려 수학사 수업에 많은 열정을 쏟지 못했습니다. 과제는 많았고 기타 여러 잡다한 문제들도 있었기 때문입니다. 그래도 잘한게 있다고 한다면 이 보고서를 제출하기 전 마지막으로 한 토론 수업이 아닐까 생각합니다. 1시간이라는 짧다면 짧은, 길다면 긴 시간동안 오롯이 수학사에 집중해 학우님들과 여러 의견을 나눴고 저 역시 답변도 하고 질문도 하면서 나름 생산적인 활동을 했다 생각합니다. 3월 보고서를 제출한 지 보름도 안 지나 4월 보고서를 제출하게 되어 가시적인 성과는 보이지 않지만 꾸준하게 쌓고 쌓아 학기 말에는 눈부신 결과물을 만들어보겠습니다.
3rd Month : 개강하고 처음으로 수업을 들으러 강의실을 찾던 게 엊그제 같은데 벌써 종강을 바라보고 있네요. 열심히 한다고는 하고 있는데 이게 맞는 방향인지를 잘 모르겠습니다. 사실 이 강의가 저와 찰떡같이 맞다고만 할 수는 없습니다. 지금까지 해 온 수업과는 방향성이 다르며 특히나 토론식 수업이 익숙치 않았기에 처음 적응하는데 어려움이 있었고 솔직히 지금도 잘 하고 있다고는 생각이 들지 않습니다. 하지만 그러한 과정 자체가 의미가 있다는걸 이제 조금씩 깨닫는 중입니다. ‘이걸 이렇게 하면 될까?’, ‘저건 저렇게 해볼까?’ 같은 고민이 쌓이고 쌓여서 그저께보다는 어제가, 어제보다는 오늘 더 나아진 제가 되어가고 있습니다. 끝이 얼마 남지 않았습니다. 마지막까지 치열하게 고민해서 사과 하나만큼의 성과를 만들 수 있다면 좋을 것 같습니다.
Last Month (Final) : 솔직히 가슴에 손을 얹고 열심히 했냐고 묻는다면 자신있게 그렇다고 대답하기엔 양심에 찔리는 게 사실입니다. 나름 노력을 많이 했지만 최선을 다 하진 못한것 같아 아쉽습니다. 다른 학우 분들과 조금 더 의견을 나누지 못했음에 아쉬움이 남습니다. 그럼에도 불구하고 타 수업에 비해 얻어가는 게 많았다고는 자신있게 말할수 있습니다. 어느 수업도 이렇게 토론이 활성화되지 못했으며 교수님과 학생들이 상호 의견 교환이 일어나지 않았습니다. 학기 초 수업을 시작하며 교수님께서 스스로 무언가를 만들어내야 한다고 하셨을 때 무슨 뜻인지 이해를 할 수 없었습니다. 아무것도 모르는 제가 무엇을 만들어낸단 말입니까. 하지만 몇 번의 토론과 PBL 보고서와 Project를 진행하며 저도 무언가를 생산해낼 수 있다는 사실을 깨달았습니다. 단순히 무언가를 아는 것과 그것을 다른 이들에게 설명할 수 있음은 다르다는 것을 깨달았고 그 설명을 통해 제가 성장할 수 있음을 깨달았습니다. 이 학교에 우리 나라에 이런 수업이 더 많이 생겼으면 하는 바람입니다. 한 학기동안 고생하신 교수님과 부족한 저와 함께 수업을 진행한 학우분들에게 감사의 인사를 전하는 바입니다. ■
After Final Exam
복학 첫 학기 수업이라 부족한 점이 많았습니다. 군대에 있는 2년이란 시간동안 머리는 굳어 있었고 떨어져 있는 학점을 올려야 한다는 부담감만 앞서 있는 상황이였습니다.
그 와중에 이상구 교수님의 수학사라는 강의를 알게 됐고 수강신청을 했습니다. 지금에서야 말 할 수 있지만 여유로운 수업이 될 것이라는 교수님의 말씀과는 다르게도 저에게는 마냥 생각없이 들을 수 있는 수업이 아니였습니다.
평소 제가 생각했던, 예상했던 수업과는 다르게 흘러 갔기에 이런 수업이 처음이였던 전 적응하기에 바빴습니다 솔직히 말해 수업 방식에 대해 의문도 많이 품었습니다. 다른 일반적인 수업과는 다르게 해야할 것들도 많았고 교수님께서 저희에게 던지는 질문도 쉽게 대답할 수 있기만 한 그런 질문은 아니였습니다. 그런 의문과 함께 한 학기를 보냈습니다.
그런데 돌이켜 보면 그러한 질문 하나 하나가 수업에서 나눈 토론이 조금씩 저의 살이 되고 피가 되었던것 같습니다. 워낙에 변화가 적었던 탓에 당장에 알 수가 없었던 것 뿐입니다. 비록 수강생의 수는 적었지만 그 안에서 나눈 의견은 지금까지 제가 들었던 어떤 수업보다 많았다고 생각합니다.
저의 의견을 나누면서 보다 다양한 의견을 나누며 생각의 지평을 넓힐 수 있었습니다.
이러한 수업이 있다는 것을 알려주시고 수업을 들을 수 있게 해주신 교수님께 다시 한번 감사의 말씀을 드리고 싶습니다.
아울러 부족한 저와 함께 의견을 나누며 함께 수업을 진행해준 학우님들께, 특히 저와 함께 조를 이루어 PBL Project를 성공적으로 완성해준 조영은 양에게도 감사의 인사를 드리는 바입니다.
모두들 한 학기 동안 고생많으셨습니다. 감사합니다. ■
1) <사진1. 레온하르트 오일러 초상화>
https://upload.wikimedia.org/wikipedia/commons/thumb/6/60/Leonhard_Euler_2.jpg/220px-Leonhard_Euler_2.jpg